SEARCH (Type the topic here)

Liver and Pancreas MCQ

1. Which of the following statements about the segmental anatomy of the liver are not true?
A. Segments are subdivisions in both the French and American systems.
B. Segments are determined primarily by the hepatic venous drainage.
C. The French anatomic system is more applicable than the American system to clinical hepatic resection.
D. Segments are important to the understanding of the topographic anatomy of the liver.
Answer: D


DISCUSSION: Segments are the major subdivision of the right and left lobes of the liver. In either the classic lobar (American) or the segmental (French) system, the most variable aspect is the biliary system. Therefore
the hepatic venous or portal system defines most segments. The French system depicts eight segments, with the caudate lobe as segment I and the other seven segments defined primarily by the hepatic venous system. Segments are not well-depicted by topography.




2. Which of the following anatomic features of the biliary system are important considerations in operative cholangiography?
A. The left hepatic duct comes off farther anterior than the right one.
B. At the confluence there may be more than just a right and a left hepatic duct.
C. Dissection of the triangle of Calot is more important than cholangiography in preventing bile duct injury.
D. Segments V, VII, or VIII sometimes join the biliary system below the confluence.
Answer: ABCD


DISCUSSION: All of these features are important. The angle of takeoff of the left hepatic duct may make operative visualization difficult with the patient in the supine position. Because there may be more than two major hepatic ducts, visualization of two large ducts does not ensure that the system is normal. Ducts from any of the right-side segments can join below the confluence. Dissecting one superior edge of the gallbladder before it joins the cystic duct is particularly important in preventing injury.




3. The hepatic artery:
A. Supplies the same amount of blood to the liver as the portal vein.
B. Provides more blood to the bile ducts than the portal vein.
C. Is autoregulated just as the portal vein is.
D. Supplies most of the blood to hepatic metastases.
Answer: BD


DISCUSSION: The portal vein provides two thirds to three quarters of the total hepatic blood flow. The portal vein is incapable of direct autoregulation. The hepatic artery after transplantation classically infarcts portions of the biliary system, whereas hepatic metastases often arrive there via the portal vein. Most of their blood supply comes from the hepatic artery.




4. Bile formation is:
A. An active secretory process.
B. Determined at two sites principally.
C. Regulated physiologically by hormones.
D. Largely determined by the intactness of the enterohepatic circulation (EHC).
Answer: ABCD


DISCUSSION: Bile formation is an active process at both the canalicular and ductular sites. The paracellular pathway probably plays a minor physiologic role. Secretin and glucagon are likely physiologic regulators of biliary secretion. Bile salts are extremely important and are probably the most important agent in the changes that occur when enterohepatic circulation is interrupted.




5. Generally, the two most important hepatic functions to consider after hepatic resection are:
A. Hepatic synthetic function.
B. Glucose metabolism.
C. The liver's role in lipid metabolism.
D. The liver's role in vitamin metabolism.
Answer: AB


DISCUSSION: While other functions undoubtedly may be important postoperatively, the most common abnormalities occurring after a major hepatic resection are related to loss of protein synthesis and consequences of glucose metabolism. Therefore, it is usually advisable to administer supplemental amounts of protein and sugar postoperatively.




6. Which of the following statements about pyogenic abscess of the liver are true?
A. The right lobe is more commonly involved than the left lobe.
B. Appendicitis with perforation and abscess is the most common underlying cause of hepatic abscess.
C. Mortality is largely determined by the underlying disease.
D. Mortality from hepatic abscess is currently greater than 40%.
Answer: AC


DISCUSSION: Involvement of the right lobe with abscess formation approximates 70% of pyogenic abscesses. This is thought to be due to the streaming effect of superior mesenteric venous inflow to the right lobe. In addition, the greater volume of the right lobe predisposes more tissue to seeding by bacterial organisms. While appendicitis comprised 25% to 40% of cases in early series, early recognition and operative therapy for appendicitis have reduced its importance significantly. In current series, malignant or benign biliary obstruction is the underlying cause of 35% to 50% of cases. Recent studies have shown that the underlying disease or an immunocompromised host is more important prognostically than solitary versus multiple abscesses.




7. Which of the following statements most accurately describes the current therapy for pyogenic hepatic abscess?
A. Antibiotics alone are adequate for the treatment of most cases.
B. All patients require open surgical drainage for optimal management.
C. Optimal treatment involves treatment of not only the abscess but the underlying source as well.
D. Percutaneous drainage is more successful for multiple lesions than for solitary ones.
Answer: C


DISCUSSION: The development of ultrasonography and computed tomography (CT) in the past two decades has enabled earlier diagnosis and advances in treatment of hepatic abscess. Formerly, open surgical drainage was considered necessary in essentially all cases of pyogenic abscess. Numerous recent series, however, have reported high success rates and low mortality from the percutaneous catheter drainage of abscesses under CT or ultrasonographic guidance. Optimal management of pyogenic abscess, however, involves not only treatment of the abscess, whether by percutaneous or surgical methods, but correction of the underlying source as well. All modes of therapy are more successful in treating solitary lesions than multiple ones.




8. Which of the following statements characterize amebic abscess?
A. Mortality is higher than that for similarly located pyogenic abscesses.
B. The diagnosis of amebic abscess may be based on serologic tests and resolution of symptoms.
C. In contrast to pyogenic abscess, the treatment of amebic abscess is primarily medical.
D. Patients with amebic abscess tend to be older than those with pyogenic abscess.
BC


DISCUSSION: Mortality for uncomplicated amebic abscess should be less than 5%, in contrast to the 15% to 20% rate for pyogenic abscess. After the demonstration by radiologic examination of an abscess, appropriate serologic tests and resolution of symptoms after a course of treatment with an antiamebic agent such as metronidazole constitute presumptive diagnosis of amebic abscess. Aspiration of abscess contents rarely yields amebic organisms. In contrast to pyogenic abscess, amebic abscess rarely requires surgical or percutaneous drainage, except in the case of an extremely large abscess or bacterial superinfection. Amebic abscess affects males in a 9:1 to 10:1 ratio and generally affects a younger population than pyogenic abscess. Additionally, in the United States the populations most affected are immigrants from endemic areas such as Mexico or Latin America and American tourists to those regions.




9. Which of the following statement(s) is/are true about benign lesions of the liver?
A. Adenomas are true neoplasms with a predisposition for complications and should usually be resected.
B. Focal nodular hyperplasia (FNH) is a neoplasm related to birth control pills (BCPs) and usually requires resection.
C. Hemangiomas are the most common benign lesions of the liver that come to the surgeon's attention.
D. Nodular regenerative hyperplasia does not usually accompany cirrhosis.
Answer: A


DISCUSSION: Adenomas typically enlarge and cause symptoms, may rupture, and have a definite malignant potential. Therefore they should generally be resected when found. FNH is not a true neoplasm and generally has an uneventful course. Both are related to BCPs, although the relationship of adenoma is more firmly established. While small bile duct hamartomas are much more common, hemangiomas are the most common lesion to come to the attention of surgeons. They should not generally be biopsied because of possible hemorrhage. By definition, nodular regenerative hyperplasia occurs in the absence of cirrhosis.




10. Which of the following statement(s) about malignant neoplasms of the liver is/are true?
A. Hepatocellular carcinoma is probably the number 1 cause of death from cancers worldwide.
B. The most common resectable hepatic malignant neoplasm in the United States is colorectal metastasis.
C. Hepatoma has at least one variant that has a much more benign course than hepatomas in general.
D. Hepatomas are generally slower growing than was formerly believed.
Answer: ABCD


DISCUSSION: Although exact comparisons are impossible, hepatoma seems to be the most common cause of cancer death worldwide, despite its relative infrequency in the United States. Colorectal metastasis is a more common indication for surgical treatment in the United States. The fibrolamellar variant and possibly the very well-differentiated tumor probably have a better prognosis than hepatomas in general. Previous studies from Africa in which there was a high incidence of rupture account for the poor prognosis that was generally attributed to hepatoma. Recent studies from Europe and the United States have shown that survival after presentation is usually measured in years.




11. Which of the following statement(s) is/are true about bile duct cancers?
A. If resected, proximal lesions are usually curable.
B. The more proximal the lesion, the more likely is resection to be curative.
C. Radiation clearly prolongs survival.
D. Transplantation is usually successful if the lesion seems confined to the liver.
E. None of the above is true.
Answer: E


DISCUSSION: Most bile duct cancers are discovered after they are incurable, and only a tiny subset of resected proximal lesions are cured. The more distal the lesion, the more likely is resection to achieve cure (e.g., approximately 30% 5-year survival for periampullary lesions as compared with 0% to 10% for hilar lesions). The use of adjuvant or primary radiation remains controversial because of the heterogeneity of the patient populations on which this modality has been used. Because of the localized nature of this disease it would seem that transplantation would produce favorable results; however, this has not been the case.




12. Echinococcosis liver disease caused by Echinococcus granulosus:
A. Is not a neoplasm.
B. Is endemic to parts of Europe, but not the United States.
C. Is usually curable by resection.
D. Is more deadly than in its Echinococcus multilocularis form.
Answer: ABC


DISCUSSION: The parasitic infection is fairly common in certain parts of Europe but very rare in the United States. Resection without peritoneal soilage is the treatment of choice. The E. multilocularis form, which is endemic to parts of the United States, is more likely to be fatal because it is rarely resectable. This form is more likely to resemble a malignancy than E. granulosus, although the natural course of the disease usually spans many years.




13. Which of the following statements about hemobilia are true?
A. Tumors are the most common cause.
B. The primary treatment of severe hemobilia is an operation.
C. Percutaneous cholangiographic hemobilia is usually minor.
D. Ultrasonography usually reveals a specific diagnosis.
Answer: C


DISCUSSION: By far the most common cause of hemobilia is trauma. Tumors also may cause the syndrome but are relatively uncommon causes. For severe hemobilia the best therapy is arteriographic embolization. Usually the site of bleeding or a false aneurysm can be identified. Operation should be reserved as a last resort or when the condition is recognized intraoperatively. Percutaneous cholangiography–associated intrabiliary hemorrhage is usually, but not always, minor and self-limiting. Ultrasonography is a very nonspecific diagnostic technique for hemobilia. Arteriography remains the best diagnostic method.




14. Ligation of all of the following arteries usually causes significant hepatic enzyme abnormalities except:
A. Ligation of the right hepatic artery.
B. Ligation of the left hepatic artery.
C. Ligation of the hepatic artery distal to the gastroduodenal branch.
D. Ligation of the hepatic artery proximal to the gastroduodenal artery.
Answer: D


DISCUSSION: Ligation of the right or left hepatic artery frequently causes enzyme elevation but is usually tolerated by the patient, particularly when this is a life-saving maneuver. Ligation of the hepatic artery distal to the gastroduodenal branch is more risky but is also usually tolerated. Ligation of the hepatic artery proximal to the gastroduodenal one does not normally cause enzyme abnormalities because of abundant collateral flow through that branch.




15. Which of the following is the most common acid-base disturbance in patients with cirrhosis and portal hypertension?
A. Metabolic acidosis.
B. Respiratory alkalosis.
C. Metabolic alkalosis.
D. Respiratory acidosis.
Answer: C


DISCUSSION: Metabolic alkalosis and hypokalemia are common in patients with cirrhosis because they often have associated secondary hyperaldosteronism (especially those with ascites), diarrhea, and frequent emesis. Hyperaldosteronism enhances H+ and K+ exchange for Na+ in the distal tubule of the kidney. The cause of diarrhea in patients with cirrhosis is unknown, but malabsorption secondary to splanchnic venous hypertension may be a contributing factor. Emesis is common in alcoholic cirrhotics and patients with tense ascites. Deleterious effects of metabolic alkalosis include impaired tissue oxygen delivery secondary to shift of the oxyhemoglobin dissociation curve to the left and conversion of ammonium chloride to ammonia, which may contribute to encephalopathy.




16. A portal venous pressure of 30 mm. Hg (elevated) and a hepatic venous wedge pressure of 5 mm. Hg (normal) may be associated with which of the following causes of portal hypertension?
A. Portal vein thrombosis.
B. Alcoholic cirrhosis.
C. Schistosomiasis.
D. Alcoholic hepatitis.
Answer: AC


DISCUSSION: Pressure measured by wedging a catheter into a hepatic vein (hepatic venous wedge pressure) closely correlates with directly measured portal venous pressure in patients with portal hypertension when the site of elevated resistance is at the sinusoidal or postsinusoidal level. Such is the case in alcoholic cirrhosis and alcoholic hepatitis. When the site of increased resistance is at the presinusoidal level, either within (schistosomiasis) or outside (portal vein thrombosis) the liver, the hepatic venous wedge pressure is normal despite markedly elevated portal vein pressure. Although schistosomiasis is one of the more frequent causes of portal hypertension worldwide, in North America presinusoidal portal hypertension is considerably less common than alcoholic liver disease. A normal hepatic venous wedge pressure in a patient who has bled from varices should lead one to suspect a presinusoidal cause. A specific diagnosis can often be made by visceral angiography or liver biopsy.




17. Which of the following is the most effective definitive therapy for both prevention of recurrent variceal hemorrhage and control of ascites?
A. Endoscopic sclerotherapy.
B. Distal splenorenal shunt.
C. Esophagogastric devascularization (Sugiura procedure).
D. Side-to-side portacaval shunt.
E. End-to-side portacaval shunt.
Answer: D


DISCUSSION: Shunt operations are the most effective means of preventing recurrent variceal hemorrhage. Rebleeding rates after endoscopic sclerotherapy range from 40% to 60%. Although extensive esophagogastric devascularization has effectively prevented recurrent bleeding in Japanese series, these operations have been followed by rebleeding rates in excess of 25% in most Western series. Although one controlled trial has shown more frequent recurrent hemorrhage following the distal splenorenal shunt than after the portacaval shunt, most series have reported rebleeding rates of less than 10% for both of these operations. Both the liver and the splanchnic viscera are important sites of ascites formation. Since the distal splenorenal shunt maintains sinusoidal and mesenteric venous hypertension and requires interruption of important retroperitoneal lymphatics, it tends to aggravate rather than relieve ascites. Hepatic sinusoidal pressure may be unchanged or even increased after an end-to-side portacaval shunt. Only side-to-side portal-systemic shunts, such as the side-to-side portacaval shunt, reliably decompress both the liver and splanchnic viscera, thus preventing ascites formation.




18. Which of the following treatments most effectively preserves hepatic portal perfusion?
A. Distal splenorenal shunt.
B. Conventional splenorenal shunt.
C. Endoscopic sclerotherapy.
D. Side-to-side portacaval shunt.
Answer: C


DISCUSSION: The conventional splenorenal shunt and side-to-side portacaval shunts completely divert portal flow away from the liver (nonselective shunts). The distal splenorenal shunt is a selective shunt that preserves hepatic portal perfusion in the majority of patients; however, the magnitude of portal flow is decreased because the gastrosplenic component is diverted into the renal vein. Additionally, many patients (especially alcoholic cirrhotics) develop collaterals between the mesenteric venous circulation and the shunt, resulting in gradual attrition of the remaining portal flow. Although there have been anecdotal reports of portal vein thrombosis after endoscopic sclerotherapy, two controlled trials have demonstrated better preservation of hepatic portal perfusion in sclerotherapy patients than in persons who receive the distal splenorenal shunt.




19. Which of the following veins is preserved in performing the extensive esophagogastric devascularization procedure described by Sugiura?
A. Left gastric (coronary) vein.
B. Short gastric vein.
C. Splenic vein.
D. Left gastroepiploic vein.
Answer: A


DISCUSSION: The Sugiura procedure consists of devascularization of the esophagus to the inferior pulmonary vein and the proximal two thirds of the stomach, splenectomy, and distal esophageal transection. The devascularization component should be done as close to the esophagus and stomach as possible. The coronary vein and paraesophageal collaterals are preserved to maintain an effective portal-systemic collateral pathway and thereby discourage reformation of varices.




20. Which of the following complications of portal hypertension often require surgical intervention (for more than 25% of patients)?
A. Hypersplenism.
B. Variceal hemorrhage.
C. Ascites.
D. Encephalopathy.
Answer: B


DISCUSSION: While many patients with portal hypertension develop hypersplenism, it is rarely clinically significant. A splenectomy should not be performed unless platelet counts are persistently less than 20,000 per cu. mm. or white blood cell counts are less than 1200 per cu. mm. Unfortunately, splenectomy is sometimes done for clinically insignificant hypersplenism, thus obviating a distal splenorenal shunt if the patient should subsequently bleed from varices. The initial treatment for most patients with bleeding esophageal varices should be endoscopic sclerotherapy; however, operation is required for the approximately one third of patients who fail sclerotherapy and for noncompliant persons, those living in remote geographic locations, and patients bleeding from gastric varices. Ascites can be controlled by a medical regimen of dietary salt restriction and diuretic therapy in more than 95% of patients. When ascites is intractable to medical management, either intermittent large-volume paracenteses or a surgical peritoneovenous shunt should be done. With rare exceptions, encephalopathy should be treated medically. Most important is elimination of any precipitating factors that led to the neuropsychological disturbance. Lactulose, neomycin, and dietary protein restriction may also be components of the medical treatment regimen.




21. Which of the following effects are advantages of combined vasopressin and nitroglycerin intravenous infusion, as compared with vasopressin infusion alone, in controlling acute variceal bleeding?
A. Lower frequency of encephalopathy.
B. Lower incidence of vasopressin side effects.
C. More effective control of bleeding.
D. Less “rebound effect” when discontinuing the infusion.
Answer: BC


DISCUSSION: Vasopressin acts through vasoconstriction of splanchnic arterioles. Both portal venous inflow and portal venous pressure are reduced, resulting in control of acute variceal bleeding in approximately 50% of patients. However, the adverse side effects of systemic hypertension, bradycardia, decreased cardiac output, and coronary vasoconstriction are quite common during vasopressin infusion. Simultaneous administration of nitroglycerin or nitroprusside eliminates these side effects—and in one controlled trial enhanced therapeutic effectiveness. Although the mechanism of action of this combined infusion is not clear, vasodilation of portal-systemic collaterals, resulting in a further reduction in portal pressure, may be responsible.




22. Which of the following statements about the peritoneovenous shunt (PVS) is/are correct?
A. For cirrhotic patients with intractable ascites, the LeVeen shunt is an effective “bridge” to liver transplantation.
B. Replacement of ascites with saline or lactated Ringer's solution reduces the coagulopathy following PVS.
C. For patients with cirrhotic ascites, the survival using repeated paracentesis with 5% albumin infusion is equivalent to that with the PVS.
D. Oliguria (less than 25 ml. per hour) in the immediate postoperative period following PVS should be treated with a 5% albumin infusion.
E. The transjugular intrahepatic portacaval shunt with stent (TIPSS) works on the same principle as the PVS.
Answer: BC


DISCUSSION: The PVS is a palliative procedure that does not prolong life. In comparing the early risks of the procedure with those of repeated paracentesis, the shunt cannot be justified as a temporizing procedure to facilitate ascites control in the patient awaiting liver transplantation. Oliguria is common in the first 24 hours after shunt insertion. A correctly placed PVS (patency confirmed using an intraoperative “shuntogram”) expands the intravascular volume with a continuous reinfusion of ascites. Inspection should identify elevation of the jugular venous pressure, and a diuretic (usually furosemide) is needed. The mechanisms of action of the two shunts are very different. TIPSS reduces portal pressure and controls ascites by reducing the rate of ascites formation. PVS reinfuses the ascites fluid, thereby reducing the prerenal stimulus to sodium retention and making the patient more responsive to diuretic therapy.




23. Which of the following clinical situations are considered good indications for PVS?
A. A 50-year-old cirrhotic man had an emergency portacaval shunt for bleeding varices and postoperatively had an ascites leak and mild superficial wound infection.
B. A 57-year-old woman with primary biliary cirrhosis (PBC) has difficult to control ascites and diuretic-induced encephalopathy.
C. A 46-year-old resistant alcoholic has chronic ascites uncontrolled by diuretics combined with repeat paracentesis.
D. A 34-year-old woman taking BCPs had rapid onset of ascites and is found to have hepatic vein thrombosis causing the Budd-Chiari syndrome.
Answer: C


DISCUSSION: Because of the high complication rate and the long-term failure rate, the PVS is used only when other, more lasting options for therapy either are not available or are contraindicated. The chronic alcoholic patient may benefit from a peritoneovenous shunt because his ascites is the dominant problem related to his chronic liver disease, and persistent alcoholism is a contraindication to liver replacement in most centers. PVS may be quite effective for the temporary management of acute intractable postoperative ascites, such as in patient A; however, it is absolutely contraindicated in the presence of infection. Patient B has ascites as her dominant problem as well; however, with PBC as the underlying liver disease, she is an excellent candidate for transplantation. Patient D also has ascites as the major problem; however, the side-to-side portosystemic shunt is a far better long-term treatment option than PVS.




24. Which of the following explanations account(s) for the fact that hepatitis C is the most common cause of posttransfusion hepatitis?
A. There are more carriers of hepatitis C virus (HCV) in the normal population who serve as blood donors.
B. Blood infected with hepatitis B virus (HBV) is eliminated through routine testing, leaving only HCV as the other blood-borne pathogen.
C. Current serologic tests for HCV antigen do not exclude carriers.
D. Questions designed to eliminate risk groups for HCV from the normal donor population may not be as specific as would be desirable.
E. Hepatitis C is a more virulent form of viral hepatitis, so it is expected that more cases of posttransfusion hepatitis would occur.
Answer: BD


DISCUSSION: The ability to specifically identify persons infected with HCV has only recently become available. Therefore, data about epidemiology are less than complete. It is very likely not true that more blood donors carry HCV because of the large preponderance of HBV in the United States. It is true, however, that successful elimination of most of the HBV carriers occurs through routine testing. Although serologic tests are available for HCV, they are tests, not of antigen, but of antibody. Therefore, this test alone may not screen out persons who are infected but have not yet developed or may never develop antibody. Risk groups for the relatively newly defined HCV may well not be comprehensively established, and therefore this explanation may be a contributor. There are no differences in virulence between these classes of hepatitis virus.




25. True or false: HBV infections:
A. Are usually asymptomatic.
B. May not be clinically recognized but may lead to chronic hepatitis.
C. Reliably protect against subsequent HBV infection regardless of the measured antibody titer to hepatitis B surface antigen (HBsAg).
D. Are completely prevented by postexposure administration of HBIg hepatitis B immunoglobulin (HBIg).
E. Preclude subsequent infection with HDV.
Answer: TRUE: BC, FALSE ADE


DISCUSSION: Although some types of hepatitis are more often asymptomatic than symptomatic, that is not the case for hepatitis B. Further, even if the HBV infection is asymptomatic, serious long-term side effects may occur. A prior infection with hepatitis B confers lifelong immunity even if the antibody titer wanes below the protective level of 10 mIU. HBIg is useful in reducing the incidence of postexposure HBV infection from around 30% with no intervention, to 15% with standard immune globulin, to about 5% to 7% with HBIg. HBV infection is required for infection with HDV and is therefore an essential step toward, rather than preventive of, HBV infection.




26. Which of the following statements about choledocholithiasis are correct?
A. Common duct stones can originate in the gallbladder and migrate to the common duct, and stones can form de novo in the duct system.
B. Calcium bilirubinate stones are associated with the presence of bacteria in the duct system.
C. Common duct stones discovered at laparoscopic cholecystectomy should be treated by postoperative endoscopic extraction.
D. The serum bilirubin value is usually greater than 15 mg. per dl. in the patient with a symptomatic common duct stone.
Answer: ABC


DISCUSSION: Most common duct stones originate in the gallbladder and migrate to the common duct, where they may become larger. These stones tend to consist predominantly of cholesterol (about 80% of gallbladder stones are predominantly cholesterol). Stones found in the bile ducts after cholecystectomy may have been overlooked, but de novo stone formation does occur. Arbitrarily, stones found 2 years after cholecystectomy are assumed to have formed within the duct system. Calcium bilirubinate stones are thought to result from precipitation of insoluble bilirubin monoglucuronide formed by deconjugation of bilirubin diglucuronide, a reaction promoted by the enzyme beta-glucuronidase, which is produced by bacteria in the biliary tract. Calcium bilirubinate stones are found almost exclusively in patients who have some form of biliary tract lesion that causes partial obstruction, and these patients tend to have bactibilia. Stones smaller than approximately 5 mm. often can be extracted through a dilated cystic duct or pushed into the duodenum. Larger stones are best left for postoperative endoscopic sphincterotomy and extraction. Patients with more than five stones or stones larger than 1.5 cm. should be treated by open choledocholithotomy or, when indicated, a biliary-enteric anastomosis. Not all patients with symptomatic common duct stones have elevated serum bilirubin, but when jaundice is present the bilirubin is only rarely greater than 15 mg. per dl.




27. A benign biliary duct stricture:
A. Need not be treated unless it causes clinical jaundice.
B. Should always be treated by percutaneous balloon drainage.
C. Is prone to recur after treatment with biliary-enteric anastomosis.
D. When due to chronic pancreatitis should be treated by side-to-side choledochoduodenostomy.
Answer: CD


DISCUSSION: Even a minor obstructing lesion in the extrahepatic duct system can produce cirrhosis over time, and the development of portal hypertension, ascites, and esophageal varices. Therefore, all biliary strictures should be treated unless this is not possible or there is no chance for success. The presence or absence of jaundice is of no significance. Often, the only biochemical abnormality is mild elevation of alkaline phosphatase. The long-term results of percutaneous balloon dilatation are not yet known, but short-term results are good. Although some argue that balloon dilatation should be the initial treatment, its role is ill-defined, and it should not be viewed as standard therapy at this time. Biliary-enteric anastomoses are predisposed to stricture, for reasons that are ill-understood. A mucosa-to-mucosa anastomosis, large size of the anastomosis, a normal duct at the point of anastomosis, and stenting appear to be elements that work against stricture. About 70% of anastomoses are not complicated by strictures. Common duct strictures caused by chronic pancreatitis are located in the distal portion of the duct and are easily treated by side-to-side choledochoduodenostomy. A wide anastomosis is usually possible, and because of this stenting often is not necessary. Although a Roux-en-Y biliary-enteric reconstruction is acceptable treatment, no advantage over choledochoduodenostomy has been demonstrated.




28. Which statements about extrahepatic bile duct cancer are correct?
A. Cholangiography is essential in evaluating patients for resectability.
B. The prognosis is excellent when appropriate surgical and adjuvant therapy are given.
C. The location of the tumor determines the type of surgical procedure.
D. The disease usually becomes manifest by moderate to severe right-side upper quadrant pain.
Answer: AC


DISCUSSION: Cholangiography is essential for both diagnosis and evaluation of resectability. Brushings of the lesion for diagnosis and temporary stenting, done percutaneously or endoscopically, are often done at the time of cholangiography. Angiography and CT are helpful, but in the absence of hepatic artery or portal vein occlusion these tests are not accurate predictors of resectability. The primary obstacles to complete resection are invasion of the portal vein or the hepatic artery and proximal extension of the tumor into the liver. The long-range prognosis for patients who undergo treatment for extrahepatic bile duct cancer is poor, even when the lesion is surgically resectable and adjuvant therapy is given. Only about 10% of patients are alive without disease at 10 years. Nevertheless, bile duct cancer tends not to metastasize to distant sites, so resection and radiation therapy are useful in prolonging symptom-free life. Tumors in the proximal third of the extrahepatic bile duct system are treated by a Roux-en-Y biliary-enteric anastomosis. To ensure excision of the entire tumor this anastomosis usually must be made to the individual hepatic ducts, which must be stented individually. Tumors of the middle third usually require anastomosis to the proximal hepatic duct. In contrast, lesions of the distal third require Whipple's procedure with appropriate reconstruction. Thus, the treatment of extrahepatic bile duct cancer depends on the location of the tumor. Pain is not a prominent feature of bile duct cancer. Most cases become manifest by the insidious development of jaundice.




29. Which of the following statements about biliary tract problems are correct?
A. Choledochal cyst should be treated by Roux-en-Y cystojejunostomy.
B. Sclerosing cholangitis is characterized by long, narrow strictures in the extrahepatic biliary duct system.
C. Operative (needle) cholangiography is indicated in patients who at operation appear to have no gallbladder.
D. The long cystic duct, which appears to be fused with the common duct and enters it distally, should be dissected free and ligated at its entrance into the common duct.
Answer: C


DISCUSSION: In the past, choledochal cyst was treated by Roux-en-Y cystojejunostomy, but long-term results were poor. Excision of the cyst is essential to prevent recurrent pancreatitis. In addition, the development of carcinoma in about 25% of patients mandates cyst excision. Accordingly, excision of the cyst with biliary reconstruction by Roux-en-Y hepaticojejunostomy and diversion of the flow of pancreatic juice through the ampulla of Vater is currently the standard treatment. Sclerosing cholangitis causes fibrosis of bile ducts both within and outside the liver. This process, which is poorly understood, causes strictures in the duct system, characteristically with normal or dilated segments between strictures. Unfortunately, this anatomic arrangement does not lend itself to biliary reconstructive procedures. Each case must be analyzed, however, because in some patients the anatomic situation may lend itself to balloon dilatation or reconstruction. When the gallbladder appears to be absent, a search should be made for an ectopically located organ in the retroduodenal area, within the falciform ligament, and within the substance of the right lobe of the liver. With true gallbladder agenesis the common duct may be dilated, and choledocholithiasis is present in about one fourth of those who undergo operation. Therefore, operative needle cholangiography should always be done. Dissection of a long, fused cystic duct is fraught with hazard because the cystic and common ducts may share a common wall and serious duct damage may occur. The cystic duct should be ligated and divided immediately proximal to the area of fusion.




30. Which of the following statements about the diagnosis of acute calculous cholecystitis are true?
A. Pain is so frequent that its absence almost precludes the diagnosis.
B. Jaundice is present in a majority of patients.
C. Ultrasonography is the definitive diagnostic test.
D. Cholescintigraphy is the definitive diagnostic test.
Answer: AD


DISCUSSION: The presence of pain is the sine qua non of acute calculous cholecystitis. Chronic cholecystitis associated with cholelithiasis may develop in the absence of pain, and in critically ill patients pain may not be a prominent feature of acute acalculous cholecystitis. Only about 10% of patients with acute cholecystitis are jaundiced. Although an occasional patient may have concomitant bile duct obstruction, the jaundice associated with acute cholecystitis is probably due to absorption of bile pigments from the diseased gallbladder. The presence of jaundice in a patient with right-side upper quadrant pain should also suggest the possibility of acute cholangitis secondary to bile duct obstruction. Ultrasonography is very accurate in the detection of gallstones, but stones may be present in the absence of acute cholecystitis. Thickening of the gallbladder wall and a collection of fluid around the gallbladder are ultrasonographic findings in some patients with acute cholecystitis, but they are not always present and are not specific. Ultrasonography may be useful when the diagnosis is obscure because other conditions in the liver, pancreas, and kidney can be detected; however, it is not the definitive test for acute cholecystitis. Cholescintigraphy is specific for the diagnosis of acute calculous cholecystitis (accuracy over 95% in experienced hands). The rapidity, simplicity, and accuracy make cholescintigraphy the definitive diagnostic test in acute calculous cholecystitis; however, it must be interpreted cautiously in the context of another critical illness or recent surgery or trauma, because false-positives are not unusual in these situations.




31. Which statements about acute acalculous cholecystitis are correct?
A. The disease is often accompanied by or associated with other conditions.
B. The diagnosis is often difficult.
C. The mortality rate is higher than that for acute calculous cholecystitis.
D. The disease has been treated successfully by percutaneous cholecystostomy.
Answer: ABCD


DISCUSSION: About half of the cases of acute acalculous cholecystitis are associated with other conditions, including sepsis, sarcoidosis, polyarteritis nodosa, and systemic lupus erythematosus. A majority of cases occur after trauma, burns, or major surgical procedures performed for other conditions. The precise pathogenesis has not been determined. The diagnosis of acute acalculous cholecystitis is often difficult because symptoms may be masked by another illness, injury, or the postoperative state. Unlike acute calculous cholecystitis, in which pain is always present, pain occurs in only about 70% of cases. In addition, cholescintigraphy is sometimes inaccurate. These factors make the diagnosis difficult, and a high index of suspicion is necessary, especially in patients who have had operations or trauma. Unexplained abdominal pain, sepsis, and ileus should prompt a thorough investigation. The mortality rate for acute acalculous cholecystitis is higher than that of the calculous type. The incidence of gangrene and perforation of the gallbladder is higher. The accompanying illnesses and conditions and the frequent delays in diagnosis undoubtedly contribute to the higher death rate. Percutaneous cholecystostomy has been used as a diagnostic and therapeutic maneuver in patients who are thought to have acute acalculous cholecystitis. Aspiration and culture of bile assist in confirming the diagnosis, and continuous drainage successfully treats the acute condition. Surprisingly, persistent gangrene and subsequent complications have been infrequent. Immediate cholecystectomy should be done if significant improvement does not take place within 12 hours of percutaneous cholecystostomy. Long-term management of the tube and the need for elective cholecystectomy must be individualized. The experience with percutaneous cholecystostomy is too small to determine whether this technique reduces the mortality rate.




32. True statements about the surgical management of patients with acute calculous cholecystitis include:
A. Operation should be performed in all patients as soon as the diagnosis is made.
B. Antibiotic therapy should be initiated as soon as the diagnosis is made.
C. Dissection of the gallbladder is facilitated by decompression of the organ with the use of a trocar.
D. An operative cholangiogram should be done in every patient.
Answer: BC


DISCUSSION: Cholecystectomy should be done in an otherwise healthy patient as soon as the diagnosis is made and the patient is properly prepared for surgery. However, patients who have one or more significant risk factors such as a recent myocardial infarction, unstable angina, clinically significant coronary artery disease, or cirrhosis should not have immediate cholecystectomy unless they do not improve within 24 to 36 hours in response to antibiotic administration and supportive care. Antibiotic administration should commence as soon as the diagnosis is made and should be continued for 24 hours postoperatively—or for 7 days if significant peritonitis is present. This use of antibiotics has significantly reduced septic complications after cholecystectomy for acute cholecystitis. In most cases the gallbladder is tensely distended, making visualization and dissection of the cystic duct area difficult and perhaps dangerous. Decompression of the gallbladder by insertion of a needle facilitates retraction and dissection of the gallbladder. Although some advise that operative cholangiography be done only on a selective basis, its routine use helps to delineate anatomy and facilitates detection of an occasionally unsuspected bile duct stone. Accordingly, it is used routinely in elective cases. In acute cholecystitis, however, the biliary duct system may be very friable, and operative cholangiography should be done only when it is safe to do so.




33. Which of the following are indications for cholecystectomy?
A. The presence of gallstones in a patient with intermittent episodes of right-side upper quadrant pain.
B. The presence of gallstones in an asymptomatic patient.
C. The presence of symptomatic gallstones in a patient with angina pectoris.
D. The presence of asymptomatic gallstones in a patient who has insulin-dependent diabetes.
Answer: A


DISCUSSION: Cholecystectomy (and concomitant operative cholangiography) are indicated for symptomatic patients to relieve pain and to prevent the development of acute cholecystitis and its complications. Morbidity and expense are not as great for elective cholecystectomy as they are for cholecystectomy for acute cholelithiasis. The risk of the development of symptoms in patients who have asymptomatic stones is approximately 2% per year, a rate associated with mortality and morbidity that do not exceed those of elective cholecystectomy. Therefore, cholecystectomy is not indicated for asymptomatic patients. Patients who have angina pectoris should not have cholecystectomy until their coronary artery disease has been treated adequately, even if this requires a coronary artery bypass procedure. Heart disease is the most frequent cause of death after cholecystectomy. Prophylactic cholecystectomy, formerly recommended for insulin-dependent diabetics, is not indicated because several studies have shown that the mortality rate from acute cholecystitis is no higher for diabetics than for nondiabetics.




34. Which of the following statements about laparoscopic cholecystectomy are correct?
A. The procedure is associated with less postoperative pain and earlier return to normal activity.
B. The incidence of bile duct injury is higher than for open cholecystectomy.
C. Laparoscopic cholecystectomy should be used in asymptomatic patients because it is safer than open cholecystectomy.
D. Pregnancy is a contraindication.
Answer: AB


DISCUSSION: Studies have clearly documented that postoperative pain following laparoscopic cholecystectomy is less than that experienced after open cholecystectomy and that patients can resume normal activity sooner. This appears to be related to the reduced trauma to the abdominal wall by virtue of the very small incisions used in laparoscopic procedures. The best evidence is that the bile duct injury rate (0.4%) is approximately double that for open cholecystectomy. The incidence of this serious complication will probably decrease with improved techniques, better training, and more advanced instrumentation. Only symptomatic patients should have cholecystectomy. Prophylactic removal of the gallbladder is not cost effective. All elective operations are contraindicated in the first trimester, so as to prevent fetal anomalies and spontaneous abortion. The laparoscopic technique is not contraindicated thereafter except in patients in whom peritoneal access cannot safely be established. This is rarely a problem. Premature labor is a risk in the third trimester. Thus, unless cholecystectomy can be avoided altogether during pregnancy, the second trimester is the most propitious time.




35. Which of the following statements about cholangitis are correct?
A. Charcot's triad is always present.
B. Associated biliary tract disease is always present.
C. Chills and fever are due to the presence of bacteria in the bile duct system.
D. The most common cause of cholangitis is choledocholithiasis.
Answer: BCD


DISCUSSION: Although Charcot's triad (pain, chills and fever, jaundice) is diagnostic of cholangitis, the complete triad occurs only in 50% to 70% of patients. Fever is the most common symptom; therefore, cholangitis should be considered in all patients who have unexplained fever. Episodes of pain, chills, and fever are often so brief as not to concern the patient. Cholangitis does not occur in the absence of partial or complete bile duct obstruction. All patients diagnosed as having cholangitis should have appropriate diagnostic studies to determine the cause. This usually involves cholangiography. The presence of bacteria in bile does not produce symptoms in the absence of partial or complete obstruction of the bile duct system. When obstruction is present, pressure within the system increases, giving rise to reflux of bacteria or their toxic products into the hepatic venous circulation. This cholangiovenous reflux produces chills, fever, and the hemodynamic changes of sepsis. Death may ensue if treatment is not instituted promptly. Choledocholithiasis, the most commonly associated problem, may produce partial or complete obstruction. When bacteria are not present in the bile duct system, choledocholithiasis may go undetected unless the degree of obstruction is sufficient to cause jaundice. Other causes of cholangitis are benign and malignant strictures, biliary-enteric anastomoses, invasive procedures, foreign bodies, and parasitic infestation of the bile ducts.




36. Recurrent episodes of cholangitis:
A. Suggest the presence of undetected or overlooked bile duct pathology.
B. Occur frequently in patients who have indwelling biliary tubes or stents.
C. May be ameliorated by long-term administration of antibiotics.
D. May be associated with the development of secondary biliary cirrhosis.
Answer: ABCD


DISCUSSION: Cholangitis does not occur in the presence of a normal bile duct system, and all patients with cholangitis have an abnormality. Thus, recurrent episodes of cholangitis signal the need for diagnostic studies. Cholangiography usually will be necessary. The presence of any foreign body in the biliary tract is frequently associated with bactibilia and recurrent episodes of cholangitis. Even a silk suture exposed to the lumen of a bile duct has been known to cause cholangitis. Pigment stone and sludge formation may result from the bacterial deconjugation of bilirubin diglucuronide to bilirubin monoglucuronide, which precipitates as calcium bilirubinate. This material can occlude indwelling tubes and predispose to more frequent episodes of cholangitis. Long-term administration of an oral antibiotic may reduce the frequency and severity of attacks of cholangitis; however, this method of management should not be routine. Correction of the underlying problem is essential. Chronic obstruction and recurrent infection eventually lead to secondary biliary cirrhosis and its complications of portal hypertension, ascites, and bleeding esophageal varices. Once this stage of the disease is reached, correction of the underlying biliary tract problem does not reverse the changes in the liver. Once again, every effort should be made to eliminate the cause of the cholangitis early in the course of disease. The only effective treatment for end-stage liver disease is hepatic transplantation.




37. The initial goal of therapy for acute toxic cholangitis is to:
A. Prevent cholangiovenous reflux by decompressing the duct system.
B. Remove the obstructing stone, if one is present.
C. Alleviate jaundice and prevent permanent liver damage.
D. Prevent the development of gallstone pancreatitis.
Answer: A


DISCUSSION: Uncontrolled sepsis and the consequent multisystem organ failure are the life-threatening sequelae of acute toxic cholangitis. Thus, the initial goal of treatment is to decompress the biliary duct system to prevent reflux of bacteria and their toxic products into the circulation. This can be done by intubating the duct system through the percutaneous, transhepatic, or the endoscopic route or by insertion of a T tube in the common duct at operation. Removal of the stone causing the obstruction is not necessary to stabilize the patient. Only after the duct is decompressed should the cause of the obstruction be addressed. When transhepatic biliary drainage has been used, endoscopic or surgical removal of the stone can be carried out after the patient has recovered completely. When initial therapy is sphincterotomy, the stone should be removed as part of the procedure. Often the stone falls out without manipulation. If surgical placement of a T tube is the initial treatment, the stone should be removed only if it is convenient to do so. The long-range goal of treatment of patients with bile duct obstruction is to prevent cirrhosis, ascites, portal hypertension, and hemorrhage from esophageal varices; however, death from sepsis is the immediate threat in acute toxic cholangitis. Gallstone pancreatitis may occur in patients who have an impacted stone in the distal duct, independent of the presence or absence of acute toxic cholangitis; however, gallstone pancreatitis is more often associated with the passage of a stone into the duodenum.




38. The clinical picture of gallstone ileus includes which of the following?
A. Air in the biliary tree.
B. Small bowel obstruction.
C. A stone at the site of obstruction.
D. Acholic stools.
E. Associated bouts of cholangitis.
Answer: ABCE


DISCUSSION: An antecedent biliary-enteric fistula is necessary to allow stone migration into the intestinal tract, and this results in air entering the biliary tree (pneumobilia). It also allows contamination of the bile ducts with intestinal bacteria, which in fact occurs in only a minority of such cases. The stone obstructs the narrower distal bowel, producing small bowel obstruction. Such a stone, if opaque, can be seen on plain radiography and, if not, can be appreciated by sonography. Stools are not acholic, since the cholecystoenteric fistula allows bile access to the intestinal lumen.




39. Which of the following statement(s) about gallstone ileus is/are not true?
A. The condition is seen most frequently in women older than 70.
B. Concomitant with the bowel obstruction, air is seen in the biliary tree.
C. The usual fistula underlying the problem is between the gallbladder and the ileum.
D. When possible, relief of small bowel obstruction should be accompanied by definitive repair of the fistula since there is a significant incidence of recurrence if the fistula is left in place.
E. Ultrasound studies may be of help in identifying a gallstone as the obstructing agent.
Answer: C


DISCUSSION: It is true that gallstone ileus occurs mostly in elderly women and should always be suspect when small bowel obstruction presents in this age group. The great majority of cases of gallstone ileus are preceded by a spontaneous fistula occurring between the gallbladder and duodenum, allowing gallstones to enter the intestinal tract, which can potentially block the terminal ileum. Finding air within the biliary tree should always arouse suspicion of the possibility of this diagnosis when it is associated with a radiographic pattern of small bowel obstruction. The initial part of the operative approach to this disease is to relieve the bowel obstruction by performing an enterotomy just proximal to the point of obstruction to remove the stone. Where possible, definitive repair of the fistula should be undertaken to avoid recurrent obstruction and to obviate the possible recurring complications of cholangitis. Percutaneous drainage of bile collections combined with endoscopic papillotomy may be sufficient treatment for external and internal biliary fistulas but is never an allowable approach in the presence of gallstone ileus with small bowel obstruction. Relief of the obstruction is mandated in this setting.




40. Which of the following lesions are believed to be associated with the development of carcinoma of the gallbladder?
A. Cholecystoenteric fistula.
B. A calcified gallbladder.
C. Adenoma of the gallbladder.
D. Xanthogranulomatous cholecystitis.
E. All of the above.
Answer: E


DISCUSSION: The prevalence of carcinoma of the gallbladder in patients who have or have had a cholecystoenteric fistula is believed to be 15%. The prevalence of carcinoma in a calcified, or “porcelain,” gallbladder is reported to range from 12.5% to 61%. It is generally accepted that adenoma of the gallbladder is a precancerous lesion that presents as a polypoid lesion. Xanthogranulomatous cholecystitis is a rare form of chronic cholecystitis believed to be associated with a higher incidence of cancer. This form of cholecystitis is also important because, grossly, it may mimic cancer of the gallbladder.




41. The preferred treatment for carcinoma of the gallbladder is:
A. Radical resection that includes gallbladder in continuity with the right hepatic lobe and regional lymph node dissection.
B. Radiation therapy.
C. Chemotherapy.
D. Combined treatment involving surgical therapy, chemotherapy, and radiation.
E. None of the above.
Answer: E


DISCUSSION: Radical resection, radiation therapy, and chemotherapy have been effective only anecdotally. Most believe that the dismal prognosis of carcinoma of the gallbladder does not justify anything more than palliative treatment. About 88% of patients are dead within a year of diagnosis, and only about 4% are alive after 5 years, regardless of the type of treatment. Those whose surgeon was unaware of the presence of the tumor at the time of cholecystectomy (approximately 12% of cases) are most likely to survive long term. There are insufficient data to support conclusively the proposition that the patient with unexpected carcinoma found on histologic examination should undergo reoperation with intent for radical excision. There also are indirect suggestions that the prognosis of gallbladder carcinoma may be improving, but it is not clear if this is spontaneous or due to either earlier diagnosis or surgical management.




42. Which of the following statement(s) about pancreatic embryonic malformations is/are correct?
A. Pancreas divisum can be a cause of gastrointestinal bleeding.
B. Heterotopic pancreatic tissue predisposes to pancreatic adenocarcinoma.
C. Annular pancreas may cause gastrointestinal obstruction in children or in adults.
D. Relative obstruction to the flow of pancreatic juice through the minor papilla appears to be the cause of pancreatitis in some patients with pancreas divisum.
Answer: CD


DISCUSSION: The clinically recognized embryonic malformations of the pancreas include heterotopic pancreas, pancreas divisum, and annular pancreas. Heterotopic pancreatic tissue most often takes the form of a firm nodule of variable size in the stomach, duodenum, small bowel, or Meckel's diverticulum. The typical complications of heterotopic pancreas include intestinal obstruction, ulceration, or hemorrhage. Pancreas divisum is an anatomic variant that results from failure of fusion of the two primordial pancreatic duct systems. In pancreas divisum the major portion of the pancreas is drained via the duct of Santorini through the minor duodenal papilla. Relative stenosis of the minor duodenal papilla can cause pancreatitis. Pancreas divisum is not associated with gastrointestinal bleeding. Annular pancreas results when histologically normal pancreatic tissue completely or partially encircles the second portion of the duodenum. Varying degrees of duodenal obstructive symptoms may be observed in both children and adults with this condition.




43. The pancreas occupies a retroperitoneal position in the upper abdomen. Which statement(s) is/are correct?
A. The superior mesenteric vein and the splenic vein join to form the portal vein posterior to the neck of the pancreas.
B. The uncinate process of the pancreas extends posterior to the inferior vena cava.
C. The tail of the pancreas extends to the left of the aorta, toward the splenic hilum.
D. The head of the pancreas is jointly supplied by arterial blood from the celiac axis and the superior mesenteric artery.
Answer: ACD


DISCUSSION: The pancreas occupies a retroperitoneal position in the upper abdomen, extending obliquely from the duodenal C loop to a more cephalad position where the pancreatic tail abuts the hilum of the spleen. The portion of the pancreas anterior to the confluence of the superior mesenteric vein, splenic vein, and portal vein is designated the neck of the gland. The uncinate process extends posterior to the superior mesenteric vein and approaches the superior mesenteric artery. The head of the pancreas is intimately associated with the second portion of the duodenum, and these two structures are jointly supplied by two arterial arcades known as the anterior and posterior pancreaticoduodenal arteries, which originate as branches of the celiac axis and superior mesenteric artery.




44. Both endocrine and exocrine tissue comprise the pancreas. Which statement(s) is/are true?
A. The islets of Langerhans total 1 million per gland and drain their secretions via intercalated duct cells through the ampulla of Vater.
B. Islet alpha cells produce glucagon.
C. Islet sigma cells produce somatostatin.
D. The acini and ductal systems constitute the exocrine portion of the pancreas.
Answer: BD


DISCUSSION: The endocrine portion of the pancreas is served by the islets of Langerhans, which number 1 million islets per gland. The islets of Langerhans drain their endocrine secretions into the bloodstream. Insulin-producing beta cells comprise the majority of the islet population. Alpha cells produce glucagon and constitute approximately 20% to 25% of the total islet cell number. Delta cells of the islets produce somatostatin. The acini and ductal systems constitute the exocrine portion of the pancreas. The acinar cells contain zymogen granules in their narrow, centrally located apical portion. The pancreatic duct system includes intercalated duct cells along the ductal pathway, terminating in the main excretory duct of the pancreas.




45. Pancreatic exocrine secretory products include a bicarbonate-rich electrolyte solution as well as digestive enzymes. Which of the following statement(s) is/are true?
A. Cholecystokinin (CCK) is the most potent endogenous stimulant of pancreatic enzyme secretion.
B. The chloride and bicarbonate concentrations of pancreatic juice vary and depend on the secretory flow rate.
C. Secretin is the most potent endogenous stimulant of pancreatic water and electrolyte secretion.
D. The peptidases synthesized by acinar cells are released into the pancreatic duct system in active form.
Answer: ABC


DISCUSSION: CCK is the most potent endogenous stimulant of pancreatic enzyme secretion. The pancreatic acinar cells respond to CCK with release of their zymogen granules into the ductal system. Peptidases are released in inactive form, later to be activated by contact with duodenal enterokinase and activated trypsin. Secretin is the most potent endogenous stimulant of pancreatic water and electrolyte secretion. The concentrations of the anions bicarbonate and chloride vary and are largely dependent on the secretory flow rate stimulated by secretin.




46. Which of the following parameters is/are not included in the Ranson's prognostic signs useful in the early evaluation of a patient with acute pancreatitis?
A. Elevated blood glucose.
B. Leukocytosis.
C. Amylase value greater than 1000 U per dl.
D. Serum lactic dehydrogenase (LDH) greater than 350 IU per dl.
E. Alanine aminotransferase greater than 250 U per dl.
Answer: CE


DISCUSSION: Several prognostic systems have been demonstrated to predict the severity of pancreatitis accurately. Two Ranson prognostic criteria have been developed: one each, for pancreatitis that is not due to gallstones and pancreatitis that is. The systems have minor differences. In both of the Ranson systems elevated blood glucose, leukocytosis, and elevations of serum LDH have proved to have prognostic importance. The degree of amylase elevation is not one of the parameters, nor is the degree of ALT elevation.




47. Standard supportive measures for patients with mild pancreatitis include the following:
A. Intravenous fluid and electrolyte therapy.
B. Withholding of analgesics to allow serial abdominal examinations.
C. Subcutaneous octreotide therapy.
D. Nasogastric decompression.
E. Prophylactic antibiotics.
Answer: A


DISCUSSION: Standard therapy for all patients with mild acute pancreatitis should include intravenous fluid resuscitation, electrolyte replacement, and analgesics. Nasogastric decompression is typically reserved for patients with significant ileus who are at risk for emesis and aspiration. Subcutaneous therapy with octreotide, the octapeptide analog of somatostatin, has not been proven to influence the outcome in patients with mild pancreatitis. Prophylactic antibiotics are not used for mild pancreatitis. Antibiotics are reserved for patients with severe pancreatitis (defined as greater than three Ranson prognostic signs with associated CT evidence of pancreatic or peripancreatic necrosis).




48. Which of the following statements about chronic pancreatitis is/are correct?
A. Chronic pancreatitis is the inevitable result after repeated episodes of acute pancreatitis.
B. Patients with chronic pancreatitis commonly present with jaundice, pruritus, and fever.
C. Mesenteric angiography is useful in the evaluation of many patients with chronic pancreatitis.
D. Total pancreatectomy usually offers the best outcome in patients with chronic pancreatitis.
E. For patients with disabling chronic pancreatitis and a dilated pancreatic duct with associated stricture formation, a longitudinal pancreaticojejunostomy (Peustow procedure) is an appropriate surgical option.
Answer: E


DISCUSSION: Chronic pancreatitis is a clinical entity that includes recurrent or persistent abdominal pain with evidence of exocrine and endocrine pancreatic insufficiency. While chronic pancreatitis may result from repeated episodes of acute pancreatitis, not all patients with recurring acute pancreatitis progress to chronic pancreatitis. The most common causes of chronic pancreatitis include alcohol abuse, hyperparathyroidism, congenital anomalies of the pancreatic duct, pancreatic trauma, and cystic fibrosis. The most useful radiographic tests in patients with suspected chronic pancreatitis are CT and endoscopic retrograde cholangiopancreatography (ERCP). Mesenteric angiography has no role in the evaluation of most patients with chronic pancreatitis. Patients with disabling chronic pancreatitis who require operative intervention are candidates for a longitudinal pancreaticojejunostomy (Peustow procedure) if pancreatography demonstrates a dilated pancreatic duct. Total pancreatectomy is rarely performed because of the significant problems associated with labile insulin sensitivity, steatorrhea, and weight loss.




49. Which of the following statements about pancreatic ascites is/are correct?
A. Patients typically present with painful ascites, reflecting the release of toxic pancreatic enzymes into the peritoneal cavity.
B. The standard evaluation of a patient with new-onset ascites includes abdominal paracentesis. In cases of pancreatic ascites, the peritoneal fluid contains high concentrations of both amylase and protein.
C. Pancreatic ascites can follow an episode of acute pancreatitis.
D. Patients with pancreatic ascites may fail to improve with nonoperative therapy and require surgical procedures. At abdominal exploration an acceptable approach to the pancreatic duct disruption involves suture ligation with omental patching.
Answer: BC


DISCUSSION: Pancreatic ascites typically occurs because of a pancreatic duct disruption, most commonly involving alcohol abuse and resultant acute pancreatitis. In pancreatic ascites, pancreatic exocrine secretions exit a pancreatic duct disruption and drain anteriorly into the peritoneal cavity. Patients typically present with painless massive ascites, as the pancreatic enzymes that extravasate into the peritoneal cavity are typically nonactivated. The diagnosis of pancreatic ascites is best made by paracentesis, in which the analysis of the ascites fluid reveals it to be high in amylase (more than 1000 U. per dl.) and high in albumin (more than 3 gm. per dl.). Nonoperative treatment is initially indicated in most patients with pancreatic ascites. Should nonoperative therapy fail, surgical therapy is directed to closure of the pancreatic duct disruption. Preoperative pancreatography is useful in directing surgical therapy. Distal pancreatic duct disruption may be treated with distal pancreatectomy or with Roux-en-Y pancreaticojejunostomy. Pancreatic leaks in the more proximal aspects of the gland are treated with Roux-en-Y pancreaticojejunostomy. Suture ligation of the pancreatic duct with omental patching is not considered appropriate therapy for pancreatic duct disruptions.




50. Which of the following statements about adenocarcinoma of the pancreas is/are correct?
A. It is the fifth most common cause of cancer death in the U.S.
B. Most cases occur in the body and tail of the pancreas, making distal pancreatectomy the most commonly performed resectional therapy.
C. For cancers of the head of the pancreas resected by pancreaticoduodenectomy, prognosis appears to be independent of nodal status, margin status, or tumor diameter.
D. The most accurate screening test involves surveillance of stool for carbohydrate antigen (CA 19–9).
Answer: A


DISCUSSION: Adenocarcinoma of the pancreas is newly diagnosed in approximately 28,000 patients in the United States every year. It is the fifth most common cause of cancer death in the United States, exceeded only by lung, colorectal, breast, and prostate cancer. The majority of cases of adenocarcinoma of the pancreas occur in the head of the gland, and if resectable, are treated via pancreaticoduodenectomy. Recent studies have shown that factors favoring long-term survival after pancreaticoduodenectomy for adenocarcinoma of the head of the pancreas include negative nodal status, negative margin status, small tumor diameter, and diploid DNA content. No accurate screening tests for adenocarcinoma of the pancreas are currently available. The best serologic test appears to be the CA 19–9, which is elevated in the majority of patients with adenocarcinoma of the head of the pancreas. Unfortunately, the test is not sufficiently sensitive or specific, and further screening tests are needed.




51. A 35-year-old woman presents with episodes of obtundation, somnolence, and tachycardia. An insulinoma is suspected based on a random serum glucose test value of 38 mg. per dl. Which of the following statements is/are true?
A. The most important diagnostic study for insulinoma is an oral glucose tolerance test.
B. It may be helpful to perform ERCP in an effort to localize the tumor.
C. Most patients with insulinoma present with extensive disease, rendering them only rarely resectable or curable.
D. An important component of the preoperative evaluation in patients with presumed insulinoma involves confirming elevated C-peptide or proinsulin levels and screening for anti-insulin antibodies.
Answer: D


DISCUSSION: Insulinoma is the most common endocrine tumor of the pancreas. Insulinoma is associated with Whipple's triad, which consists of (1) symptoms of hypoglycemia at fasting; (2) documentation of blood glucose levels of less than 50 mg. per dl.; and (3) relief of symptoms following administration of glucose. The most reliable method for diagnosing insulinomas is a monitored fast. Neither an oral or an intravenous glucose tolerance test is indicated in the majority of patients being evaluated for insulinoma. Support for the diagnosis of insulinoma can come from documenting elevated C peptide and proinsulin levels. Screening for anti-insulin antibodies is indicated to rule out the possibility of surreptitious insulin administration. Tumor localization is typically performed with CT, endoscopic ultrasonography, or angiography. ERCP is not indicated for evaluation of most pancreatic endocrine tumors, as the tumors only rarely communicate with the main pancreatic duct system. As many as 90% of patients with insulinoma have benign solitary pancreatic adenomas amenable to surgical cure.




52. Which of the following statements about gastrinoma (Zollinger-Ellison syndrome) is/are correct?
A. As many as 75% of gastrinoma patients have sporadic disease; 25% have gastrinoma associated with multiple endocrine neoplasia type 1 (MEN 1).
B. Extrapancreatic gastrinomas are common, and exploration should include careful assessment of the duodenum and peripancreatic lymph nodes.
C. Diarrhea may be a prominent presenting feature of some patients with gastrinoma.
D. Before elective operation acid-reducing medications such as omeprazole should be administered.
Answer: ABCD


DISCUSSION: Gastrinoma patients typically present with peptic ulceration of the upper gastrointestinal tract and abdominal pain. As many as 50% of patients may have diarrhea, which may be a prominent feature in some cases. Approximately 25% of gastrinoma patients have the disease associated with the MEN-1 syndrome, whereas 75% have a sporadic variety of the disease. Recent evidence indicates that extrapancreatic gastrinomas are common. Careful attention must be paid to the duodenum and peripancreatic lymph nodes at the time of abdominal exploration. Before elective operation it is imperative that the gastric acid hypersecretion be controlled. The control of gastric hypersecretion is best performed by the administration of one of the substituted benzimidazoles, such as omeprazole or lansoprazole.




53. With regard to the control of pancreatic exocrine function, which of the following statement(s) is/are correct?


a. Cholecystokinin, a hormone released from the duodenal mucosa, is the predominant stimulus for pancreatic enzyme secretion
b. Gastrin is a major stimulant for pancreatic bicarbonate secretion
c. Secretin is released from the duodenum upon mucosal acidification and stimulates pancreatic bicarbonate secretion
d. Acetylcholine, released from pancreatic nerves, stimulates enzyme secretion
Answer: a, c, d


Enzyme secretion is regulated primarily through hormonal and neural factors. The enteric hormone cholecystokinin, released from endocrine cells in the duodenal mucosa, is the predominant regulator and stimulates acinar cells through specific membrane-bound receptors. Acetylcholine strongly stimulates acinar cells when released from postganglionic fibers of the pancreatic plexus and acts in synergy with CCK to potentiate enzyme secretion. Secretin weakly stimulates acinar cell secretion and potentiates the effect of cholecystokinin on the acinar cells.
Bicarbonate is formed from carbonic acid by the enzyme carbonic anhydrase. Secretin, the major stimulant for bicarbonate secretion, is released from the duodenal mucosa in response to a duodenal luminal pH of less than 3.0. Cholecystokinin only weakly stimulates bicarbonate secretion, whereas it potentiates secretin-stimulated bicarbonate secretion. Gastrin and acetylcholine are weak stimulants of bicarbonate secretion.




54. In the performance of a pancreaticoduodenectomy (Whipple procedure), the superior mesenteric vein is an important landmark. Which of the following statements is/are true with regard to the superior mesenteric vein?


a. Small venous branches enter the superior mesenteric vein anteriorly as it courses beneath the neck of the pancreas
b. The superior mesenteric vein joins the splenic vein at the superior border of the pancreas to form the portal vein
c. Small venous branches enter the superior mesenteric vein laterally as it courses beneath the neck of the pancreas
d. The superior mesenteric vein courses anterior to the neck of the pancreas
Answer: b, c


The venous drainage of the pancreas and duodenum follows the arterial supply. The anterior and posterior venous arcades drain the head; the body and tail drain into the splenic vein. All venous effluent from the pancreas ultimately drains into the portal vein which is formed by the confluence of the superior mesenteric vein and the splenic vein at the superior border of the pancreas. The anterior and posterior venous arcades in the head of the pancreas drain directly into the suprapancreatic portal vein. The anteroinferior pancreaticoduodenal arcades drain with the right gastroepiploic vein to form a common venous trunk with the right colic vein. This trunk is known as the gastrocolic trunk and enters the superior mesenteric vein at the inferior border of the neck of the pancreas. The posteroinferior venous arcade empties directly into the superior mesenteric vein. The veins of the head drain laterally into the superior mesenteric and portal veins. There are no venous tributaries entering the superior mesenteric vein anteriorly. For this reason, it is safe to dissect the neck of the pancreas directly anterior to the superior mesenteric and portal veins when performing a pancreaticoduodenectomy.




55. Pancreas divisum results from incomplete fusion of the ventral pancreatic duct with the dorsal pancreatic duct during embryologic development. Which of the following statements correctly describes pancreas divisum?


a. The body and tail of the pancreas drain via an accessory ampulla distal to the ampulla of Vater. The uncinate process drains via the ampulla of Vater
b. The entire pancreatic ductal system drains via the ampulla of Vater
c. The entire pancreatic ductal system drains via an accessory ampulla proximal to the ampulla of Vater
d. The body and tail of the pancreas are absent. The uncinate process drains via the ampulla of Vater
Answer: c


In 90% of individuals, the main pancreatic duct, or duct of Wirsung, runs the entire length of the pancreas and joins the common bile duct to empty into the duodenum at the ampulla of Vater. The pancreatic duct is 2 to 3.5 mm in diameter and contains 20 secondary branches, which drain the tail, body, and uncinate process. The drainage of the lesser duct, or duct of Santorini, is variable. The lesser duct commonly drains the superior portion of the head of the pancreas. It empties separately into the second portion of the duodenum through the lesser papilla located 2 cm proximal to the ampulla of Vater. Pancreas divisum results from an incomplete fusion of the ventral pancreatic duct with the dorsal duct during fetal development and is present in 5% of patients. In this anomaly, the lesser duct drains the entire pancreas via an accessory ampulla located proximal to the ampulla of Vater. Inadequacy of this pattern of drainage can result in chronic pain.




56. Which of the following statements is/are correct with regard to the blood supply of the pancreas?


a. The inferior pancreaticoduodenal artery, a branch of the celiac artery, divides into anterior and posterior branches to supply the pancreatic head
b. The body and tail of the pancreas are supplied by branches of the splenic artery
c. The superior pancreaticoduodenal artery is a branch of the gastroduodenal artery
d. The body and tail of the pancreas are supplied by branches derived from the left renal artery
Answer: b, c


The pancreas receives its blood supply from a variety of major arterial sources. In the head of the pancreas, there are arcades in the anterior and posterior surfaces, which generally collateralize. These arcades arise from branches of the gastroduodenal and the superior mesenteric arteries. Just distal to the first portion of the duodenum, the gastroduodenal artery becomes the superior pancreaticoduodenal artery, which divides into anterior and posterior branches. The inferior pancreaticoduodenal artery is the first branch of the superior mesenteric artery and divides into anterior and posterior branches.
The body and tail of the pancreas are supplied by the splenic artery. The splenic artery arises from the celiac trunk and courses along the superior surface of the pancreas to the spleen. Approximately ten branches of the splenic artery supply the body and tail of the pancreas.




57. Orally administered glucose provokes a greater insulin response than an equivalent amount of intravenously administered glucose. The incremental response to ingested glucose is due to the effects of which of the following hormones?


a. Gastric inhibitory peptide
b. Somatostatin
c. Pancreatic polypeptide
d. Secretin
Answer: a


Orally administered glucose stimulates a greater insulin response than an equivalent amount of intravenous glucose through the release of enteric hormones that potentiate insulin secretion. This effect is known as the enteroinsular axis. Gastric inhibitory polypeptide (GIP) appears to be an important regulator of this effect, although other gut peptides, such as glucagon-like peptide I (GLP-1), may contribute to this effect as well. Nutrients that regulate insulin secretion include amino acids, such as arginine, lysine, and leucine, and free fatty acids. Hormones that stimulate insulin secretion include glucagon, GIP, and cholecystokinin, whereas somatostatin, amylin, and pancreastatin are inhibitory. Insulin is also stimulated by sulfonylurea compounds, which act independently of the glucose concentration and form the basis of treatment of type II, or insulin-independent, diabetes.




58. The islets of Langerhans contain four major endocrine cell types that secrete which of the following hormones?


a. Insulin, somatostatin, glucagon, secretin
b. Insulin, somatostatin, cholecystokinin, pancreatic polypeptide
c. Insulin, somatostatin, glucagon, pancreatic polypeptide
d. Insulin, secretin, glucagon, cholecystokinin
Answer: c


Within the pancreas are small nests of cells that are responsible for the secretion of hormones that control glucose homeostasis. These nests are called islets of Langerhans and constitute 2% of the pancreatic mass. The islets contain an average of 3000 cells and range in diameter from 40 to 900 mm. The islets are composed of four major cell types—alpha (A), beta (B), delta (D), and PP or F cells, which secrete glucagon, insulin, somatostatin, and pancreatic polypeptide, respectively. The B cells are centrally located within the islet and constitute 70% of the islet mass, whereas the PP, A, and D cells are located at the periphery of the islet. They constitute roughly 15%, 10%, and 5% of the islet cell mass, respectively.




59. A 50-year-old man develops acute pancreatitis due to alcohol abuse. Hyperamylasemia resolves by the third day after admission. By the eighth hospital day, the patient is noted to have recurrent fever (38.5°C), progressive leukocytosis (18,500 WBC/mm3), and tachypnea. The most appropriate management includes which as the next step?


a. Laparotomy with pancreatic debridement
b. CT guided aspiration of peripancreatic fluid collections
c. ERCP with sphincterotomy and placement of biliary stent
d. Intravenous amphotericin B
Answer: b


The common causes of pancreatic abscesses are infected pancreatic pseudocysts and necrotizing pancreatitis. The diagnosis is suggested by persistent fever, leukocytosis, and a palpable abdominal mass. Bacteremia and systemic toxicity are late clinical features. Percutaneous aspiration with positive cultures is the definitive preoperative test, facilitated by CT scanning or ultrasound-guidance to suspicious peripancreatic fluid collections. When diagnosed, the treatment of choice is wide surgical débridement with removal of all infected and revitalized tissues. Generous drainage is mandatory. One of the major sources of morbidity and mortality in this situation is the late development of mycotic visceral pseudoaneurysms, particularly involving the splenic circulation. These may be complex management problems, requiring angiographic embolization or other innovative treatment strategies.




60. The patient in the above question is treated by observation for 8 weeks. He continues to be symptomatic with epigastric pain. A repeat abdominal CT scan reveals a persistent 6 cm pseudocyst in the region of the body of the pancreas. The pseudocyst is unilocular and demonstrates a well-defined rim of fibrous tissue. The gastric antrum is displaced anteriorly. Using CT guidance, 300 ml of fluid is aspirated from the lesion which is shown to be collapsed radiographically. No further intervention is performed. What is the risk of pseudocyst recurrence after simple aspiration?


a. 80–85%
b. 60–65%
c. 40–45%
d. 20–25%
Answer: d


Generally, a pancreatic pseudocyst can be observed for a period of weeks or months in an effort to allow for spontaneous resolution. Percutaneous ultrasound-or CT-directed aspiration or drainage catheter placement is an initial treatment option. Simple aspiration is performed if the initial aspirate is sterile; if the aspirate is infected, a catheter or open drainage procedure is appropriate. Determination of pancreatic ductal anatomy is important. Contrast injection into the pseudocyst at the time of aspiration should be considered to assess the possibility of pancreatic ductal communication and obstruction, or multiple cysts. The pseudocyst recurrence rate after simple aspiration is about 20% to 25%.




61. In prospective, randomized trials which of the following agents or therapeutic measures has/have been demonstrated to accelerate recovery from acute pancreatitis?


a. Peritoneal lavage
b. Anticholinergic blockade
c. Octreotide
d. H2 receptor blockade
e. None of the above
Answer: e


A variety of pharmacologic agents that directly or indirectly reduce acinar cell enzyme release or ductal secretion have undergone clinical evaluation for the treatment of acute pancreatitis—generally with unimpressive results. Among the first were anticholinergic drugs. Despite extensive experience over many years, no objective data have emerged to support their use. Clinical trials of glucagon and calcitonin based on the same principle have produced a similar lack of supportive data. More recently, a somatostatin analog has been subjected to clinical trials for patients with acute pancreatitis. Somatostatin inhibits pancreatic enzyme and bicarbonate secretion by preventing the normal release of cholecystokinin, secretin, and other gut peptides. Despite the theoretical appeal, it has not been possible to demonstrate that somatostatin alters the natural history or prognosis of simple acute pancreatitis, although it does diminish pancreatic secretion.
Peritoneal lavage as a specific therapy for acute pancreatitis was proposed after experimental studies demonstrated improved survival in animals with fulminant pancreatitis. The concept was appealing in that activated proteases and other vasoactive substances identifiable in peritoneal aspirates from patients with pancreatitis would be removed, rather than systemically absorbed. Unfortunately, clinical trials using this approach have produced disappointing results, and the eventual overall mortality rate appears unchanged.




62. Which of the following medical procedures has/have been associated with an increased risk of post-procedure acute pancreatitis?


a. Common bile duct exploration
b. Endoscopic retrograde cholangiopancreatography
c. Coronary bypass grafting
d. Distal gastrectomy
Answer: a, b, c, d


Many surgical procedures in the upper abdomen are associated with postoperative pancreatitis. The incidence of acute pancreatitis after gastric resection ranges from 0.6% to 1.23%. After biliary tract surgery, particularly after common bile duct exploration itself, acute pancreatitis occurs with an incidence of 0.5% to 3%. Direct manipulation or retraction of the pancreas or pancreatic duct appears to be the most common cause. About 1% of patients develop acute pancreatitis after endoscopic retrograde cholangiopancreatography (ERCP). This is a predictable event, and the risk can be minimized by limiting the pressure used for contrast injection of the pancreatic duct. Acute pancreatitis also occurs in patients after coronary artery bypass surgery and a variety of other procedures remote from the pancreas. Although pancreatitis in this circumstance is thought to result from ischemia, hypotension is not always noted. The systemic consequences of activation of the inflammatory system may contribute to changes in microvascular blood flow.




63. A 42-year-old male develops acute pancreatitis in the setting of acute alcohol abuse. One week after onset of symptoms, computed tomography of the abdomen reveals a pancreatic phlegmon and associated pseudocyst. Which of the following factors, if present, would decrease the likelihood of spontaneous resolution of the pseudocyst?


a. Size greater than 5 cm
b. Diffuse calcification of the pancreatic gland
c. Multilocularity
d. Location in the pancreatic tail
Answer: a, b, c


Initial management of pancreatic pseudocysts is based on symptoms. If the patient is asymptomatic and the cyst is small (< 5.0 cm) it can be safely observed as many of these will resolve over a period of weeks. Concurrent chronic alcoholic pancreatitis (by history or as indicated by pancreatic calcification), pseudocyst size greater than 5 cm, the presence of a multilocular or debris-filled pseudocyst cavity, and chronicity (longer than 6 weeks) are all factors that are associated with a lower probability of spontaneous resolution. 64. Which of the following is/are prognostic signs reported by Ranson to predict outcomes associated with acute pancreatitis? a. Age greater than 60 years b. Hematocrit decrease of 105 within 48 hours of hospital admission c. Serum amylase value greater than 4 times upper limit of normal d. Serum glucose greater than 200 mg/dL on admission e. Ca2+ level less than 8 mg/dL within 48 hours of hospital admission Answer: b, d, e Ranson prognostic signs include: ON ADMISSION Age above 55 years White blood cell count above 16,000/µL Glucose level above 200 mg/dL Lactase dehydrogenate level above 350 IU/L SGOT value above 250 IU/L AFTER 48 HOURS Hematocrit decrease of 10% Blood urea nitrogen level increase of 5 mg/dL Ca2+ level below 8 mg/dL PaO2 level below 60 mmHg Base deficit value above 4 mEq/L Fluid sequestration greater than 6 L 65. A 36-year-old woman is admitted to a the hospital with upper abdominal pain, hyperamylasemia, elevation of serum alkaline phosphatase and ultrasound evidence of cholelithiasis. With intravenous hydration and analgesia, symptoms rapidly resolved. After 48 hours, serum amylase and alkaline phosphatase values had returned to normal and physical examination revealed lessening tenderness in the right upper quadrant of the abdomen. Appropriate management consists of which of the following as the next step? a. Cholecystectomy and intraoperative cholangiography before hospital discharge b. Elective cholecystectomy at approximately 8 weeks c. Endoscopic sphincterotomy before discharge followed by cholecystectomy at approximately 8 weeks d. Observation Answer: a A patient who has simple cholelithiasis and an episode of acute pancreatitis is usually treated nonoperatively until clinical resolution of the pancreatitis occurs. The rate of recurrent biliary pancreatitis is as high as 34% to 56% within 6 weeks; therefore, an aggressive operative approach is appropriate. Cholecystectomy is often performed after the resolution of acute pancreatitis but before hospital discharge. Common bile duct instrumentation in this setting has a substantially increased risk of recurrent acute pancreatitis. 66. For the patient in the preceding question, symptomatic recurrence at 3 weeks after aspiration is confirmed ultrasonographically. Endoscopic retrograde pancreatography does not demonstrate communication of a major pancreatic duct with the pseudocyst. Appropriate management includes which of the following? a. Pancreatectomy to include the pseudocyst b. Cystgastrostomy c. Repeat aspiration followed by injection of sodium morrhuate into the pseudocyst cavity d. Pancreatic debridement followed by external drainage Answer: b The operative treatment for pseudocysts depends on the underlying cause of the cyst, as well as the size, location, and maturity of the pseudocyst wall. Whenever possible, the status of the pancreatic duct should be assessed preoperatively, preferably by ERCP. Operative drainage can be either external or internal. External drainage is chosen in the presence of infection or an immature capsule. The disadvantages of external drainage include the risk of pancreatic fistula formation and a pseudocyst recurrence. External drainage has been associated with a higher mortality rate, probably because it is used in patients at higher risk, especially those with sepsis, pancreatic abscesses, or ruptured pseudocysts. The type of internal drainage procedure selected depends on the location of the pseudocyst and whether or not there is associated pancreatic ductal pathology. Cystogastrostomy is the simplest and safest alternative if the pseudocyst is appropriately adjacent to the posterior wall of the stomach. Cystojejunostomy using a Roux-en-Y or loop jejunostomy may also be appropriate, depending on the location and specific anatomy of the pseudocyst. Pancreatic resection is associated with the lowest recurrence rate (3%), but is limited to pseudocysts occurring in the tail of the pancreas. 67. With regard to acute pancreatitis: which of the following statements is/are correct? a. The majority of patients presenting with acute pancreatitis of biliary type are female b. The majority of patients presenting with acute pancreatitis of alcoholic type are female c. The most common cause of acute pancreatitis in the United States is alcohol use d. Patients with alcohol-induced pancreatitis tend to be older than those with biliary-induced disease Answer: a, c In autopsy series, the evidence for past acute pancreatitis averages 0.31%. Variations among populations are highly dependent on social factors such as ethanol use and on environmental and hereditary determinants such as the incidence of gallstones. Acute pancreatitis may occur at any age but is most common in adults between 30 and 70 years of age. In general, patients with gallstone-induced pancreatitis are older (age 40 to 60 years), whereas those with alcohol-associated pancreatitis are younger (age 30 to 40 years). The sex distribution of acute pancreatitis depends on the clinical cause of the disease, with women representing 68% of patients with gallstone-associated pancreatitis. Conversely, when alcohol is the primary association, men account for most patients. Clinical associations with acute pancreatitis can be divided into three broad categories-biliary stones, ethanol, and others. Biliary tract stone disease and ethanol-induced pancreatitis account for most cases of acute pancreatitis reported worldwide. The particular distribution of causes reflects the source of the patient population evaluated. In a summary of 18 different reports of acute pancreatitis in the United States with a combined total of 7147 patients, 53% of patients were believed to have ethanol-induced disease, whereas 28% had proven biliary stones. In contrast, of 1539 patients reported on from Great Britain, 52% had gallstones, 7% were ethanol related, and 34% had no identifiable cause. 68. Which of the following statement(s) relating to chronic pancreatitis is/are correct? a. In the United States, the most common cause of chronic pancreatitis is alcohol abuse b. Approximately 50% of chronic alcoholics develop chronic pancreatitis c. Clinically significant chronic pancreatitis develops on average after five years of alcohol abuse in men d. The risk of alcohol-induced chronic pancreatitis can be decreased by consumption of a high-protein diet Answer: a In the United States, alcohol consumption is the major cause of chronic pancreatitis: with approximately 70% of cases attributable to this factor. Most patients with symptomatic chronic pancreatitis have consumed large volumes of alcohol daily for a prolonged period of time. The average daily intake of alcohol is 150 to 175 g with the mean duration of alcoholism before recognition of pancreatitis being 18 years for men and 11 years for women. The incidence of chronic pancreatitis on autopsy studies of chronic alcoholics is 50 times the rate of non-drinking controls. Only 10% of alcoholics develop chronic pancreatitis—suggesting that factors other than long-term alcohol exposure may also influence susceptibility. In both experimental and clinical studies, the risk of alcohol-induced chronic pancreatitis is increased by a high-protein, high-fat diet. 69. The most appropriate test to confirm a clinical diagnosis of early chronic pancreatitis is which of the following? a. Serum amylase determination b. Calculation of urinary amylase clearance c. Measurement of para-aminobenzoic acid absorption d. Endoscopic retrograde cholangiopancreatography Answer: d Routine tests of blood or serum are not helpful in making a diagnosis of chronic pancreatitis. Although serum amylase levels are almost always elevated in acute pancreatitis—amylase levels may be normal, elevated, or subnormal in chronic pancreatitis. Determination of urinary amylase secretion and calculation of urinary amylase clearance does not improve sensitivity or specificity. Indirect tests of pancreatic function which measure absorption of nutrients that first require pancreatic digestion are not helpful in early cases of chronic pancreatitis. Clinically detectable malabsorption is absent until 90% of exocrine function is lost. Because of this, indirect tests of pancreatic function do not detect early disease. In addition, false positive tests may occur in other disease states associated with malabsorption (Crohn’s disease, sprue, postgastrectomy states, or in association with diabetes mellitus, cirrhosis, or renal disease. ERCP has become widely recognized as the most sensitive and reliable method for diagnosing chronic pancreatitis. Sensitivity approaches 90% with equal specificity. 70. A 52-year-old male, known to be alcoholic, is evaluated because of chronic abdominal pain. The clinical diagnosis of chronic pancreatitis is supported by ERCP findings of pancreatic ductal ectasia with alternating areas of stricture and dilatation. Several pancreatic ductal stones are also noted. With chronic pain as the operative indication, the most appropriate procedure would be: a. 80% distal pancreatectomy with splenectomy b. Longitudinal pancreaticojejunostomy c. Distal pancreatectomy with end pancreaticojejunostomy d. Total pancreatectomy Answer: b When patients with chronic pancreatitis have pancreatic ductal dilatation (greater than 8 mm. ductal decompression using longitudinal pancreaticojejunostomy may be employed for relief of pain. The finding that pancreatic ductal hypertension exists in patients with painful chronic pancreatitis and that surgical decompression reduces intrapancreatic pressure to normal provides the rationale for this operation. The anterior surface of the pancreas is exposed through the lesser sac. The entire pancreatic duct is opened from the pancreatic tail to a point 1 cm from the duodenum. A side-to-side anastomosis is then performed between the opened pancreatic duct and a loop of jejunum. Splenectomy is not necessary. In-hospital mortality rates of less than 5% have been widely reported. Approximately 80% of patients report complete or substantial improvement of pain following longitudinal pancreaticojejunostomy. 71. For the patient in the preceding question, the most appropriate long-term management is which of the following? a. Endoscopic stenting of the distal common bile duct b. Choledochoduodenostomy c. Pancreaticoduodenectomy (Whipple procedure) d. Percutaneous transhepatic drainage of the common hepatic duct Answer: b Operative management of patients with stricture of the common bile duct associated with chronic pancreatitis is justified to treat symptoms and to prevent development of biliary cirrhosis. Operative indications include progressive jaundice, cholangitis, liver biopsy evidence of biliary cirrhosis, persistent elevation of alkaline phosphatase at greater than three times normal, and progressive stricture demonstrated by radiologically progressive dilatation of extrahepatic and intrahepatic biliary ducts. Both choledochoduodenostomy and choledochojejunostomy are excellent operative choices. 72. Which of the following is the most common cause of obstructive jaundice in patients with chronic pancreatitis? a. Adenocarcinoma of the head of the pancreas b. Choledocholithiasis c. Fibrotic stricture of the common bile duct d. Pancreatic pseudocyst formation Answer: c Biliary complications involving the common bile duct can occur in chronic pancreatitis because of the intimate association of that structure with the head of the pancreas. In two-thirds of individuals, the common bile duct traverses the pancreatic parenchyma and in an additional 25%, the common bile duct lies within a groove along the posterior surface of the pancreas. Fibrosis associated with chronic pancreatitis can encase and compress the common bile duct. Common bile duct stenosis is relatively common in chronic pancreatitis, occurring in approximately 10% of cases observed long-term. Cholangiography typically reveals a long, gradually tapering stricture conforming to the intrapancreatic portion of the common bile duct. In contrast, malignant strictures usually result in abrupt termination of the biliary duct. The proximal suprapancreatic portion of the bile duct is variably dilated. 73. Alcohol-induced and hereditary chronic pancreatitis are the two most common etiologies observed in North American patients. Most of the remaining patients fall into which of the following categories? a. Chronic pancreatitis secondary to hyperparathyroidism b. Chronic pancreatitis caused by protein-calorie malnutrition c. Chronic pancreatitis secondary to congenital pancreatic ductal obstruction d. Idiopathic chronic pancreatitis Answer: d After alcohol-induced and hereditary disease, idiopathic chronic pancreatitis is the most common cause of calcifying pancreatitis in North American patients. This designation is given to those cases without a recognizable cause. Idiopathic pancreatitis accounts for about 15% of the cases and has two peaks in incidence, suggesting that differing underlying causes may exist. The first peak occurs in young adulthood and the second has an occurrence at approximately 60 years of age. 74. Which of the following statements regarding prognosis in chronic pancreatitis is/are correct? a. Patients with chronic pancreatitis have decreased long-term survival compared with the general population b. Patients with chronic pancreatitis exhibit no excess mortality relative to the general population c. Excess mortality in patients with chronic pancreatitis is related to cancers of the aerodigestive system, complications of diabetes, and complications of cirrhosis d. Excess mortality in patients with chronic pancreatitis is due to development of adenocarcinoma of the pancreas and to the complications of recurrent pancreatitis Answer: a, c Patients with chronic pancreatitis have decreased long-term survival compared to the general population. An excess of mortality of 30% over 20 years has been estimated. Less than 20% of deaths are directly attributable to pancreatitis or its complications. Excess mortality is related to extrapancreatic complications of alcoholism and smoking, including cancers of the aerodigestive system, complications of diabetes, and complications of cirrhosis. 75. Which of the following is the most common clinical manifestation of chronic pancreatitis? a. Epigastric pain with radiation to the hypogastrium b. Diabetes mellitus c. Steatorrhea d. Epigastric pain with radiation to the upper lumbar vertebrae Answer: d Pain is a predominant symptom complex in most patients with chronic pancreatitis. Chronic pancreatic pain is usually localized to the epigastrium with radiation to the back in the region of the upper lumbar vertebrae. Discomfort may be exacerbated by eating and is usually alleviated by abstinence from food and by bending forward. Malabsorption and weight loss, clinical manifestations of steatorrhea, are only observed when greater than 90% of exocrine tissue has been destroyed. Clinical signs of malabsorption are a late manifestation of chronic pancreatitis. Although abnormal glucose tests can be demonstrated in 50% to 70% of patients with chronic Pancreatitis: overt diabetes mellitus is present in only 30% to 40%. Endocrine deficits are usually progressive. If individual patients are repetitively tested, progressive deterioration is often observed. 76. For the patient in the preceding question, appropriate management includes which of the following? a. Distal pancreatectomy b. Cystjejunostomy c. Percutaneous drainage d. Primary radiotherapy and chemotherapy Answer: a The proper treatment is surgical removal of the tumor; aggressive pancreatic resection is appropriate. It is crucial to avoid mistaking a mucinous cystic tumor for a pancreatic pseudocyst. Internal drainage of a malignant mucinous cystic tumor results in catastrophic tumor dissemination and should never be performed. With appropriate treatment, all patients with histologically benign tumors should be cured; for tumors demonstrating malignant change, 5-year survival after surgery is about 60%. 77. A 72-year-old man develops jaundice and is demonstrated to have a 2.5 mass in the pancreatic head by computed tomography. There are no signs of unresectability on CT examination. Fine needle aspiration cytology is positive for adenocarcinoma. Which of the following intraoperative findings would indicate unresectability? a. Fibrotic reaction in the body and tail of the pancreas b. Microscopic tumor cells in perigastric lymph nodes on frozen section c. Inability to develop an avascular plane anterior to the superior mesenteric vein d. Cholelithiasis Answer: b, c During performance of pancreaticoduodenectomy, the lesser sac is opened widely through the gastrocolic omentum. This maneuver allows inspection of the body and tail of the gland to determine the extent of the tumor involvement and allows examination of lymph nodes along the superior and inferior body of the pancreas and around the celiac axis. Enlarged nodes in these areas should undergo biopsy and be submitted for frozen-section examination, since tumor in these areas is beyond the bounds of standard pancreaticoduodenectomy and constitutes a contraindication to resection. If there is no evidence of lymphadenopathy, a dissection between the anterior surface of the portal vein and the posterior surface of the neck of the pancreas is performed. Ordinarily, only thin areolar tissue lies between the pancreas and the portal vein, and a communication behind the neck of the pancreas can be established. If there is hard tissue intervening and such communication cannot be established, this implies invasion of the anterior surface of the portal vein and signals unresectability by standard methods. 78. A 67-year-old male presents with complaints of itching, dark urine, and epigastric pain. Physical examination reveals jaundice. Initial laboratory tests show total bilirubin of 6.5 mg/dL, alkaline phosphatase elevated at 3 the upper limit of normal, and mild elevations in serum transaminases. Appropriate management includes which diagnostic test next? a. Abdominal ultrasonography b. Computed tomography of the abdomen c. Magnetic resonance imaging of the abdomen d. Endoscopic retrograde cholangiography Answer: a Standard transcutaneous ultrasonography is the appropriate first test in the evaluation of the patient with jaundice, because the presence of a dilated common bile duct or intrahepatic bile ducts is essentially diagnostic of extrahepatic biliary obstruction. This finding directs the physician to a search for the cause of the obstruction. If the bile ducts are not dilated, mechanical obstruction is unlikely and the diagnostic thrust should move toward hepatocellular disease. Ultrasonography is also the best test to determine whether gallstones are present; this is extremely important because choledocholithiasis is one of the conditions most likely to cause jaundice in the elderly population. 79. Which of the following statements regarding ductal adenocarcinoma of the pancreas is/are correct? a. For ductal adenocarcinoma, 60–70% of tumors arise in the head of the gland, 15% in the body, and 10% in the tail, the remainder are diffuse b. Fifty percent of pancreatic adenocarcinomas involve the gland diffusely at the time of diagnosis c. For ductal adenocarcinomas, tumors of the body and tail are usually larger at the time of diagnosis than those arising in the head of the gland d. Pancreatic adenocarcinomas occur with equal frequency within the head, body, and tail of the gland Answer: a, c Sixty to 70% of pancreatic ductal adenocarcinomas occur in the head of the gland. About 15% reside in the body of the gland, another 10% are in the tail, and the remaining 5% to 15% are diffuse. The predilection of pancreatic cancer to develop in the head of the gland is unexplained, but has the practical consequence that tumors in the head are diagnosed earlier because they cause obstructive jaundice whereas tumors in the body and tail tend to be more advanced at the time of symptomatic presentation. Tumors in the body and tail are typically larger at the time of diagnosis (average, 7 to 8 cm) than in the head (average, 4 to 5 cm). 80. The most common cause of death in the postoperative period following pancreaticoduodenectomy is which of the following? a. Myocardial infarction b. Intraperitoneal hemorrhage c. Pulmonary embolism d. Pneumonia Answer: b Pancreaticoduodenectomy is a formidable operation, and until recently, average operative mortality was reported to approximate 20%. In the past few years, several centers have reported large series with operative mortalities lower than 5%. The most dreaded complication of pancreaticoduodenectomy is disruption of the pancreaticojejunostomy, which occurs in about 10% of patients. Anastomotic breakdown may lead to the development of an upper abdominal abscess or may present as a external pancreatic fistula. In its most virulent form, disruption leads to necrotizing retroperitoneal infection which may erode major arteries and veins of the upper abdomen, including the portal vein or its branches or the stump of the gastroduodenal artery. Impending catastrophe is often preceded by a small herald bleed from the drain site. Such an event is an indication to return to the operating room to widely drain the pancreaticojejunostomy and to repair the involved blood vessel. Open packing of the wound may be necessary in controlling diffuse necrosis and infection. On rare occasions, completion pancreatectomy is required to control sepsis. Intraperitoneal hemorrhage is the most common cause of death from pancreaticoduodenectomy. 81. For the above patient, ultrasonography reveals dilated extrahepatic and intrahepatic bile ducts and the absence of gallstones within the gallbladder. A 2 cm mass within the pancreatic head is visualized. Computed tomography of the abdomen is performed. Which of following CT findings indicate probable unresectability? a. Common bile duct diameter of 2 cm b. Ascites c. Infiltration of the transverse colonic mesentery d. Dilatation of the main pancreatic duct to 1.5 cm Answer: b, c CT scans provide the best available radiologic information to determine whether or not a pancreatic neoplasm is resectable, but they cannot be considered absolutely definitive in this regard. Only about half of pancreatic tumors that appear to be confined to the pancreas on CT scan are found to be resectable in the operating room. CT scanning is more accurate in the diagnosis of unresectability. CT findings that indicate that the tumor is unlikely to be surgically curable include vascular invasion, enlarged lymph nodes outside the boundaries of resection, ascites, distant metastases (usually liver), and distant organ invasion (usually colon). When a CT scan shows distant metastases or extensive local invasion, the positive predictive value of the technique is high; some 90% of such patients have unresectable disease at laparotomy. 82. Which of the following have been shown to be risk factors for development of adenocarcinoma of the pancreas? a. Cigarette smoking b. Coffee drinking c. Adult-onset diabetes mellitus d. Chronic coumadin usage e. Prior gastrectomy Answer: a, e Most cases of pancreatic cancer have no obvious predisposing host factors. The most consistently observed risk factor for pancreatic cancer is cigarette smoking. Most studies estimate that smoking results in a two-to three-fold increase in risk of developing pancreatic cancer. Alcohol consumption has been implicated in some case-control studies of pancreatic cancer, but the overall evidence is inconsistent and alcohol is not likely to be a major factor in the development of the disease. Although considerable public interest was focused in the past few years on coffee consumption as a risk factor for pancreatic cancer, evidence linking coffee consumption to pancreatic cancer is not compelling. Abnormal glucose tolerance is present in about 80% of patients with pancreatic cancer, if carefully sought. Although diabetes and pancreatic cancer occur together far more frequently than would be expected by chance, persons with long-standing diabetes are not at increased risk of developing pancreatic cancer. Chronic pancreatitis is a significant risk factor for the development of pancreatic cancer. It appears that all forms of chronic pancreatitis are associated with an increased risk of pancreatic cancer, suggesting that it is the pancreatitis and not the injuring agent which is responsible for the augmented cancer risk. Studies indicate that patients who have previously undergone gastric resection may be from three to seven times more likely to develop pancreatic cancer than a control population. There have been several reports of familial clustering of pancreatic cancer. Recent epidemiological studies suggest that about 7% of pancreatic cancer patients have a positive family history of the disease. For most cases, however, no hereditary basis for pancreatic cancer has been identified. 83. Which of the following surgical procedures has the lowest incidence of recurrent jaundice when used in the context of unresectable carcinoma of the head of the pancreas? a. Choledochoduodenostomy b. Cholecystojejunostomy c. Cholecystoduodenostomy d. Choledochojejunostomy Answer: d When jaundiced patients undergo exploration in the hopes of resection but unresectable disease is found, biliary bypass should be performed. The jejunum is typically chosen as a conduit in preference to the duodenum because duodenal obstruction may occur as the tumor becomes more advanced. There has been much discussion over the use of the bile duct or the gallbladder for biliary decompression. Operative mortality and mean survival (about 6 months) do not differ between patients with cholecystojejunostomy and choledochojejunostomy. Recurrent jaundice is more common after cholecystojejunostomy. Because recurrent jaundice constitutes a failure of palliation, the use of the common duct for biliary bypass is preferable in most patients. There are circumstances, however, in which it may be more appropriate to use the gallbladder. Such instances include patients with poor performance status, cases in which the tumor is bulky and invades the porta hepatis, or when periductal varices have developed as a result of portal vein thrombosis. The suitability of the gallbladder as a biliary conduit must be proven intraoperatively. If, on aspiration, the gallbladder contains colorless fluid, the cystic duct may be assumed to be obstructed, and the gallbladder should be removed and not used for bypass. If there is green bile in the gallbladder, patency of the cystic duct should be proved by cholangiography before a bypass is performed. 84. A 45-year-old woman is evaluated for epigastric and back pain. Physical examination is normal. Computed tomography of the abdomen reveals a 8 cm cystic lesion in the region of the tail of the pancreas. The cyst demonstrates internal septations and papillary projections from its walls. Which of the following diagnoses is most likely in this patient? a. Pancreatic lymphoma b. Retroperitoneal liposarcoma c. Pancreatic pseudocyst d. Pancreatic mucinous cystadenoma Answer: d Mucinous cystic neoplasms account for about 2% of pancreatic exocrine tumors. Most patients with mucinous cystic tumors present with abdominal pain or an abdominal mass. There may be associated weight loss, steatorrhea, or diabetes. The diagnosis is best made by CT scanning and ultrasonography, which demonstrate a mass containing fluid-filled structures and internal septations. Occasionally, it is possible to see the papillary tumor excrescences on the cyst walls. The tumor occurs six times as often in females as in males. About 80% of the tumors are located in the body and tail of the pancreas. They present as large (average, 10 cm), soft, and somewhat irregular tumors. Microscopically, the cysts are lined by columnar epithelium which contains mucin. Although most of the cells may appear benign histologically, most tumors larger than 3 cm contain areas of premalignant or malignant change and all mucinous cystic tumors should be considered to have malignant potential. 85. A 45-year-old woman develops upper gastrointestinal hemorrhage. Evaluation by upper endoscopy reveals three ulcers in the second portion of the duodenum. Bleeding is controlled using an endoscopic heat probe. Further investigation reveals a serum gastrin value of 240 pg/mL. Which of the following would support the presumptive diagnosis of gastrinoma? a. An increase of 320 pg/mL in serum gastrin upon intravenous infusion of secretin b. Gastric acid analysis demonstrating fasting acid secretion of 3 mEq/h c. Enlarged gastric rugae on upper gastrointestinal contrast study d. An increase of 150 pg/mL in serum gastrin upon intravenous infusion of cholecystokinin Answer: a, c The indications for the measurement of gastrin include the presence of peptic ulcer disease, patients with prolonged undiagnosed diarrhea, patients within MEN-1 families and patients with prominent gastric rugal folds on upper GI series. In most patients with gastrinoma, the fasting serum gastrin level is elevated above 200 pg/ml. Gastrin values over 1000 pg/ml are virtually diagnostic of gastrinoma. However, fasting hypergastrinemia alone is not sufficient for the diagnosis of gastrinoma. Gastric acid analysis is an important test in the evaluation of patients with suspected gastrinoma, as it can differentiate between ulcerogenic causes of hypergastrinemia and nonulcerogenic causes of hypergastrinemia. The diagnosis of gastrinoma is supported by a basal acid output above 15 mEq/hour in nonoperated patients. Following documentation that hypergastrinemia is associated with excessive acid secretion, provocative testing using secretin should be performed to differentiate between gastrinoma, antral G cell hyperplasia/hyperfunction, and the other causes of ulcerogenic hypergastrinemia. The secretin stimulation test is carried out in the fasting state by obtaining peripheral serum samples for gastrin in the basal period, administering secretin (2 units/kg body weight) as an intravenous bolus, and obtaining serum samples for gastrin at five minute intervals for 30 minutes. An increase in the gastrin level of more than 200 pg/ml above the basal level is supportive of the diagnosis of gastrinoma. 86. For the patient in the preceding question, an insulin/glucose ratio of 0.5 was documented at 28 hours of fasting. Symptoms of mental obtundation developed concurrently and were reversed by oral glucose administration. Endoscopic ultrasonography demonstrated a 1.2 cm mass in the head of the pancreas. Appropriate management consists of which of the following? a. Surgical enucleation of the tumor b. Total pancreatectomy c. Long-term octreotide administration d. Primary radiotherapy Answer: a The treatment of insulinoma is surgical in nearly all cases. Insulinomas are found evenly distributed within the pancreas, with approximately one-third being located in the head and uncinate process, one-third in the body of the gland, and one-third in the tail of the gland. Ninety percent of patients will be found to have benign solitary adenomas amenable to surgical cure. Small benign insulinomas not in close proximity to the main pancreatic duct may be removed by enucleation, independent of their location within the gland. In the body and tail of the Pancreas: insulinomas greater than 2 cm in diameter, and those in close proximity to the pancreatic duct are most commonly excised by distal pancreatectomy. Large insulinomas deep in the head or uncinate process of the pancreas may not be amenable to local excision, and may require pancreaticoduodenectomy. 87. A 35-year-old woman is evaluated for seizure disorder, mental obtundation, and personality change. Physical examination is normal. Fasting serum glucose is 44 mg/dL. Other serum values are normal. Subsequent investigations should include which of the following? a. Oral glucose tolerance test b. Determination of fasting insulin/glucose ratios c. Assay of serum C-peptide levels d. Determination of serum prolactin levels Answer: b, c A common mistake made in the evaluation of a patient with suspected insulinoma is to commence the evaluation with an oral glucose tolerance test. Instead, insulinoma is most reliably diagnosed using the technique of a monitored fast. During a monitored fast, blood for glucose and insulin determinations is sampled every four to six hours, and at the time of symptom occurrence. Hypoglycemic symptoms typically occur when glucose levels are less than 50 mg/dl, with concurrent serum insulin levels often being greater than 25 µU/ml. Additional support for the diagnosis of insulinoma comes from the calculation of the insulin to glucose ratio (I:G ratio) at different time points during the monitored fast. Normal individuals will have I:G ratios less than 0.3, while patients with insulinoma typically demonstrate I:G ratios greater than 0.4 after a prolonged fast. Other measurable beta cell products synthesized in excess in patients with insulinoma include C peptide and proinsulin. Elevated levels of C peptide and proinsulin are typically found in the peripheral blood in patients with insulinoma. The possibility of surreptitious insulin or oral hypoglycemic agent administration should be considered in all patients with suspected insulinoma. C peptide and proinsulin levels will not be elevated in patients self-administering insulin. Additionally, patients self-administering either bovine or porcine insulin may demonstrate anti-insulin antibodies in circulating blood. 88. The most common location(s) for development of gastrinoma is/are which of the following? a. Pancreas to the right of the superior mesenteric vein b. Pancreatic body and tail c. Gastric antrum d. Duodenum Answer: a, d The majority of gastrinomas have been identified to the right of the superior mesenteric vessels within the head of the pancreas or the duodenum. Intraoperative ultrasonography should be available to assist in tumor localization. In addition, intraoperative upper endoscopy may be helpful by allowing transillumination of the duodenal wall and identification of small duodenal gastrinomas. At exploration, any suspicious peripancreatic lymph nodes are excised and submitted for frozen section. Primary tumors located within the substance of the pancreas that are small (< 2 cm) and well encapsulated may be carefully enucleated. Pancreatic tumors without defined capsules or situated deep in the pancreatic parenchyma may require partial pancreatic resection. In the absence of an identifiable pancreatic or duodenal tumor, a longitudinal duodenotomy may be performed at the level of the second portion of the duodenum to allow for eversion of the duodenum in a search for duodenal microgastrinomas. Primary gastrinomas identified within the duodenal wall are resected locally, with primary closure of the duodenal defect. 89. Neoplastic hypersecretion of the hormone vasoactive intestinal peptide is associated with which of the following features? a. Hypokalemia, hypochlorhydria, diarrhea b. Hyperglycemia, necrolytic rash, hypoaminoacidemia c. Constipation, gallstones, hyperglycemia d. Hyperkalemia, necrolytic rash, diarrhea Answer: a Patients characteristically present with intermittent severe diarrhea, typically of a watery nature, averaging 5 liters/day. Malabsorption and steatorrhea are not common. Hypokalemia results from the fecal loss of large amounts of potassium (up to 400 meq/day), and low serum potassium levels may be associated with muscular weakness, lethargy, and nausea. Most patients are hypochlorhydric or achlorhydric. Half of the patients have some degree of hyperglycemia and hypercalcemia, while cutaneous flushing can be observed in a minority of patients. The diagnosis of VIPoma is typically made after excluding other more common causes of diarrhea. The active agent in the VIPoma syndrome is usually vasoactive intestinal polypeptide (VIP), with a minority of patients having elevations of other candidate mediators such as peptide histidine-isoleucine (PHI) or prostaglandins. 90. A patient with biochemically confirmed gastrinoma undergoes computed tomography for tumor localization. CT reveals a 2 cm mass in the head of the pancreas and multiple nodules within right and left lobes of the liver. Appropriate management includes which of the following? a. Omeprazole administration b. Radiotherapy c. Pancreaticoduodenectomy d. Proximal gastric vagotomy Answer: a Gastrinoma patients whose localization and staging studies are indicative of unresectable hepatic metastases should undergo percutaneous or laparoscopically-directed liver biopsy for histologic verification. If unresectable gastrinoma is confirmed, then open surgical exploration is not performed and the patient is maintained on long-term omeprazole therapy. Virtually all patients can be rendered achlorhydric with appropriate dose adjustment of omeprazole. Noncompliant patients who refuse to take appropriate doses of omeprazole and who develop complications related to their ulcer diathesis may require total gastrectomy for management. Total gastrectomy removes the end organ (parietal cell mass) and was once the procedure of choice for gastrinoma. Today its use in gastrinoma patients has markedly declined. 91. The following statement(s) is/are true concerning the widely accepted French or Couinaud’s nomenclature for liver anatomy. a. The liver is divided into eight discrete segments based on portal pedicle branches and hepatic venous drainage b. This anatomy is particularly useful in allowing less than lobar segmental anatomical resections that minimize blood loss and loss of hepatic reserve c. Enumeration of the system begins from right to left d. Segments II and III are synonymous with the left lateral segment based on English nomenclature Answer: a, b, d In the now widely accepted French (Couinaud’s) nomenclature, the liver can be divided into eight discrete segments based on portal pedicle branches and hepatic venous drainage. Enumeration of the segments begins left to right, beginning with segment I, the caudate lobe. The left lateral sector consists of a superior segment II and an inferior segment III and is synonymous with the left lateral segment in older terminology. The major advantage to this detailed segmental anatomy, which is based on discrete portal pedicle branches, is to accurately locate individual lesions in the hepatic substance by preoperative imaging and intraoperative ultrasound and to allow the possibility of less than lobar segmental anatomical resections that minimize blood loss and functional loss of hepatic reserve. 92. In the patient described above, which of the following are important operative steps in the performance of a right hepatic lobectomy? a. The use of an ultrasonic dissector is essential for division of the hepatic parenchyma b. If temporary portal inflow occlusion is used (Pringle maneuver), it is not necessary to reestablish blood flow during the course of the parenchymal division c. The greater omentum may be used to buttress the transected liver edge d. Control of the main right hepatic vein should eliminate all forms of venous drainage Answer: c The steps involved in a right hepatic lobectomy involve adherence to the tenet of optimal operative exposure and control of vascular inflow and outflow. In select circumstances, control of the vena cava may be desired. Either the individual portal structures can be identified and ligated early in the course of the procedure, or simply the entire portal triad can be circled with an umbilical tape tourniquet in preparation for the Pringle maneuver. If temporary portal inflow occlusion is used, intermittent 10 to 20 minute intervals of clamping with 3 to 5 minutes to reestablish blood flow is recommended. The division of the hepatic parenchyma begins with scoring of Glisson’s capsule with cautery or knife and proceeds with division of the hepatic surface using either blunt dissection by finger fracture, the blunt edge of an instrument or suction tip, or using an ultrasonic dissector. Individual vessels and bile ducts are cauterized, sutured, or clipped in rapid succession from anterior to posterior. The hepatic veins are encountered in the hepatic substance near the vena cava and are carefully clamped and suture ligated to complete the resection. In addition, there are also several posterior accessory veins (up to 10 in number) which drain the medial aspect of the right lobe and empty directly into the right anterior surface of the IVC. 93. Intraoperative ultrasound is now commonly used by the hepatic surgeon. Which of the following statement(s) is/are true concerning intraoperative ultrasound and hepatic surgery? a. An intraoperative ultrasound offers no advantage to conventional transcorporial ultrasound in detection of hepatic lesions b. Portal structures can be differentiated from hepatic veins by the extension of Glisson’s capsule surrounding these structures c. It is difficult on ultrasound to differentiate a vascular structure from a mass d. The short hepatic veins are difficult to detect with intraoperative ultrasound Answer: b Over the past 10 years, detailed anatomic description of the hepatic veins, portal pedicles, and the inferior vena cava have been possible through the use of intraoperative ultrasound. Cooperation between radiologists and hepatic surgeon with the use of intraoperative ultrasound has allowed the identification of lesions during surgery that were not visible by conventional transcorporial ultrasound or CT scanning. Beginning superiorly at the inferior vena cava, the confluence and course of each of the hepatic veins can easily be determined. More inferiorly, the main right and left portal pedicles can be seen coursing transversely in the transverse scissura. Portal structures can easily be differentiated from hepatic veins by the hyperechoic extensions of Glisson’s capsule which surround these structures. When a circular structure is encountered, a mass or metastasis may be suspected. Scanning away from the mass may reveal a tubulovascular shape which has been imaged and cross sectioned. Flattening of the circular mass by external compression with the ultrasound probe will also differentiate a vascular structure from a solid mass. 94. The following statement(s) is/are true concerning hepatic anatomical nomenclature. a. In the traditional English system, the right lobe is divided into anterior and posterior segments by an intersegmental line with no topographic landmarks or interparenchymal septi b. The caudate lobe in the French or Couinaud’s nomenclature is referred to as segment I c. The right lobe of the liver by English nomenclature is subdivided in the French system into segments V–VIII d. In the English system, the left lobe of the liver is divided into the medial segment and lateral segment by the falciform ligament Answer: a, b, c, d Until recently, anatomic descriptions in the English literature began with the major divisions of liver into a right and left lobe separated by a vertical line drawn from the gallbladder fossa to the inferior vena cava. The left lobe is further divided by the falciform ligament into a medial segment and lateral segment. The right lobe is further divided into an anterior and posterior segment by an intersegmental line which has no reliable topographical landmarks and no interparenchymal septi to allow easy identification. The French nomenclature also known as Couinaud’s nomenclature enumerates the segments of the liver beginning with segment I or the caudate lobe. Segments II, III, and IV make up most of the “English” left lobe, while segments V through VIII represent the English nomenclature right lobe. 95. A 57-year-old man with a history of Duke’s C colon cancer is being evaluated for a rising CEA. Which of the following statement(s) is/are correct concerning the use of CT scanning for this indication? a. Conventional CT scanning will detect lesions well below 1 cm in size b. CT arterio-portography involves immediate CT scanning after direct injection into both the common hepatic artery and superior mesenteric artery c. A double helical (spiral) CT scan may eliminate the need for invasive angiography d. Magnetic resonance imaging of the liver will add little to the workup of this patient Answer: b, c, d CT scanning has been used increasingly to screen for hepatic and other intra-abdominal or retroperitoneal lesions. Conventional CT scanning includes 0.5–1 cm axial images of the liver after oral administration of barium and bolus injection of intravenous contrast. Although resolution has improved, hepatic lesions below 1 cm in size or lesions that are isodense with hepatic parenchyma may be missed. Resolution of hepatic lesions has been greatly enhanced by the combination of visceral angiography and CT scanning, known as CT arterio-portography (CTAP). Immediate CT scanning after injection of contrast directly into the common hepatic artery may identify small hepatic lesions which usually show increased density relative to the surrounding hepatic parenchyma. CT arterio-portography also includes direct injection of contrast into the splenic or superior mesenteric arteries, with CT imaging during the portal venous phase of this injection. Hepatic lesions supplied by the hepatic artery thus appear as discrete hypodense lesions surrounded by normal hepatic parenchyma enhanced by portal venous contrast. Recently, double helical (spiral) CT scanning has become available and shows considerable promise to complement or replace CTAP for preoperative imaging. This technique allows total hepatic imaging in both the arterial and arterial/venous phases after a single rapid bolus injection of intravenous contrast during a single breath hold by the patient. It is possible to visualize the portal structures and hepatic veins on a single scan and give a high resolution of small hepatic lesions. In addition, three-dimensional reconstructions can be created to further delineate hepatic parenchyma and demonstrate a CT constructed hepatic arteriogram. This technique may completely replace the need for invasive arteriography to characterize the blood supply to the liver prior to hepatic resection or after hepatic transplantation. Magnetic resonance imaging of the liver has results similar to CT scanning, but to date has not demonstrated improvements sufficient to justify the increased cost associated with the technique. 96. A solitary 6 cm lesion is identified in the right hepatic lobe in the patient described above. Which of the following statement(s) is/are true concerning the initial operative management? a. To facilitate mobilization and assessment with intraoperative ultrasound, complete mobilization including dividing the left and right triangular ligaments would be necessary b. In dividing the right triangular ligament, care must be taken to avoid injury to accessory right hepatic veins draining directly into the vena cava c. Unless a considerable length of hepatic vein is found outside the hepatic parenchyma, early hepatic vein ligation should be avoided d. Ligation of the portal arterial structures is always necessary before proceeding with hepatic lobectomy Answer: a, b, c For major hepatic resections and for complete intraoperative ultrasound, complete mobilization of the liver will be required. After detachment of the hepatic flexure of the colon and division of the falciform ligament, both the left and right triangular ligaments must be sharply taken down to fully mobilize the liver. During division of the right triangular ligament, care must be taken to avoid injury to the right diaphragm, the right adrenal gland and adrenal vein, the right phrenic vein, and several moderate-size accessory right hepatic veins draining directly into the vena cava. After mobilization, digital and bimanual palpation is performed and intraoperative ultrasound may be performed. Dissection of the porta hepatis is performed by many hepatic surgeons to identify the main bifurcations of the hepatic artery, bile duct, and portal vein. This allows individual ligation of unilateral branches of each of these structures during hepatic lobectomy but prior to parenchymal dissection. An alternative approach has been recently described where the main portal structures are left undisturbed and branches to a given lobe are ligated during parenchymal transection. Hemorrhage can be minimized by intermittent portal inflow occlusion by clamping or compression of the portal triad (Pringle maneuver). There has been considerable debate over early versus late isolation and ligation of a given hepatic vein during lobectomy since the extraparenchymal component of the hepatic vein may be quite short or absent. Since hemorrhage in this location may be difficult to control, a safe strategy is to always avoid early isolation of a given hepatic vein or to attempt isolation only when a considerable length of vein is found on mobilization of the respective triangular ligament. 97. Which of the following statement(s) is/are true concerning the arterial venous anatomy of the liver? a. Most commonly, the right, left, and middle hepatic veins join the inferior vena cava as a separate trunk b. Most frequently, the entire length of each hepatic vein is within the parenchyma of the liver c. A replaced right hepatic artery may be placed in jeopardy during performance of a pancreaticoduodenectomy d. There is little collateral arterial circulation between the right and left hepatic lobes Answer: b, c There are three major hepatic veins which carry blood from the central veins of the hepatic substance to the inferior vena cava (IVC). In two-thirds of patients, there is a single large right hepatic vein which joins the right anterior wall of the IVC and a middle and a left hepatic vein which converge one-to-two cm from the IVC and enter the left anterior wall of the IVC as a single vessel. In one-third of patients, each major hepatic vein joins at the same horizontal level of the IVC as a separate trunk. In some patients, there is a short but definable extraparenchymal segment of one or more of the hepatic veins at the confluence with the IVC. More frequently, the entire length of the hepatic veins is intraparenchymal, which may preclude early, safe hepatic venous isolation during hepatic resection. There is considerable variability in the origin and course of the right and left hepatic arteries. The most common finding (55% of patients) is a transverse common hepatic artery from the celiac trunk which gives off the gastroduodenal, right gastric, and supraduodenal arteries and courses obliquely in the left anterior aspect of the hepatoduodenal ligament as a proper hepatic artery. After giving off the cystic artery to the gallbladder, there is then a fairly low trifurcation into a single right, middle, and left hepatic arteries. Knowledge of the most common variations is extremely importance since inadvertent division may occur during gastric, pancreatic, and hepatobiliary procedures. There may be a replaced or accessory left hepatic artery which arises from the left gastric artery and courses transversely in the lesser omentum. With nearly equal frequency, there is a replaced or accessory right hepatic artery from the superior mesenteric artery near its origin which courses posterior or through the head of the pancreas obliquely along the right posterior border of the hepatoduodenal ligament. Although original anatomic descriptions deny the existence of collateral vessels to the opposite hepatic lobe, image perfusion studies after ligation of main or replaced hepatic arteries have clearly demonstrated the presence of collateral flow to the deprived lobe. 98. The liver plays a vital role in carbohydrate metabolism and regulation of blood glucose. The following statement(s) is/are true concerning carbohydrate metabolism by the liver. a. Glycogen, a complex polymer of glucose, is synthesized by the hepatocyte in a remarkably energy efficient process b. Glucagon stimulates glycogenesis c. Glycolysis, the process by which glucose is converted to two molecules of pyruvate, occurs in the liver mitochondria d. If glycogen stores become depleted, the liver is capable of synthesizing new glucose by the process of gluconeogenesis, which is stimulated by insulin Answer: a Serum glucose is tightly regulated by the liver despite wide fluctuations in dietary ingestion. The liver can take up as much as 100 g/day of glucose and convert it to glycogen by the process of glycogenesis. The liver can also release glucose into the blood by glycogenolysis, the breakdown of glycogen, or by gluconeogenesis, the formation of new glucose from substrates such as alanine, lactate, glycerol or dietary amino acids. Hormones play a key role in hepatic regulation of glucose metabolism. Insulin, for example, stimulates glycogenesis, and glucagon stimulates glycogenolysis and gluconeogenesis. Gluconeogenesis is also enhanced by fasting, critical illness and periods of anaerobic metabolism. Glycogen is a complex polymer of glucose. Liver cells can store up to 8% of their weight as glycogen. The first step in glycogen storage is the transport of glucose through the hepatocyte plasma membrane. About 90% of portal venous glucose is removed from the blood by liver cells through carrier-facilitated diffusion. The rate of glucose transport is enhanced by insulin. Once in the hepatocyte, glucose and ATP are converted by the enzyme glucokinase to glucose-6-phosphate (G6P), the first intermediate in the synthesis of glycogen. Because complete oxidation of one molecule of G6P generates 37 molecules of ATP, and storage only uses one molecule of ATP, the overall efficiency of glucose storage in glycogen is a remarkable 97%. Glycolysis is the pathway by which glucose is converted to two molecules of pyruvate and occurs in the cytoplasm in contrast to the citric acid cycle which occurs in the mitochondria. 99. Transport of substances from the blood into the hepatocyte occurs through the sinusoidal membrane. The following statement(s) is/are true concerning this plasma membrane. a. The high lipid content of this phospholipid bilayer allows lipid-soluble molecules to enter the cell by simple diffusion b. Carrier proteins within the phospholipid bilayer bind to a solute in blood and by conformational change allow it to be transported into the cell c. Large glycoprotein molecules of the sinusoidal membrane known as receptors always transport the binding ligand into the cell d. The transmission of a signal to the interior of the cell by receptor-ligand binding which generates intracellular second messengers is known as signal transduction Answer: a, b, d The hepatocyte plasma membrane consists of a phospholipid bilayer in which hydrophobic fatty acid tails are oriented to the interior membrane and hydrophilic phospholipid head groups are oriented to the exterior (sinusoidal or cytoplasmic) membrane. Within this phospholipid bilayer are proteins which serve either structural functions or metabolic functions. The hepatocyte sinusoidal plasma membrane is heavily studded with microvilli to increase the absorptive area in contact with sinusoidal blood. The cell membrane, by virtue of its high lipid content, allows lipid-soluble molecules to enter the cell by simple diffusion. Polar molecules must enter cells via membrane transport proteins. Channel proteins allow molecules to diffuse simply into cells without binding, whereas carrier proteins first bind the solute and, by conformational change, allow it to be transported into the cell. The glucose carrier in hepatocytes is an example of carrier-facilitated diffusion. The sinusoidal membrane is studded with receptors, which are large glycoprotein molecules that span the plasma membrane lipid bilayer. A ligand-binding site of this receptor molecule projects into the space of Disse. When appropriate ligand-receptor binding occurs, the entire ligand may be internalized for intracellular degradation or biliary transport, or the ligand may transmit a signal to the interior of the hepatocyte by a number of intracellular second messenger systems, a process known as signal transduction. Such second messengers include cAMP, inositol triphosphate, and diacylglycerol. Each of these structurally simple chemicals can amplify cell membrane events and bring about major changes in cellular physiology. 100. The liver is an important site of protein metabolism. Which of the following statement(s) is/are true concerning protein metabolism by the liver? a. Amino acids are taken up by hepatocytes by active transport mechanisms and are generally stored long-term for later synthetic activity b. Under certain conditions the amine group is removed from the amino acids in the liver and the carbon chain used for carbohydrate, lipid, or nonessential amino acid synthesis c. The most important route of detoxification of ammonia formed as the result of deamination of amino acids is via excretion of ammonia into the urine d. Proteins synthesized by the liver include albumin, transferrin, fibrinogen, and apolipoproteins e. Albumin is a sensitive indicator of hepatic synthetic function Answer: b, d Essentially all of the end products of dietary protein digestion are amino acids, which are absorbed by the enterocytes into the portal circulation in ionized states. Amino acids are taken up by hepatocytes by one of several active transport mechanisms. Amino acids are not stored in the liver but are rapidly used in the production of plasma proteins, purines, heme proteins, and hormones. Under certain conditions, the amine group is removed from the amino acids, and the carbon chain is used for carbohydrate, lipid, or nonessential amino acid synthesis. The ammonia formed as the result of deamination of amino acids is detoxified by one of two routes. The most important pathway involves conversion of ammonia to urea by enzymes of the Krebs-Henseleit cycle, found only in the liver. A second route of ammonia metabolism involves deamination of L-glutamine by the kidney, with excretion of ammonia into the urine. Essentially all albumin, fibrinogen, and apolipoproteins are derived from the liver which can add up to 50 grams of protein to the plasma per day. Of total hepatic protein synthesis, 75% is destined for export in plasma. Albumin, an important plasma protein synthesized in the liver, has a long half-life in plasma of about 19 days. This long half-life makes albumin an insensitive indicator of hepatic synthetic function. 101. The following statement(s) is/are true concerning hepatic blood flow. a. Although constituting only 2.5% of total body weight, the liver receives 25% of the cardiac output b. Hepatic blood flow is equally derived from the portal vein and hepatic artery c. The liver serves as a physiologic blood reservoir either releasing blood back into the systemic circulation at times of acute blood loss or in situations of volume overload serving as a site of extra blood storage d. An important function of the liver is to filter particulate debris which is performed by phagocytic Kupffer cells which line the hepatic sinusoidal endothelium Answer: a, c, d The liver constitutes about 2.5% of the total body weight but receives 25% of the cardiac output. Total hepatic blood flow is 100 to 130 ml/min/kg. About two-thirds of total hepatic blood flow is derived from the portal vein and one-third from the hepatic artery. The liver also serves as a physiologic blood reservoir. About 25% to 30% of the liver volume is accounted for by blood, and in cases of acute blood loss up to 30%, or as much as 300 ml of the hepatic blood volume can be released into the systemic circulation without adverse effects on liver function. Conversely, in the case of right heart failure or other causes of systemic volume overload, as much as one liter of extra blood can be stored in the liver before passive congestion and liver injury occur. The hepatic sinusoids are lined by an endothelium punctuated with pores that allow proteins and other particles to diffuse out of the vascular tree and into proximity with hepatocytes. This extreme permeability of the liver allows rapid exchange of a diverse number of nutrients, hormones and environmental agents between the blood and the hepatocyte. The liver also acts as a filter for particulate debris, which enters the portal circulation through intestinal capillaries. Particles such as bacteria are ingested by Kupffer cells by the process of phagocytosis. Kupffer cells line the hepatic sinusoidal endothelium where formed blood elements and matter may be in direct contact with these phagocytic cells. 102. The liver synthesizes key metabolites essential for the production of fuel substrates for other organs. These key metabolites include: a. Glucose-6-phosphate (G6P) b. Acetyl CoA c. Pyruvate d. Oxaloacetate Answer: a, b, c Hepatic processes in the liver are essential for the production of fuel substrates for other organs. The liver, by virtue of its terminal position in the portal system, is the organ that must regulate intestinally absorbed nutrients for tissue consumption or storage. The liver accomplishes its task by synthesizing three key metabolites: -glucose-6-phosphate, pyruvate and acetyl CoA. G6P can be stored as glycogen or converted into glucose, pyruvate, or ribose-5-phosphate (a nucleotide precursor). Pyruvate can be converted into lactate, alanine (and other amino acids), and acetyl CoA, or it can enter the tricarboxylic acid cycle. Acetyl CoA is converted to HMG-CoA (a cholesterol and ketone body precursor) or citrate (for fatty acid and triglyceride synthesis), or it is degraded to carbon dioxide and water for energy. 103. The following statement(s) concerning hepatic bile formation/secretion is/are true. a. The adult human liver secretes less than 1000 cc of bile daily b. Most bile is secreted by hepatocytes (canalicular bile) c. Primary bile acids include cholic acid, chenodeoxycholic acid, and deoxycholic acid d. The enterohepatic circulation is tremendously efficient in reabsorption of intestinal bile acids e. Bile acids are the primary determinant of bile flow Answer: b, d, e The adult human liver secretes about 1.5 liters of bile daily. Eighty percent of this volume is secreted by the hepatocytes (canalicular bile) and 20% is secreted by the bile duct epithelial cells (ductular bile). Solutes constitute about 3% of bile. The major solutes are conjugated bile acids, phosphatidyl choline, cholesterol, protein and bilirubin. Bile acids are the main determinant of bile production, and canalicular bile flow is traditionally divided into bile acid-dependent and bile acid-independent components. Primary bile acids are synthesized from cholesterol in the liver and in humans consist of cholic acid and chenodeoxycholic acid. Secondary bile acids are formed in the intestinal lumen by bacterial dehydroxylation and consist of deoxycholic acid and lithocholic acid derived from cholic acid and chenodeoxycholic acid, respectively. Essentially all primary and secondary bile acids are conjugated with the amino acids glycine or taurine. The human liver synthesizes 300 to 400 mg per day of bile acids from cholesterol, or about 10% of the total bile salt pool. Normally intestinal bile acids are efficiently (about 95%) taken up by the enterohepatic circulation. Luminal bile acids are transported by carrier proteins in the distal ileum and appear in the portal venous effluent. The hepatocyte extracts more than 95% of portal venous bile acids for resecretion into the bile. 104. The following statement(s) is/are true containing lipid metabolism in the liver. a. Hepatic mitochondrial hydrolysis of fatty acids is a tremendous source of ATP b. Significant hepatic storage of triglyceride or fatty infiltration can cause hepatic fibrosis or necrosis c. Approximately 90% of cholesterol synthesis occurs in the liver d. Most cells in the body are capable of phospholipid synthesis, therefore the liver plays a minimal role in this process Answer: a, c The liver has a number of important functions in the metabolism of lipids: 1) the synthesis of apolipoproteins, 2) the degradation of fatty acids into energy substrates, 3) the synthesis of triglycerides from carbohydrates and proteins, and 4) the synthesis of cholesterol and phospholipids from fatty acids. The mitochondrial hydrolysis of fatty acids is a source of large quantities of ATP. The conversion of stearic acid to CO2 and H2O, for instance, generates 136 ATP molecules and demonstrates the highly efficient storage of energy in fat. In times of unrestrained lipolysis, such as starvation, uncontrolled diabetes or other conditions of triglyceride mobilization from adipose tissue, the ability of the liver to perform beta-oxidation may be inadequate. Under these circumstances, significant hepatic storage of triglycerides or fatty infiltration of the liver may occur. Triglyceride storage by itself does not appear to be a cause of hepatic fibrosis or necrosis, but fatty infiltration may be a marker for derangement of normal processes by alcohol or drug toxicity, diabetes, chronic parenteral nutrition, or morbid obesity. Cholesterol is an important regulator of membrane fluidity and is a substrate for bile acid and steroid hormone synthesis. Cholesterol may be available by dietary intake or by de novo synthesis. In mammals, about 90% of new cholesterol is synthesized by the liver from its precursor, acetyl CoA. Dietary cholesterol intake suppresses endogenous synthesis by inhibiting the rate-limiting enzyme in cholesterol by a synthetic pathway, HMG-CoA reductase. There are three major classes of phospholipids synthesized by the liver: the lecithins, the cephalins, and the sphingomyelins. Although most cells in the body are capable of some phospholipid synthesis, the liver produces 90%. 105. Hepatic biotransformation is defined as the intracellular metabolism of endogenous and exogenous organic compounds. Which of the following is/are enzyme families responsible for hepatic bile transformation? a. Cytochromes P-450 b. UDB-glucuronyl transferases c. Glutathione-transferases d. Sulfotransferases Answer: a, b, c, d The liver contains enzyme systems that can expose functional groups such as hydroxyl ions and alter the size and solubility of a wide variety of organic and inorganic compounds by conjugation with small polar molecules. The general strategy of the liver is to convert hydrophobic, potentially toxic compounds into hydrophilic conjugates that can then be excreted into bile or urine. There are four general enzyme families responsible for hepatic bile transformation. The cytochromes P-450 catalyze reactions such as oxidation, hydroxylation, sulfoxide formation, oxidative deamination, dealcoholization and dehalogination. Such reactions allow further phase II conjugation with polar groups such as glucuronate, glutathione and sulfate. Glucuronidation is the conjugation of UDB-glucuronic acid to a wide variety of xenobiotics by either esther or ether linkages. The glutathione transferases and sulfotransferases play a role in conjugation of P-450 derivatives. However, the glucuronyl transferase system is the predominant mechanism. 106. The following statement(s) is/are true concerning the differential diagnosis between an amoebic and a pyogenic liver abscess. a. The clinical presentation is often clearly distinguishable b. A history of travel or origin from a high risk area might suggest an amebic liver abscess c. Routine liver chemistries frequently can distinguish pyogenic from amoebic liver abscess d. Serologic testing for the presence of antibody to entamoeba histolyctica is the only specific and sensitive way to confirm the diagnosis of amoebic liver abscess e. Distinguishing pyogenic from hepatic abscesses preoperatively is not important since surgical drainage is imperative for both Answer: b, d Distinguishing amoebic from pyogenic liver abscess can be a diagnostic challenge. It is of major importance, however, because effective medical therapy with metronidazole can obviate the need for either percutaneous or surgical drainage in most cases of amoebic abscess. The clinical presentation for both conditions with acute onset of fever, abdominal pain, and altered liver function tests are almost identical. Important features such as travel to or origin from a high risk area is particularly important for amebic liver abscess. Routine liver chemistries and radiographic studies can rarely distinguish between amoebic and pyogenic liver abscesses. Specific serologic tests for the presence of antibody to E. histolytica are specific and sensitive for amoebic hepatic abscess being positive in 95% of the cases, and therefore, are key in distinguishing the two infections. 107. A patient is found to develop evidence of hepatitis approximately eight weeks after receiving blood transfusions during a surgical procedure. Which of the following statement(s) is/are true? a. The virus responsible is most likely hepatitis C b. A chronic carrier state will ultimately develop in most patients c. There is no role for interferon in the treatment of chronic hepatitis C viral infection d. Chronic infection with hepatitis C is not associated with an increased risk of developing hepatocellular carcinoma Answer: a, b Hepatitis C virus is a virus that is responsible for more than 90% of post-transfusion hepatitis and most sporadic non-A, non-B hepatitis throughout the world. The most common identifiable sources of acquisition of hepatitis C virus are prior transfusion of blood or blood-derived products or a history of intravenous illicit drug use. The usual incubation period of post-transfusion hepatitis C viral infection is 5 to 10 weeks. An initial elevation of liver enzymes may be associated with little or no clinical disturbance. In some patients, acute hepatitis C viral infection does not progress to chronic infection, however, chronic hepatitis C viral infection develops in up to 70% of patients with post-transfusion hepatitis C infection with many progressing to cirrhosis. Hepatitis C does not appear to alter life expectancy at least in the first 15 years of infection. However, once cirrhosis and end stage liver disease develop, the clinical syndrome is indistinguishable from other forms of chronic liver disease with a predisposition to the development of hepatoma. Interferon alpha is the only FDA approved therapy for chronic hepatitis C viral infection. There is some evidence that early administration of interferon in acute hepatitis C viral infection may reduce the risk of progression to the chronic state. As yet, there is no evidence that interferon alters the natural history of chronic hepatitis C viral infection or changes the incidence. 108. A surgeon is suspected of having contacted hepatitis B virus via needle stick. Which of the following statement(s) is/are true concerning his diagnosis and outcome? a. Incubation of hepatitis B virus is about two weeks b. Jaundice is the first serologic indicator of hepatitis B infection c. The patient has about a 10% chance of developing a chronic carrier state d. All susceptible household or sexual contacts of the surgeon should receive hepatitis B viral vaccine e. The surgeon should receive hepatitis B immunoglobulin as soon as possible after the accidental needle stick Answer: c, d, e Hepatitis B viral infection is insidious. The incubation period of the virus is about eight weeks. The first serum indicator of infection by hepatitis B virus is detection of the serum hepatitis B surface antigen (HBsAg) which may proceed the onset of jaundice. In most cases, hepatitis B infection is self-limited and does not progress to chronic hepatitis. However, some 10% of patients with acute hepatitis B viral infection, whether it is clinical or subclinical, will develop a chronic carrier state. The carrier state is defined by the presence of HBsAg in serum for longer than six months. The best method of treatment of hepatitis B viral infection is primary prevention by vaccination. All susceptible household or sexual contacts of a person with a positive serum test for HBsAg should be advised to receive a full course of hepatitis B viral vaccine. Passive prophylaxis with hepatitis B immunoglobulin should be provided to any susceptible contact in whom there is recent potential parenteral exposure such as an accidental needle stick. 109. The following statement(s) is/are true concerning the diagnosis and treatment of hydatid cysts. a. Percutaneous aspiration is an important aspect of diagnosis and treatment of a hydatid cyst b. CT scan will oftentimes show the classic findings of a cystic liver lesion with a calcific rim c. At operation, care must be taken to protect the operative field from spillage of the cyst fluid d. The use of a scoleocide has become obsolete with current surgical techniques Answer: b, c Hydatid cysts are most commonly the result of infection with the tape worm, Echinococcus granulosis. Routine laboratory tests in patients with hydatid cysts are normal or nonspecifically abnormal. Although routine chest or abdominal radiographs may show a mass with a calcific rim, sonography and CT scan are the favored means of imaging hydatid cysts. The presence of calcifications and daughter cysts within the parent cyst suggests Echinococcus. Percutaneous needling of a hydatid cyst is unwise unless precautions against anaphylaxis are undertaken. A cyst’s fluid is often under pressure, and needling may precipitate rupture with the potential for anaphylaxis or intraperitoneal seating. The classic treatment of hydatid cysts is operative. The surgical aim is to remove the cyst or cysts without dissemination of the organism. At operation, the cyst is drained of fluid through a cannula after carefully protecting the operative field from fluid leakage. If the aspirate is clear a parasiticidal fluid (ethyl alcohol or 20% sterile saline) is injected into the cyst to kill any adherent scoleces. The cyst contents and the pericystic wall is then removed with careful surgical dissection. 110. Which of the following statement(s) is/are true concerning treatment of pyogenic liver abscess? a. Antibiotic therapy alone may be advisable in patients with multiple small abscesses b. Percutaneous drainage provides comparable results to surgical drainage in patients with unilocular large abscesses c. Sufficient antibiotic coverage for most hepatic abscesses includes coverage for gram-positive aerobic bacteria only d. In patients with a primary biliary origin for the hepatic abscess, treatment must also be addressed at underlying biliary pathology such as choledocholithiasis or biliary ductal obstruction Answer: a, b, d The preferred treatment of most patients with hepatic abscesses is broad-spectrum antibiotic coverage and drainage. A number of studies have demonstrated for most patients with large unilocular abscesses that percutaneous catheter drainage is as effective as surgical drainage. Bacteria that predominate in pyogenic liver abscesses are gram-negative aerobes, streptococcal species, and anaerobes. Therefore, broad-spectrum antibiotic coverage is necessary. Antibiotic coverage alone may be advisable in occasional patients who have multiple small abscesses not accessible to percutaneous or surgical drainage. Since many of these patients have an underlying biliary pathology as the source of the hepatic abscess, correcting this underlying pathology, for example, establishing biliary drainage surgically or nonoperatively is important. 111. Which of the following statement(s) is/are true concerning acute, fulminant hepatic failure? a. The most frequent cause of acute hepatic failure world-wide is hepatitis B infection b. Higher grades of encephalopathy are associated with a worse prognosis c. Hypoglycemia is a common complication of all liver diseases d. Liver transplantation would appear indicated in all patients with hepatic coma secondary to acute liver failure Answer: b The diagnosis of acute (fulminant) hepatic failure is based on the development of encephalopathy within eight weeks of the onset of symptoms. The overall prognosis is poor, but the hepatic lesions are potentially reversible, and recovery can lead to restoration of normal liver function. The most frequent cause of acute hepatic failure world-wide is non-A, non-B viral hepatitis. A variety of other viral agents and hepatotoxins can also cause this condition. No reliable criteria predict outcome and response to treatment. Higher grades of encephalopathy (depth of coma) on admission are associated with the worst prognosis. Management should include general supportive measures and specific treatment for hepatic encephalopathy, cerebral edema, electrolyte and metabolic disturbances, infection, and pain. Hypoglycemia is an unusual complication of most liver diseases except in patients with acute hepatic failure or hepatic neoplasms. The enormous reserve capacity of the liver accounts for the rarity of hypoglycemia except as a preterminal event. Bleeding is also a frequent cause of death in patients with acute hepatic failure secondary to depressed liver synthesis of clotting factors and qualitative or quantitative platelet disorders. The lack of a definitive medical treatment for acute hepatic failure makes liver transplantation seem attractive especially for patients with little or no chance of recovering normal liver function. Perhaps the most significant drawback to widespread acceptance of liver transplantation for acute hepatic failure is the lack of criteria reliability to predict which patients are likely to benefit from operation. Patients with mild to moderate degrees of coma are likely to recovery spontaneously without the need for liver transplantation while rapid deterioration and neurologic status to grade III or grade IV coma are associated in some centers with a mortality of 95%. 112. Which of the following statement(s) is/are true concerning the natural history and clinical features of alcoholic cirrhosis? a. In patients with compensated cirrhosis, the probability of survival at 10 years approaches 50% b. The development of clinical evidence of hepatic decompensation reduces five year survival to less than 20% c. Continued consumption of alcohol worsens prognosis d. The risk of death after variceal hemorrhage depends more on the severity of underlying liver disease than the type of therapy Answer: a, b, c, d Recent studies have analyzed the natural history of cirrhosis as a function of the degree of hepatic decompensation at the time of diagnosis. A high proportion of patients with compensated cirrhosis remain well for many years after diagnosis. In these studies the probability of remaining compensated 10 years after diagnosis was 42%, and survival probability of compensated patients was 47%. The prognosis worsened considerably once patients developed clinical evidence of hepatic decompensation (ascites, jaundice, encephalopathy, or gastrointestinal hemorrhage). Among these patients, the probability of five-year survival was only 16%. The risk of death from variceal hemorrhage depends much more of the severity of the underlying liver disease than on the type of therapy. It would also appear from natural history studies that continued alcohol consumption does affect survival. In one study, the overall five-year survival is 63% for abstainers versus 40.5% for those who continued to drink. Continued alcohol consumption may have less of an effect on survival than the intensity of alcohol consumption. Furthermore, the degree of hepatic compensation at the time of inclusion into the study may have also been an important factor. 113. Which of the following statement(s) is/are true concerning the morphologic and histologic findings of cirrhosis? a. Micronodular cirrhosis is a pattern typical of chronic alcoholic liver disease b. Mallory bodies and megamitochondria are typical findings of alcoholic cirrhosis c. Bile leaks caused by rupture of bile ducts with extravasation of bile into portal triads is a common finding in post-necrotic cirrhosis secondary to hepatitis d. Large regenerating nodules separated by coarse irregular scars in piecemeal parenchyma necrosis is common in liver disease secondary to chronic active hepatitis Answer: a, b, d Morphologic classification of cirrhosis includes micronodular, macronodular, and mixed forms. Micronodular cirrhosis is characterized by uniform nodules and scars. The nodules are usually less than 3 mm in diameter and are typically associated with Laennec or nutritional cirrhosis in alcoholics. Post-necrotic cirrhosis is characterized by large regenerating nodules separated by coarse irregular broad as well as thin scars. This pattern is frequently seen in patients with viral hepatitis. Biliary cirrhosis is characterized by a coarsely granular macronodular liver. This condition results from long-standing cholestasis secondary to obstruction of intrahepatic or extrahepatic bile ducts. The most distinctive feature of large duct obstruction is the presence of bile leaks caused by rupture of bile duct with extravasation of bile into portal triads. Portal cirrhosis, which is typically observed in alcoholics, can generally be distinguished histologically by the presence of several specific hepatocellular alterations such as Mallory bodies and megamitochondria. 114. The following statement(s) is/are true concerning the management of ascites associated with chronic liver disease. a. Spontaneous bacterial peritonitis is an insignificant complication b. Large volume paracentesis is unsafe due to excessive volume loss from the intervascular space c. Peritoneovenous shunting is a trivial surgical procedure with minimal perioperative morbidity and mortality d. Transjugular intrahepatic portosystemic shunts (TIPS) can effectively treat ascites in patients refractory to conventional medical therapy Answer: d The onset of ascites usually indicates the presence of advanced liver disease. Cirrhotic ascites is usually straw colored, clear, or greenish. Spontaneous bacterial peritonitis occurs as a complication of cirrhotic ascites in up to 10% of patients. Spontaneous bacterial peritonitis is defined as infected ascitic fluid without a demonstrable other site of infection. This is a serious complication with reported in-hospital mortality rates of 60% to 90%. The rational approach of therapy for ascites includes sodium and fluid restriction, the use of diuretics, and the use of therapeutic paracentesis. Several studies have shown that repeated paracentesis in stable cirrhotic patients may be safe and effective as medical therapy and shortens the length of hospitalization. Single, large volume paracentesis has been reported to be effective and safe. Up to 10 liters of ascites can be removed in one hour if salt-poor albumen is administered simultaneously. In a small percentage of patients, surgical implantation of a peritoneovenous shunt may be advisable. The principal indication for use of peritoneovenous shunt is to stabilize ascites that is refractory to conventional medical therapy and therapeutic paracentesis. Despite the simplistic nature of the device, postoperative mortality and morbidity rates of 20% to 60%, respectively have been reported. Precipitation of disseminated intravascular coagulopathy, variceal hemorrhage, or hepatic failure may complicate this procedure. Transintrahepatic portosystemic shunts (TIPS) have been demonstrated to control ascites in one study in over 90% of patients with ascites refractory to medical management. However, patients with poor hepatic reserve in this study all died if orthotopic liver transplantation was not performed. This data suggests that TIPS is effective for refractive ascites in patients with good to moderate hepatic reserve but poor risk cirrhotics require orthotopic liver transplantation to correct this problem. 115. Which of these statement(s) is/are true concerning the etiologic factors in the development of cirrhosis? a. Viral hepatitis of any type (A, B, or non-A, non-B) can all progress to cirrhosis b. Acetaminophen can cause acute liver failure and necrosis but will not lead to cirrhosis c. Alcohol exerts toxic effects on the liver via reactive intermediates such as acetaldehyde d. Long-standing congestive heart failure can lead to cirrhosis secondary to centrilobular congestion, hemorrhage, and necrosis Answer: c, d Liver cells are sensitive to a variety of physical, microbiologic, and chemical agents, all of which may produce cellular injury. The eventual development of cirrhosis is determined by the nature and severity of the cellular injury and the liver’s ability to regenerate. Most infectious hepatitides are viral in origin. There is no documentation that hepatitis A progresses to cirrhosis, however hepatitis B and non-A, non-B hepatitis do demonstrate a propensity to become chronic with the development of cirrhosis. Chemical hepatotoxicity include direct and indirect actions. Acetaminophen under normal circumstances is detoxified chiefly by conjugation with glucuronic acid or sulfate. Saturation of glucuronic pathway with large doses of acetaminophen results in progressive depletion of intracellular glutathione stores, accumulation of toxic intermediate, and eventual cell necrosis which may progress to either acute liver failure or chronic cirrhosis. Alcohol can affect liver cell function in a number of ways. Like many hepatotoxins, the toxic effects of alcohol are caused indirectly by reactive intermediates. Acetaldehyde is the principal reactive compound generated by alcohol metabolism. The hepatotoxicity of acetaldehyde is related to its binding two major constituents of cellular membranes altering membrane integrity and enzymatic function frequently to the detriment of the cell. Early morphologic changes of long-standing cardiac decompensation and right-sided heart failure are central lobular congestion, hemorrhage, and necrosis combined with phlebosclerosis of central veins and scars connecting centrizonal areas (cardiac cirrhosis). 116. Important spontaneous portosystemic collaterals which develop in the face of portal hypertension include: a. The hemorrhoidal veins b. Left renal vein c. The paraumbilical venous plexus d. The coronary, short gastric, and paraesophageal veins Answer: a, b, c, d If portal pressure is elevated, spontaneous portosystemic collaterals develop in an attempt to decompress the portal system. Such collaterals increase venous return to the heart and increase cardiac output. In humans, the most important collaterals develop as tributaries of the coronary, short gastric, and paraesophageal veins; intercostal, esophageal, and azygous veins; the superior, middle, and inferior hemorrhoidal veins; and the peraumbilical plexus. Retroperitoneal veins and veins draining to the left renal vein from the splenic, adrenal, and gonadal veins may also serve as sites of the development of venous collaterals. 117. Which of the following statement(s) is/are true concerning the pathophysiology of variceal hemorrhage? a. All patients with portal hypertension will develop esophageal varices b. All patients with esophageal varices eventually bleed c. Variceal size can predict the incidence of variceal hemorrhage d. Control of acid secretion by H2 blockade can decrease the incidence of rebleeding after esophageal hemorrhage e. None of the above Answer: e About two-thirds of patients with portal hypertension develop varices; of these only two-thirds subsequently experience variceal hemorrhage. A number of factors are important in the pathogenesis of variceal hemorrhage. These include portal pressure, intravariceal pressure, variceal size and structure, and other factors. Variceal size alone is not predictive of variceal hemorrhage. Evidence conflicts about whether erosive esophagitis is a cause of variceal rupture. Control of acid reflux by H2 blockade has not been shown to decrease the incidence of rebleeding after esophageal hemorrhage. 118. Which of the following statement(s) is/are true concerning the use of transjugular intrahepatic portosystemic shunts (TIPS) in the treatment of variceal bleeding? a. This procedure effectively creates an end-to-side portocaval shunt b. Procedure-related mortality is generally in excess of 20% c. TIPS has been used successfully as a pretransplant procedure to reduce portal pressure d. The placement of a TIPS is not associated with the development of encephalopathy Answer: c Transjugular intrahepatic portosystemic shunts (TIPS) refer to an implantable, expandable metal stent placed radiologically through the hepatic parenchyma to establish a track between branches of the hepatic and portal veins. TIPS results in similar hemodynamics as a side-to-side portal systemic shunt. There is firm clinical data that TIPS provides effective control of acute variceal hemorrhage and portal hypertension regardless of the etiology of the underlying liver disease or the degree of hepatic decompensation. TIPS has also been used for preoperative portal decompression to facilitate orthotopic liver transplantation. Pretransplant TIPS should reduce portal pressure thereby reducing operative time and blood loss. The major complications of TIPS include encephalopathy and stenosis or occlusion of this stent. Encephalopathy occurs in 10% to 20% of patients after TIPS. This complication appears to correlate with increasing age of the patient and increased shunt diameter and shunt flow. 119. Hepatic encephalopathy is a common systemic manifestation of chronic liver disease. Which of the following statement(s) is/are true concerning this condition? a. Blood ammonia levels correlate well with the stage of hepatic encephalopathy b. Alterations in central nervous system neurotransmitters such as the neurotransmitter g- aminobutyric acid (GABA) have been proposed in the pathogenesis of hepatic encephalopathy c. Lactulose can be used to decrease intestinal ammonia absorption d. Patients can be expected to have an increased sensitivity to benzodiazepines Answer: b, c, d Hepatic encephalopathy, a poorly explained neuropsychiatric syndrome, characterized by diverse neurologic abnormalities, is the pathologic evidence of nonspecific structural changes in neurons, and a variable prognosis. Several hypotheses to explain the pathogenesis of hepatic encephalopathy have been proposed. Ammonia has been widely implicated in the pathogenesis of hepatic encephalopathy despite conflicting evidence. Blood ammonia levels correlate poorly with the stage of encephalopathy, however, one of the mainstays of treatment is measures to decrease ammonia absorption from the gut including the oral administration of lactulose. Another hypothesis has implicated false neurotransmitters in the pathogenesis of hepatic encephalopathy. Evidence suggests that activation of the GABA system may be important in the pathogenesis of hepatic encephalopathy. The GABA receptor binds several classes of ligands including GABA and drugs such as benzodiazepines. For reasons that are unclear, hepatic failure appears to increase the brain density of GABA receptors. This observation may explain the increased sensitivity to benzodiazepines and other inhibitory neurotransmitters observed with patients with chronic liver disease. 120. Which of the following statement(s) is/are true concerning the management of gastroesophageal variceal hemorrhage? a. Vasopressin decreases portal pressure through the process of splanchnic vasoconstriction b. Somatostatin is as effective as vasopressin but without the cardiac side effects c. Balloon tamponade provides good long-term control of bleeding esophageal varices d. Endoscopic sclerotherapy is more effective than conservative medical therapy in the treatment of bleeding esophageal varices e. Sclerotherapy, although excellent for the control of bleeding short-term, does not prolong overall survival Answer: a, b, d A number of invasive and noninvasive therapies exist for the treatment of bleeding esophageal varices. Vasopressin is frequently used in the treatment of acute variceal hemorrhage and acts by decreasing portal venous pressure or flow through splanchnic vasoconstriction. Vasopressin alone has been reported to temporarily control variceal hemorrhage in 50% to 75% of patients, however, it can be associated with cardiac and peripheral extremity cutaneous ischemia. Somatostatin also acts as a vasoconstrictor to reduce splanchnic flow, with trials demonstrating similar efficacy in controlling acute hemorrhage when compared to vasopressin but without the cardiac side effects. Balloon tamponade is generally used for the temporary control of acute variceal hemorrhage unresponsive to vasopressin or sclerotherapy. Initial control of acute variceal hemorrhage occurs in about 80% of patients, but bleeding recurs promptly on deflation of the balloons in over 50%. Endoscopic sclerotherapy has become the primary treatment for bleeding esophageal varices. A number of clinical trials demonstrate that emergent sclerotherapy is able to halt variceal bleeding that fails to respond to more conservative measures, with no increase in frequency or severity of complications. The long-term survival of patients treated with sclerotherapy continues to be debated. Metaanalysis has been performed on the data from seven randomized clinical trials evaluating the effect of repeated sclerotherapy on long-term survival. This analysis demonstrates that sclerotherapy reduces the number of deaths by 25% therefore supporting the use of sclerotherapy as an effective means of prolonging survival in patients who have experienced variceal hemorrhage. 121. Which of the following statement(s) is/are true concerning the surgical management of bleeding esophageal varices. a. A side-to-side portacaval shunt may be associated with the development of hepatofugal blood flow b. Selective shunts preserve prograde (hepatopedal) blood flow while decompressing esophageal varices or reducing portal pressure c. The presence of intractable ascites is a contraindication to the Warren shunt d. If the patient is considered a liver transplant patient, an interposition mesocaval shunt is a suitable alternative Answer: a, b, c, d Although portosystemic shunts are the most effective therapy for preventing recurrent variceal hemorrhage, they are associated with the increased incidence of encephalopathy. A number of types of shunts have been described, primarily to avoid the consequences of complete diversion of portal blood flow or to simplify the operation. End-to-side portocaval shunts are hemodynamically unique in that all portal flow is diverted and the hepatic limb of the portal vein is ligated, thus preventing hepatofugal blood flow from the liver. With lateral side-to-side shunts, the hepatic limb of the portal vein remains patent. A greater compensatory increase in hepatic arterial flow occurs when the portal vein serves as an outflow track and the liver extracts oxygen and metabolites from the blood exiting through the patent limb of the portal vein (hepatofugal). The goal of selective shunts is to preserve prograde (hepatopedal) portal flow to the liver while selectively decompressing gastroesophageal varices (Warren shunt) or reducing portal pressures sufficiently to prevent variceal hemorrhage (small-diameter shunts). Most surgeons with a special interest in this field attempt to construct a selective shunt when the operation is elective, unless contraindications are present. The distal splenorenal shunt of Warren and the small-diameter interposition portocaval shunt of Sarfeh are the most common selective shunts used. The Warren shunt is an ascitogenic operation and therefore the presence of ascites that is difficult to control medically is a contraindication of this operation. An interposition mesocaval shunt is frequently preferred in emergent situations because the shunt is relatively safe to construct and promptly halts variceal hemorrhage. If future hepatic transplantation is contemplated, this shunt offers the advantages in that it does not involve dissection of the area of the hepatoduodenal ligament. 122. Which of the following statement(s) is/are true concerning the results of portosystemic shunting? a. When comparing shunts with nonshunting procedures, only minor differences in long-term survival are reported, but the mode of death usually changes b. Distal splenorenal shunts are associated with the development of less hepatic encephalopathy c. Survival statistics following distal splenorenal shunt in good risk patients (Child’s Class A) are in excess of 80% d. None of the above Answer: a, b, c Multiple randomized comparisons of distal splenorenal shunt with other types of portosystemic shunts have been reported. Operative mortality and long-term prognosis are similar, but patients undergoing distal splenorenal shunts develop less encephalopathy. Less encephalopathy is also seen with the selective Sarfeh shunt and after nonshunting procedures, such as sclerotherapy or devascularization. When comparing shunts with nonshunting procedures, only minor differences in long-term survival are reported, but the mode of death usually changes. With nonshunting procedures, a greater proportion of patients die of recurrent hemorrhage; after construction of a shunt, a greater proportion die of hepatic failure. Class A patients undergoing distal splenorenal shunts can be predicted to have an actuarial survival including operative mortality in excess of 80%. This is a substantial improvement over previous experience with portocaval shunts and is likely better than the results that can be obtained with hepatic transplantation. 123. Which of the following statement(s) is/are true concerning radiologic examinations used for the assessment of hepatic neoplasms? a. Magnetic resonance imaging is considered the test of choice for distinguishing hemangiomas from other mass lesions b. Dynamic CT scanning is the dominant imaging modality for routine screening and diagnosis c. Radionucleotide imaging plays an important role in modern screening and detection for liver lesions d. CT angio-portography (CTAP) is the gold standard for early detection of metastatic hepatic lesions Answer: a, b, d In the Western world, the test most commonly used for screening, detection, and diagnosis of hepatic neoplasms are dynamic bolus-enhanced CT scan, MRI, and ultrasound. Dynamic CT scanning remains the dominant imaging modality for routine screening and diagnosis because it is sensitive, widely available, provides helpful anatomic information and allows the evaluation of other intraabdominal structures in the detection of extrahepatic disease. The most sensitive test for the detection of liver masses is CTAP, however, like angiography is generally reserved for preoperative evaluation of resectability and not for screening. Hepatic MRI imaging is another useful tool in the detection and diagnosis of the liver lesions, with sensitivities equal to that of CT. MRI with T1 and T2-weighted images and gadilinium enhancement can aid in the differential diagnosis of a mass. For example, MRI is now considered the test of choice for distinguishing hemangiomas from other mass lesions. Radionucleotide imaging is less sensitive and specific than CT, MRI, or ultrasound, and has a high percentage of false-positive and false-negative results. Therefore, nuclear medicine scans play a limited role in modern screening and detection, but they can help differentiate discrete masses. 124. A 45-year-old woman undergoes an ultrasound because of vague right upper quadrant pain and epigastric fullness. A 7 cm cystic lesion is detected. Which of the following statement(s) is/are true concerning the patient’s diagnosis and management? a. Simple aspiration is indicated for treatment and diagnosis b. Bile stained fluid suggests underlying biliary pathology c. The cyst is likely lined by cuboidal epithelium d. Laparoscopic unroofing of the cyst can provide satisfactory treatment Answer: c, d A symptomatic solitary hepatic cyst may cause vague right upper quadrant discomfort or pain, a sensation of epigastric fullness or heaviness, and early satiety, however, most cases are asymptomatic. Complications are rare but include hemorrhage into the cyst, secondary bacterial infection, or obstructive jaundice from compression of extrahepatic ducts. In the absence of complications, laboratory abnormalities are uncommon. Cysts are somewhat more common in females, are more common in the right lobe and are often multilocular rather than unilocular. They are lined with cuboidal epithelium resembling bile duct epithelium and are filled with fluid that may be clear, mucoid, bloody, or bilious. If the patient has no symptoms and the cyst was discovered incidentally and there is no evidence of infection or malignancy, one may observe the patient. Neither percutaneous aspiration nor surgery is indicated. Cysts nearly always recur after simple aspiration. Treatment of symptomatic cysts is surgical. Indications for surgery include symptoms, rupture, hemorrhage, or infection. Asymptomatic, uninfected simple cysts are best treated by excision, if possible. Larger cysts may be inroofed with free peritoneal drainage unless there is a history of hemorrhage or evidence of biliary communication. A laparoscopic approach to the unroofing of cysts has recently been reported with excellent success. If this cyst communicates with the biliary system (grossly by cholangiography) the leak may be oversewn or the cyst drained by a Roux-en-Y cystojejunostomy. 125. Which of the following statement(s) is/are true concerning biopsy techniques for hepatic masses? a. A fine needle aspiration (FNA for cytology is contraindicated for patients with hypervascular masses) b. Percutaneous biopsy should be performed only if results may obviate the need for exploratory laparotomy c. Needle track seeding of tumor is not a risk associated with percutaneous biopsy d. Laparoscopy and biopsy play little role in the management of liver lesions Answer: b Biopsy of a liver mass can be done percutaneously (with or without CT or ultrasound guidance), laparoscopically, or at laparotomy. The biopsy may be done for cytology only (FNA) or for histology (larger-core biopsy). Guided FNA has an overall sensitivity of 77% to 94% and may allow a distinction between primary and secondary malignancy. The risks associated with needle biopsy include bleeding, infection, needle track seeding of tumor, and sampling error. Hypervascular masses, coagulopathy, and ascites are contraindications to percutaneous core biopsy, however, FNA biopsy is generally considered safe under these circumstances. In evaluation of any liver mass, percutaneous biopsy should be performed only if it can reasonably be expected to obviate the need for exploratory laparotomy. Biopsy of suspected primary metastatic malignancy with clinical indications of unresectability may spare the patient an unnecessary laparotomy. Laparoscopy with biopsy may also be used to evaluate liver masses and to possibly avoid laparotomy in patients considered to be borderline resectable. 126. A 55-year-old woman presents with vague right upper quadrant pain and a palpable liver. Laboratory tests are normal and a noncontrast CT scan (patient has a history of contrast allergy) reveals an 8 cm right hepatic mass. Which of the following statement(s) is/are correct concerning the patient’s diagnosis and management. a. A gadilinium-enhanced MRI would be indicated to define the extent of the lesion and confirm the diagnosis of hemangioma b. A fine needle aspiration should be performed regardless of radiographic workup c. Hepatic embolization is the treatment of choice d. The lesion should be resected because of concern for malignant degeneration Answer: a Cavernous hemangiomas of the liver are the most common benign hepatic tumor and are detected in some 2% to 7% of autopsies. Overall, hemangiomas are exceeded only by hepatic metastases as the most common hepatic tumor. Cavernous hemangiomas consist histologically of cystically dilated, endothelium-lined vascular spaces. They occur in all ages but are observed more commonly in females. Hemangiomas are not premalignant. Less than half of affected patients have symptoms. Those that have symptoms usually have large masses. Symptoms usually occur including vague right upper quadrant discomfort, pain, fullness, and early satiety. Physical examination may be notable for hepatomegaly, mass or bruit. There are no laboratory abnormalities in patients with hemangiomas. The most useful radiologic test for diagnosing hemangiomas are MRI, CT and tagged red blood cell scanning. These tests have largely replaced angiography. CT with vascular contrast often demonstrates a diagnostically characteristic enhancement pattern. Gadilinium-enhanced MRI has recently been shown to be sensitive and specific in the diagnosis of hemangioma and has better resolution than tagged red blood cell scans. FNA biopsy of suspected hemangiomas can be performed, however this procedure should be avoided if the diagnosis is secure using noninvasive procedures. Given the natural history of hemangiomas and its low risk of rupture, observation is indicated for asymptomatic patients, especially for lesions smaller than 4 cm. Surgical excision is the only consistently effective treatment for symptomatic masses and should be performed if the lesion is localized and accessible with an acceptable operative risk. Embolization is indicated only for unresectable lesions and is only modestly effective. 127. Which of the following statement(s) is/are true concerning the pathogenesis of hepatocelluar carcinoma? a. Hepatocellular carcinoma is associated with infection with hepatitis A, hepatitis B, and hepatitis C b. The risk of developing hepatocellular carcinoma related to hepatitis B viral infection is related to the chronicity of the infection c. Hepatocellular carcinoma only develops in cirrhosis associated with hepatitis d. Alfatoxin B1 is a potent carcinogen in the development of hepatocellular carcinoma Answer: b, d Hepatocellular carcinoma has three well-known epidemiologic associations: hepatitis B infection, cirrhosis, and various hepatotoxins, most notably aflatoxin B1 (a microtoxin from the fungus Aspergillus flavus). Hepatocellular carcinoma is strongly associated with hepatitis B viral infection but there is no link between the neoplasm and the hepatitis A viral infection. There does appear to be some link between hepatocellular carcinoma and hepatitis C virus. Cirrhosis, predominantly macronodular, is a frequent result of hepatitis B viral infection and is a risk factor for hepatocellular carcinoma. Nevertheless, the micronodular form, which is more common in early alcoholic cirrhosis, will progress to hepatocellular carcinoma in about 2% to 3% of cases. Thus, hepatocellular carcinoma can develop in patients with hepatitis B viral infection and cirrhosis, hepatitis B viral infection alone, and with cirrhosis from any cause alone. 128. A 38-year-old woman with a 17 year history of oral contraceptive use presents with right upper quadrant pain. A CT scan demonstrates a 4 cm lesion in the right lobe of the liver. Which of the following statement(s) is/are true concerning the patient’s diagnosis and management? a. The lesion is likely premalignant b. A 99mTc sulfur colloid scan will distinguish this benign lesion from a malignant hepatoma c. The lesion, although benign, may be associated with life-threatening hemorrhage d. The lesion would be expected to be hypervascular on angiographic study Answer: c, d Hepatic adenomas are clearly linked with the use of oral contraceptives. The likelihood of developing a hepatic adenoma appears to be related to the duration and dosage of estrogen, and is greater at ages above 30 years. The majority (75%) of these lesions occur in the right lobe of the liver. Although adenomas may be a symptomatic, most patients have symptoms, usually of abdominal pain (as many as 50%), and 10% to 33% of patients present with acute signs and symptoms secondary to bleeding or rupture with intraperitoneal hemorrhage. No radiologic test is specific for adenomas. CT is the most useful preliminary test and often reveals areas of hemorrhage and necrosis. Angiography may add to CT findings by demonstrating the hypervascular tumor with a peripheral blood supply. 99mTc sulfur colloid scans show a cold spot thereby distinguishing adenoma from focal nodular hyperplasia but not from other solid masses. For asymptomatic patients or patients with minimal symptoms, surgery is the treatment of choice given the tumors have the tendency to bleed and a small chance of coexisting malignancy. 129. Which of the following statement(s) is/are true concerning focal nodular hyperplasia (FNH)? a. The lesion predominantly affects young women b. The lesion is associated with the use of oral contraceptives and other estrogens c. Radionucleotide scanning can be useful in the specific diagnosis of FNH d. Excisional biopsy is indicated in almost all cases because of the risk of bleeding Answer: a, c Focal nodular hyperplasia (FNH) should not be confused with a hepatic adenoma. Although FNH predominantly affects young women, it is also found in men and children. Unlike adenomas, there is no clear relationship between oral contraceptives and the development of FNH. FNH is most commonly asymptomatic and does not have a propensity to bleed or undergo malignant change. Histologically, FNH contains normal-appearing hepatocytes, bile ducts, and Kupffer cells in distinction to adenomas. Radionucleotide imaging can be useful in diagnosing FNH because FNH is the only lesion that contains Kupffer cells and therefore appears isodense rather than a filling defect. Treatment of asymptomatic patients is conservative when the diagnosis is clear. If there is doubt regarding the diagnosis, then excisional biopsy is indicated for small, easily removable lesions. 130. Which of the following statement(s) is/are true concerning the prognosis of patients with hepatic metastases and colorectal carcinoma? a. Over half of these patients will survive one year without treatment b. Five year survivals following hepatic resection for an isolated metastasis is in excess of 25% c. Survival beyond five years after resection suggests a high probability of cure d. Survival rates are improved with a margin of resection greater than 1 cm e. The size of a liver metastasis is not a significant factor in predicting recurrence if adequate margins can be obtained Answer: b, c, d, e Studies consistently report five-year survival rates averaging 25% for hepatic resection for colorectal metastases. Those who survive beyond five years seem to do well with only an additional 5% dying of recurrence within the next five years. Median survival of patients with untreated metastases is on the order of three to 10 months, with only 20% surviving past one year. Overall survival is significantly improved with surgical margins greater than 1 cm with decreased survival in patients with positive margins or margins less than 1 cm in size. The number of metastasis is a less consistent but statistically significant factor. Patients with four or more metastases have a poorer prognosis. As for the size of the metastasis, it is not a significant factor except that a larger total liver volume of metastasis requires a larger hepatic resection. Larger size may preclude adequate margins and indicate longer development of time with an increased likelihood of micrometastases. 131. Which of the following statement(s) is/are true concerning the treatment and prognosis of hepatocellular carcinoma? a. The fibrolamellar variant of hepatocellular carcinoma has a distinctly better prognosis than other forms of the disease b. Patients with untreated hepatocellular carcinoma rarely survive a year c. Multiagent chemotherapy is extremely effective in hepatocellular carcinoma and should be considered for the treatment for most patients d. Hepatic artery ligation or embolization has been demonstrated to be highly effective for hepatocellular carcinoma Answer: a, b Hepatocellular carcinoma has a tendency for local and vascular invasion. Untreated, patients have a dismal prognosis with a median survival of only three to four months after diagnosis with only rare survival beyond one year. The fibrolamellar variant of hepatocellular carcinoma has a distinctly better prognosis with a five year survival after resection of 50% to 60% versus 25% for other varieties. Surgical resection offers the only chance for cure for primary hepatic malignancy, and survival is better if tumors are small and asymptomatic. The role of orthotopic liver transplantation for hepatocellular carcinoma continues to evolve. For patients with severe hepatic dysfunction or larger or centrally located tumors or bilobar tumors, this may be the preferred approach. Extrahepatic disease including lymph node involvement is a contraindication to resection or transplantation. No single-or multiple-agent chemotherapy is particularly effective in treating hepatocellular carcinoma. The rationale for treating vascular liver tumors with arterial obstruction is the fact that the nutritional blood supply of these a. Chronic acalculous cholecystitis or gallbladder dyskinesia is seldom associated with classic biliary colic symptoms b. The most specific test for diagnosing gallbladder dyskinesia is CCK-enhanced cholescintigraphy with assessment of gallbladder ejection fraction c. An ejection fraction greater than 75% is considered abnormal and indicative of gallbladder dyskinesia d. Cholecystectomy is not indicated for chronic acalculous cholecystitis Answer: b Motility abnormalities of the gallbladder and cystic duct present with symptoms suggesting gallstones. The most common presentation for patients with gallbladder motility disorders such as chronic acalculous cholecystitis or gallbladder dyskinesia is recurrent biliary-type pain. Currently, the most specific test for diagnosing gallbladder dyskinesia is CCK-enhanced cholescintigraphy with assessment of gallbladder ejection fraction. CCK is infused intravenously 15 to 30 minutes after ejecting an analogue of 99MTC imminodiacetic acid and calculating the ejection fraction of the isotope by the contracting gallbladder. An ejection fraction of less than 35% is considered abnormal and cholecystectomy may be indicated. Most patients will have relief of symptoms following cholecystectomy. 135. The following statement(s) is/are true concerning the embryology of the biliary tree. a. The primordial anlagen of the liver and biliary tract arises from the entoderm b. Superior and inferior caudal buds form as the hepatic diverticulum develops c. The development of the liver is a separate process from that of the gallbladder and distal biliary tree d. The biliary tree develops in association with the dorsal pancreas Answer: a, b The primordial anlagen of the liver, extrahepatic bile ducts, gallbladder, and ventral part of the pancreas develop as a thickened area of entoderm on the ventral surface of the caudal portion of the foregut where it joins the midgut. Superior and inferior caudal buds form as the hepatic diverticulum grows out into the ventral mesogastrium. The solid mass of endodermal cells spreading with this cephalic bud forms the right and left lobes of the liver. The superior growth of the cranial portion of the hepatic diverticulum, which extends from the duodenum to the liver, results in the formation of the hepatic, common hepatic, and common bile ducts. The caudal portion of the hepatic diverticulum develops into the gallbladder and cystic duct. The common bile duct is attached to the ventral aspects of the duodenum and is in close contact with the ventral pancreatic bud. 136. The following statement(s) is/are true concerning biliary motor function. a. The contracted sphincter of Oddi impairs bile flow into the duodenum and directs it into the gallbladder b. In the postprandial state about 70% of hepatic bile flows into the gallbladder before reaching the duodenum c. During the interdigestive period, only a small fraction of gallbladder bile enters the duodenum d. Gallbladder emptying during fasting is associated with phase III of the interdigestive migrating motor complex (MMC) e. After a meal, the gallbladder empties by a steady tonic contraction thought to be due to release of endogenous motilin from the mucosa of the small intestine Answer: a, b, c As bile is secreted from the liver, it flows through the hepatic ducts into the common hepatic duct and continues through the common bile duct into the duodenum. With an intact and contracted sphincter of Oddi, bile flows directly into the gallbladder where it is concentrated and stored. In the postprandial state, about 70% of hepatic bile flows into the gallbladder before reaching the duodenum and entering the enterohepatic cycle. During the interdigestive phase, 90% of bile from the liver enters the gallbladder, while only a small fraction of gallbladder bile enters the duodenum. Gallbladder emptying during fasting is associated with phase II of the interdigestive migrating motor complex (MMC). Motilin may account for this stimulatory effect since plasma elevations of motilin seem to correlate with the onset of phase II waves. Following a meal, the gallbladder empties by a steady tonic contraction thought to be due to release of endogenous cholecystokinin (CCK) from the mucosa of the small intestine. 137. The following statement(s) is/are true concerning the anatomy of the gallbladder. a. The gallbladder lies between the right, left, and quadrate hepatic lobes or hepatic segments IV and V b. The cystic duct contains the spiral valve of Heister which serves an important valvular function for the gallbladder c. The cystic artery arises from the right hepatic artery in 95% of cases d. The cystic artery crosses anterior to the hepatic duct in the majority of cases Answer: a, c The gallbladder is a pear-shaped organ bound to a fossa on the right inferior surface of the liver located between the right, left, and quadrate hepatic lobes or hepatic segments IV and V. The gallbladder can be divided into four areas: the fundus, body, infundibulum, and neck. The body of the gallbladder extends from the fundus into the tapered portion, or neck, which curves backward and upward toward the transverse fissure of the liver and terminates in the cystic duct. The cystic duct lumen contains a thin mucosal septum, the spiral valve of Heister; the valve may make catheterization to the cystic duct difficult but does not have true valvular function. The arteries of the gallbladder are derived from the cystic branch of the hepatic artery which in 95% of cases originates from the right hepatic artery. From its origin, the cystic artery usually crosses behind the hepatic duct (84% of cases) but is sometimes anterior to that structure. The cystic artery proceeds to the neck of the gallbladder, where it divides into anterior and posterior divisions that supply the corresponding areas of the gallbladder. The cystic veins empty into the right branch of the portal vein indirectly into the liver. 138. The gallbladder plays an important role in altering bile composition by absorption and secretion. Which of the following statement(s) is/are true concerning this mucosal function? a. The absorption of water by the gallbladder can result in concentration of the solute components of bile from 2-to 10-fold b. Gallbladder mucosal absorption can occur by both active and passive mechanisms c. Cyclic adenosine monophosphate (cAMP) stimulates NaCl-coupled transport and may also influence tight junction permeability d. Secretory products of the gallbladder include bicarbonate and glycoproteins Answer: a, b, d The gallbladder rapidly absorbs water and solutes from bile and concentrates the solute components 2-to 10-fold. The gallbladder has an active mucosa and is able to absorb water and solutes against significant concentration gradients. Water absorption is linked to the transport of ions. The two major mechanisms of absorption are active and passive. In passive absorption, sodium and chloride enter the gallbladder epithelial cells because of electrochemical gradients. This results in an osmotic gradient, and water flows into the cell. Intracellular sodium is extruded across the basolateral membrane into the lateral intercellular spaces by active transport. The active transport of sodium against an electrochemical gradient is associated by an Na+-K+-ATPase pump. Cyclic AMP may inhibit NaCl-coupled transport and may also influence tight junction permeability. Other peptides, such as secretin, glucagon, and gastric inhibitory peptide, have been shown to inhibit absorption. Secretion by the gallbladder occurs either by inhibiting net ion and fluid absorption or with stimulation of bicarbonate secretory mechanisms. Gallbladder epithelium may also secrete mucin and nonmucin glycoproteins which may play a role in gallstone formation. 139. Abnormalities of the sphincter of Oddi have been recently recognized to cause symptoms which are referable to the biliary tree or pancreas. The following statement(s) is/are true concerning sphincter of Oddi motor function. a. The sphincter’s basal resting pressure is 10 to 15 mm Hg above duodenal pressure b. Contraction of the sphincter occurs with CCK stimulation c. Vagal stimulation results in relaxation of the sphincter d. Manometry of the sphincter of Oddi may be performed at the time of ERCP to characterize basal pressure, amplitude, frequency of contraction, and direction of propagation of contractile waves e. Stenosis of the sphincter of Oddi is characterized by abnormally elevated basal pressure on sphincter of Oddi manometrics Answer: a, c, d, e The sphincter of Oddi is about 4 to 6 mm in length. The sphincter’s basal resting pressure is about 13 mm Hg above duodenal pressure. The sphincter exhibits phasic contractions at a frequency of 4 per minute and a duration of 8 seconds. The regulation of bile flow is primarily controlled by the sphincter and not by the surrounding smooth muscle of the duodenum. Relaxation of the sphincter occurs with CCK stimulation leading to diminished amplitude of phasic contractions and reduced basal pressure, allowing increased passive flow of bile into the duodenum. Parasympathetic stimulation also causes intermittent relaxation of the sphincter, and sympathetic splanchnic stimulation causes increased pressure. Abnormalities of the sphincter of Oddi may cause symptoms which are referable to the biliary tree or pancreas. Manometry of the sphincter of Oddi may be performed at the time of ERCP to characterize its basal pressure, the amplitude and frequency of contractions, and the direction of propagation of contractile waves. Stenosis of the sphincter of Oddi is characterized by abnormally elevated basal pressure (> 40 mm Hg) whereas dyskinesia is characterized by abnormalities of other manometric parameters. tumors comes primarily from the hepatic artery. Hepatic artery ligation or embolization alone or in combination with regional chemotherapy has been shown to be ineffective for liver cancer. In addition, hepatic artery ligation has a high complication rate and cannot often be used in patients with compromised liver function from cirrhosis. 132. The following statement(s) is/are true concerning the relationship of the biliary tree and the hepatic artery and portal vein. a. The common hepatic and common bile duct lie immediately anterior to the portal vein b. The cystic artery, which usually arises from the right hepatic artery, crosses behind the hepatic duct in most cases c. A replaced right hepatic artery arising from the superior mesenteric artery system will run to the right of the common bile duct d. The arterial supply of the extrahepatic biliary ducts is derived from major trunks running along the medial and lateral walls of the common duct at the 3 o’clock and 9 o’clock position Answer: a, b, c, d The common hepatic duct passes downward in the superior and lateral portions of the hepatoduodenal ligament and lies in front of the portal vein and to the right of the hepatic artery. The cystic artery which in most cases arises from the right hepatic artery usually crosses behind the hepatic duct (84%) of cases but is sometimes anterior to that structure (16%) of cases. A number of anatomic variations of the arterial supply of the liver can be seen. In patients in which the right hepatic artery arises from the superior mesenteric artery system, the "replaced" right hepatic artery usually runs to the right of the bile duct and portal vein. The arteries to the extrahepatic biliary ducts anastomose freely within the duct walls. The ductal arterial supply is derived primarily from the gastroduodenal and right hepatic arteries with major trunks running along the medial and lateral walls of the common duct at the 3 o’clock and 9 o’clock positions. 133. An understanding of the anatomy of the extrahepatic biliary tree is essential in performing biliary tract surgery. Which of the following statement(s) is/are true concerning biliary ductal anatomy? a. The majority of patients have the "classic" anatomical description b. The common hepatic duct unites with the cystic duct to form the common bile duct c. An accessory right hepatic duct occurs in 5% of patients d. A common channel or "Y" configuration of the distal bile duct and pancreatic ducts occur in approximately 70% of patients Answer: b, c, d The classic anatomical description of the extrahepatic bile ducts and their arteries is present in only one-third of individuals. The left hepatic duct usually has a longer extrahepatic course than does the right hepatic duct. The common hepatic duct is formed by the union of the right and left hepatic ducts close to the emergence from the liver. The duct passes downward in the superior portion of the hepatoduodenal ligament and lies in front of the portal vein and to the right of the hepatic artery. The common hepatic duct unites with the cystic duct to form the common bile duct. An accessory right hepatic duct occurs in 5% of cases. The cystic duct passes downward, backward, and to the left in the hepatoduodenal ligament and usually unites with the main hepatic duct at an acute angle. Its course and mode of insertion into the common duct is highly variable. The common bile duct is formed by the union of the common hepatic and cystic ducts and is usually about 7 to 9 cm long. The junction of the distal common bile duct and pancreatic duct at the ampulla can take one of three configurations. In approximately 70% of patients, there is a common channel of the bile and pancreatic ducts, thus a "Y" configuration. In approximately 20%, the common channel is nonexistent, and in another 10%, the two ducts enter the duodenum via separate openings. 134. A 35-year-old female presents with typical biliary colic symptoms, however her sonogram shows no gallstones. Which of the following statement(s) is/are true concerning her diagnoses?




140. A 32-year-old woman with symptomatic gallstones wishes to discuss nonsurgical options for her gallstones. Which of the following statement(s) are true?


a. The best commercially available oral dissolution agent, ursodeoxycholic acid, is associated with a complete dissolution rate of less than 50%
b. If the gallstones dissolve, there is minimal risk of gallstone recurrence
c. Contact dissolution is applicable regardless of stone type
d. Extracorporial shock wave lithotripsy (ESWL) in combination with oral dissolution agents is an appropriate technique for most patients and can result in complete stone fragment clearance in over 90% of patients by one year
Answer: a


Prior to the introduction of laparoscopic cholecystectomy, there was tremendous enthusiasm for a number of nonsurgical techniques for managing gallstones. Dissolution of existing gallstones with pharmacologic agents has been addressed by several national cooperative studies. Ursodeoxycholic acid is the best, safest, and most effective commerically available drug currently available. However, it still has a rate of complete dissolution of only 40%. It is also estimated that only 10% of patients will be suitable candidates for this therapy. Furthermore, gallstone recurrence is a major problem. Actuarial life table analysis indicates that the risk of gallstone recurrence in patients who have undergone dissolution of gallstones with oral bile acid therapy is 50% by five years.
The addition of extracorporial shock wave lithotripsy increases the efficiency of gallstone clearance and in selected patients complete fragment clearance can be obtained in over 90% of patients by one year. However, optimal results can be obtained only by setting relatively strict criteria for inclusion. In applying such criteria, less than 20% of patients in the United States would be considered eligible for ESWL. Similar problems with gallstone recurrence have been observed with this technique. Finally, contact dissolution primarily with the ether solvent methyl tert-butyl ether (MTBE) is extremely effective in dissolving cholesterol gallstones. This technique, however, will work only in stones which are composed of cholesterol and the patient must have a demonstrably patent cystic duct before considering this treatment. As with oral dissolution and lithotripsy, gallstone recurrence will remain a problem with this technique.




141. A 48-year-old woman presents with several hours of acute right upper quadrant pain, low grade fever, and nausea and vomiting. Which of the following statement(s) is/are true concerning her diagnosis and management?


a. A mild elevation of her bilirubin (< 3 mg/dl) would strongly suggest a common bile duct stone b. A positive bile culture can be expected in virtually 100% of patients with this scenario c. Laparoscopic cholecystectomy is clearly contraindicated d. Appropriate antibiotic coverage should include coverage for gram-negative aerobes Answer: d Acute cholecystitis occurs in about 10% to 20% of patients with symptomatic gallstones. Although the clinical manifestations of biliary colic and acute cholecystitis may overlap and clinical distinction is often difficult, persistence of pain beyond a matter of hours and fever would strongly suggest acute cholecystitis. The primary events in the development of acute cholecystitis are chemical in nature with bacterial infection playing a minor role in the genesis of the disease. In normal healthy subjects without gallstones, incidence of positive bile cultures is essentially zero. In contrast, between 30 and 70% of patients with the clinical diagnosis of acute cholecystitis will have positive bile cultures. The incidence of positive bile cultures who undergo cholecystectomy increases significantly with age. Septic complications continue to be a significant source of morbidity after cholecystectomy for acute cholecystitis. These septic complications can best be prevented by the judicious use of appropriate antimicrobial agents. The goal of antimicrobial therapy should be establishment of adequate serum and tissue levels of antibiotic rather than selection of an antibiotic that is excreted into the bile. Given the bacteriology that is typical in patients with uncomplicated cholecystitis, an appropriate antibiotic regimen should provide for adequate coverage of gram-negative aerobes. Although technically more difficult, laparoscopic cholecystectomy can be completed safely in the majority of patients with acute cholecystitis. Significant experience and good judgment, however, is essential in insuring optimal results. Laboratory data are often nonspecific with acute cholecystitis. Mild jaundice may be present in up to 20% of patients and is typically due to inflammation as opposed to bile duct obstruction secondary to stones. 142. Laparoscopic cholecystectomy has become the procedure of choice for the management of symptomatic gallstones. Which of the following statement(s) is/are true concerning laparoscopic cholecystectomy? a. Injury to biliary tree occurs more frequently with laparoscopic cholecystectomy than open cholecystectomy b. Laparoscopic cholecystectomy is contraindicated in patients with acute cholecystitis c. Bile duct injuries are more likely to occur during the surgeon’s early operative experience with the procedure d. Previous upper abdominal surgery is an absolute contraindication to laparoscopic cholecystectomy Answer: a, c Laparoscopic cholecystectomy was first reported in 1988 and in recent years has supplanted open cholecystectomy as the treatment of choice in the management of calculous biliary tract disease. As experience with the technology increases, recommendations regarding contraindications for laparoscopic cholecystectomy have evolved. Absolute contraindications include the inability to perform laparoscopic cholecystectomy because of inadequate training or equipment, poor candidate for general anesthesia, uncontrolled coagulopathy, peritonitis, or suspected gallbladder carcinoma. Factors previously considered relative contraindications have been identified but with appropriate skill and judgment do not preclude the laparoscopic technique. These include acute cholecystitis, morbid obesity, previous upper abdominal surgery, cirrhosis, portal hypertension, and pregnancy. Laparoscopic cholecystectomy for acute cholecystitis is clearly more difficult and while the conversion rate of elective laparoscopic cholecystectomy to an open procedure is less than 5% in the setting of acute cholecystitis, the rate is approximately 20–30%. The incidence of bile duct injury in large surveys in this country and abroad suggest that the incidence of bile duct injury with open cholecystectomy is less than 0.2%. Initially much higher, (approaching 1%), several series of laparoscopic cholecystectomy have documented that injuries to the bile ducts currently occur at the rate of 0.2–0.6% of cases. There is a definite learning curve associated with performing laparoscopic cholecystectomy with most bile duct injuries likely to occur during a surgeon’s early operative experience with this procedure. 143. Which of the following statement(s) is/are true concerning gallstone ileus? a. The diagnosis may be suggested by plain abdominal radiograph b. Primary surgical management consists of relief of obstruction and cholecystectomy c. Gallstone ileus accounts for less than 5% of all causes of intestinal obstruction d. Typical patients are elderly with long-standing gallstone disease Answer: a, c, d Cholecystoenteric fistulas are the most common form of gallbladder perforation accounting for 40% of all cases. Depending on the size of the fistulous communication, a gallstone may pass through this tract. If the stone is large enough (> 2 cm), it may become lodged in a portion of the gastrointestinal tract and cause mechanical small bowel obstruction. This condition, called gallstone ileus, is relatively rare and accounts for fewer than 5% of cases of intestinal obstruction. The diagnosis of gallstone ileus may be suggested by the presence of intrahepatic biliary air in the setting of a small bowel obstruction seen on abdominal radiography. Patients with gallstone ileus are best managed as if they had a mechanical small bowel obstruction with aggressive fluid resuscitation, broad spectrum antibiotics, and early laparotomy. The primary goals at laparotomy are correction of the obstruction and removal of the offending gallstone. Since many of the patients are elderly and ill, cholecystectomy and takedown of the biliary enteric fistula may not be appropriate. Enterolithotomy alone, without cholecystectomy, has an associated mortality of 5% in contrast to 15% mortality rate in patients who undergo both procedures at the same time.




144. The pathogenesis of cholesterol gallstones is multifactorial. A number of key processes, however, appear to interact closely in the formation of cholesterol gallstones. These include:


a. Cholesterol supersaturation
b. Gallbladder stasis
c. Accelerated nucleation
d. High bilirubin concentration
Answer: a, b, c


The development of gallstones is a complex multifactorial process. Although a number of factors may contribute to gallstone formation, a number of steps are considered critical in the formation of gallstones. These include cholesterol supersaturation whereby the amount of cholesterol present is in excess of the ability of the micelles and vesicles to transport cholesterol in a soluble state. In addition, the process by which cholesterol crystals form and agglomerate is referred to as nucleation. It is likely that the process of nucleation is altered by a number of biliary proteins or other compounds which may promote or inhibit the nucleation process. Gallbladder stasis provides the proper environment for gallbladder crystal precipitation and stone formation. The stagnant pool of bile within the gallbladder may facilitate both cholesterol supersaturation, the nucleation process, as well as altering the gallbladder’s absorptive or secretory functions. Although bilirubin has been suggested to play a small role in cholesterol gallstone formation, perhaps in either nucleation or solubilization of cholesterol, excessive production of bilirubin as seen in hemolytic disorders is associated primarily with the formation of pigment gallstones.




145. Which of the following statement(s). is/are true concerning the diagnosis of biliary tract disease?


a. Nonvisualization of the gallbladder on oral cholecystogram is diagnostic of biliary calculous disease
b. Ultrasonography has a diagnostic accuracy and sensitivity for cholelithiasis in excess of 95%
c. Ultrasonography is the preferred test to distinguish chronic from acute cholecystitis
d. Hepatobiliary scintigraphy is primarily indicated to confirm the clinical diagnosis of acute cholecystitis
Answer: b, d


A number of radiographic studies can be used to diagnose cholelithiasis. Visualization of gallstones on plain abdominal radiographs is possible in about 20% of patients whose stones are grossly calcified. Traditionally, oral cholecystogram has been the gold standard for the diagnostic evaluation of patients with suspected gallstones. Although the accuracy for oral cholecystography has been reported to be as high as 95%, several conditions preclude satisfactory examination, including acute illness; poor patient compliance; inability to absorb the tablets as the result of emesis, malabsorption, or diarrhea; and jaundice or hepatic dysfunction. Abdominal ultrasonography has therefore become the preferred test for evaluation of patients with suspected gallstones. Most large series suggest that diagnostic accuracy and sensitivity for cholelithiasis exceeds 95%. In addition to detecting gallstones, information is provided concerning the size and shape of the gallbladder, gallbladder wall thickness, presence of pericholcystic fluid, or a sonographic Murphy’s sign. The latter has been suggested to have an 85% accuracy rate in patients with acute cholecystitis. Nonetheless, while abdominal sonography is most helpful in identifying the presence or absence of gallstones, it is of limited use in distinguishing chronic from acute cholecystitis. Hepatobiliary scintigraphy provides information on the patency of the bile ducts, including the cystic duct. Recognition that cystic duct obstruction is the sine qua non of acute cholecystitis, failure of the gallbladder to visualize during hepatobiliary scintigraphy is highly suggestive of acute cholecystitis. Although a number of false positive exams can occur, nonvisualization of the gallbladder during hepatobiliary scanning in the appropriate clinical setting is highly diagnostic of acute cholecystitis.




146. Which of the following statement(s) is/are true concerning the solubilization of cholesterol in bile?


a. Cholesterol is highly soluble in both serum and bile
b. Mixed micelles are the primary transport mechanism for biliary cholesterol
c. Most cholesterol found in bile is the result of excretion from serum
d. Biliary vesicles are composed primarily of biliary phospholipid
Answer: d


Bile is secreted by the liver and is composed primarily of water, electrolytes, and organic solutes. Bile salts, cholesterol, and phospholipids are the main solutes found within bile and account for about 80% of the dry weight of bile. Most of the cholesterol found in bile is synthesized de novo in the liver. Cholesterol is an organic molecule that is virtually insoluble in an aqueous medium such as bile. Therefore, mechanisms for maintaining cholesterol in solution have been evolved. For decades, the mixed micelle which is composed of the amphiphatic bile salts and phospholipid was considered the primary carrier of bile. More recently, it has been demonstrated that up to 70% of the total amount of cholesterol normally found in gallbladder bile is transported and solubilized in the vesicular form. Bile vesicles are composed primarily of phospholipid of which in the human, lecithin accounts of 90% of the phospholipid content.




147. Appropriate options for management of common bile duct stones identified at laparoscopic cholecystectomy include:


a. Conversion to open cholecystectomy and common duct exploration
b. Transcystic duct dilatation and exploration
c. Laparoscopic choledochotomy
d. Complete the laparoscopic cholecystectomy with postoperative ERCP and stone removal
Answer: a, b, d


Common bile duct stones found unexpectedly during laparoscopic procedures presents the surgeon with a different set of choices from that of stones found during an open procedure where one would invariably proceed to common duct exploration. The practice of converting to an open procedure for the purposes of common duct exploration is acceptable and should be considered based on the surgeon’s experience and expertise and other options available. Transcystic duct dilatation and exploration with stone removal is being performed in increasing numbers and appears to be safe and effective. Although experience with laparoscopic choledochotomy and common duct exploration has been reported with limited experience, this technique is probably not appropriate for the average surgeon. Many surgeons, depending on their clinical situation, may opt to complete the laparoscopic removal of the gallbladder and proceed to postoperative management of the stone using endoscopic technique.




148. Risk factors associated with development of gallstones include:


a. Increasing age
b. Obesity
c. Rapid weight loss
d. Cirrhosis
e. Diabetes mellitus
Answer: a, b, c, d, e


Incidence of gallstones varies widely throughout the world} In the United States, about 10% of the population has cholelithiasis. Although the physiologic explanation for increasing incidence of gallstone disease in the elderly is unclear, epidemiologic studies have demonstrated a relationship between increasing age and the prevalence of cholelithiasis in both males and females. Gallstone disease is a much more common in women than men, however with the higher incidence of cholelithiasis in women persisting until the seventh or eighth decade of life, at which time the incidence approaches 20% in both men and women. Clinical evidence suggests a two to three-fold increase in the incidence of cholelithiasis among morbidly obese patients compared to age matched normal weight subjects. Interestingly, rapid weight loss has also been associated with secretion of cholesterol saturated bile and increased incidence of gallstones. Patients with diabetes mellitus have a two-fold increase of gallstone disease when compared to non-diabetics. Although the influence of alcohol on gallstone formation remains poorly defined, autopsy studies suggests that the incidence of cholelithiasis is significantly increased in patients with alcoholic cirrhosis.




149. Which of the following statement(s) is/are true concerning a 35-year-old woman found to have asymptomatic gallstones?


a. The patient should undergo attempts at medical dissolution
b. Cholecystectomy should only be performed if the laparoscopic technique is an available option
c. The patient has less than 10% chance of developing significant symptoms over the next five years
d. The risk of gallbladder cancer with stones detected at such a young age, warrants cholecystectomy
Answer: c


The optimal management of individual patients with asymptomatic gallstones has been debated for years. Most evidence, although still somewhat controversial, would indicate that the natural history of asymptomatic gallstones is benign and that early or prophylactic cholecystectomy, either laparoscopic or otherwise, is rarely indicated. It is suggested that less than 10% of patients with asymptomatic gallstones will develop significant symptoms over a five year period. Similarly, medical dissolution with oral agents, extracorporial biliary lithotripsy, or contact dissolution, is not indicated in these patients. The risk of gallbladder cancer is so low as to not warrant cholecystectomy in asymptomatic patients.




150. In which of the following clinical situations is pigment rather than cholesterol gallstones a more common problem?


a. Alcoholic cirrhosis
b. Morbid obesity
c. Diabetes mellitus
d. Hereditary spherocytosis
e. Biliary tract infection
Answer: a, d, e


World wide, pigment stones are the most common type of calculus found in the gallbladder. In the United States, although cholesterol calculi predominate, pigment stones constitute about 30% of all gallbladder stones. Pigment stones are characterized by a high concentration of bilirubin and a low cholesterol content. The high incidence of pigment gallstones in patients with hemolytic disorders probably results from the excessive loads of bilirubin being presented to the liver for excretion. Infection has been thought to be a key factor in the pathogenesis of pigment gallstones in that unconjugated bilirubin, produced by bacterial deconjugation, is insoluble in water, and combines with calcium in bile to produce a calcium bilirubin matrix. Specific alterations in bile acid metabolism have been proposed to explain the increased incidence of pigment gallstones found in cirrhotics. Gallstones associated with morbid obesity and rapid weight loss appear to be primarily due to the increased hepatic secretion of cholesterol leading to a cholithogenic state.




151. The following statement(s) is/are true concerning the mode of spread of gallbladder cancer.


a. The most common mode of spread for gallbladder cancer is to the liver by hematogenous spread
b. Lymphatic drainage of the gallbladder can extend from the cystic duct lymph node to periportal lymph nodes and the celiac and superior mesenteric lymph nodes
c. Gallbladder cancers rarely extend past the cystic duct
d. Direct extension to the liver, particularly liver segments IV and V, is the most common mode of spread for gallbladder cancer
e. Most patients with gallbladder cancer present with a very limited extent of disease
Answer: b, d


The mode of spread of gallbladder cancer can be predicted by the lymphatic and venous drainage of the gallbladder. Venous drainage is into the venules that drain directly into the adjacent liver. The most common spread of gallbladder cancer is through direct extension into the liver, particularly liver segments IV and V. The lymphatic drainage of the gallbladder is to the cystic duct lymph node, to periportal lymph nodes, and then to celiac and superior mesenteric lymph nodes. These tumors can also spread into and around the cystic duct and may extend into the common bile duct, causing biliary obstruction. Besides direct extension and lymphatic spread of the tumor, distant metastases is possible but is less commonly seen. A staging system has been developed which takes into consideration lymphatic and venous drainage of the gallbladder. The most advanced stage, stage V, includes distant spread, either into the liver or adjacent organs, which unfortunately is the case at the time of presentation for most patients.




152. A 55-year-old woman undergoes cholecystectomy for symptomatic gallstones. The final pathology returns one week after the operation showing gallbladder carcinoma. Which of the following statement(s) is/are true concerning this patient’s prognosis and management?


a. If the gallbladder carcinoma is limited to the mucosa and/or submucosa, cholecystectomy alone is adequate treatment
b. If the carcinoma involves deeper layers of the gallbladder wall, the prognosis will be significantly worse
c. If reoperation is performed, the chance of finding residual disease will be low
d. If a full thickness tumor had been recognized at the time of cholecystectomy, an "extended cholecystectomy" may have been appropriate
Answer: a, b, d


Gallbladder cancers with the best prognosis are those that are found incidentally at the time of cholecystectomy for symptomatic gallstone disease. This point emphasizes the importance of opening all gallbladders at the time of cholecystectomy so that any suspicious lesions can be immediately examined histologically. When gallbladder cancer is limited to mucosa and submucosa, cholecystectomy is adequate treatment and has a good prognosis, with up to 100% five year survival. When the cancer involves the deeper layers of the gallbladder wall, the prognosis is more grim. A 64% five year survival was reported in patients who had carcinoma confined to the mucosa and submucosa, whereas none of the patients with cancer involving all layers of the gallbladder wall survived more than 2.5 years. Even though these tumors are relatively localized at the time of cholecystectomy, cholecystectomy alone was not adequate treatment for long-term survival. In hopes of improving these survival rates and considering the lymphatic and venous drainage of the gallbladder, it has been recommended that gallbladder cancers be treated by cholecystectomy with wide resection of the liver around the gallbladder bed (liver segments IV and V) and regional lymphadenectomy. This procedure has been termed an "extended cholecystectomy." Although this treatment has prolonged median survival, an overall improvement in five year survival has not been observed. When a gallbladder cancer has been recognized incidentally at the time of pathology examination after routine cholecystectomy, reexploration will likely show evidence of residual disease in the lymph nodes or adjacent liver and therefore resection of this residual disease may improve survival. More radical excision including hepatic resection and bile duct resection and sometimes even pancreaticoduodenectomy has been reported. However, the associated increased morbidity and mortality of these operations has not resulted in an improvement in survival.




153. Gallbladder cancer accounts for 3% to 4% of all gastrointestinal tumors. Which of the following statement(s) is/are true concerning the incidence of gallbladder cancer?


a. Gallbladder cancer is much more frequent in men
b. Southwest Native Americans, Alaskans, Mexicans, and Hispanics have a greater incidence of gallbladder cancer than the general population
c. The majority of patients with gallbladder cancer have gallstones
d. The association of gallstones with gallbladder cancer is greater in patients with multiple small stones
Answer: b, c


Gallbladder cancer accounts for 3% to 4% of all gastrointestinal tract tumors and about 2% of all biliary procedures are done for gallbladder cancers. Gallbladder cancer is much more frequent in women, with a female/male ratio of 3:1. The disease is most commonly seen in elderly women with a mean age of 65 at diagnosis. Southwest Native Americans, Alaskans, Mexicans, and Hispanics living in the United States have an estimated five to six times greater incidence of gallbladder cancer than the general population. On the other hand, gallbladder cancer is much less common in blacks. The association of gallstones with gallbladder cancer is well known. Seventy to ninety percent of all patients with gallbladder cancers have gallstones, and about 0.4% of all patients with gallstones have gallbladder cancer. The association of gallstones with cancer may be related to gallstone size; larger stones have a greater cancer risk. There is a 10-fold increase in the incidence of gallbladder cancer in patients who have gallstones that are larger than 3 cm in diameter.




154. The following statement(s) is/are true concerning the prognosis for gallbladder cancer.


a. Average survival is in the range of one year
b. Five year survival rates approach 50%
c. The combination of postoperative adjuvant radiation and chemotherapy have been associated with overall improved survival
d. For most patients, the goal of treatment is palliation
Answer: d


The prognosis for gallbladder cancer remains poor, with an average survival in the range of six months. Less than 5% of patients survive five years because 90% of gallbladder cancer patients present with stage V disease. For stage V disease, the goal of treatment is palliation. If these patients present with obstructive jaundice, a major goal of treatment is relief of jaundice and its attendant symptoms such as pruritus and cholangitis. Although radiation and chemotherapeutic regimens have been tried, none have been associated with a good response.




155. Which of the following statement(s) is/are true concerning gallbladder polyps?


a. Sonographic findings of a gallbladder polyp include a filling defect which does not move with change in position
b. Benign gallbladder polyps can cause symptoms similar to those caused by gallstones
c. Ultrasonography can readily distinguish benign from malignant gallbladder lesions
d. An asymptomatic gallbladder polyp 1 cm in size can usually be followed with serial ultrasounds and does not require cholecystectomy
Answer: a, b


Benign gallbladder polyps cause symptoms similar to those caused by cholelithiasis. Biliary symptoms including right upper quadrant pain and discomfort, fatty food intolerance, nausea, vomiting, and an increase in flatulence are common complaints in patients with symptomatic benign gallbladder polyps. Gallbladder tumors, when diagnosed preoperatively, may be visualized by ultrasound, oral cholescintography, or less commonly, computed tomography. A filling defect that does not move with changes in position is probably a benign or malignant gallbladder polyp. Symptomatic patients who have lesions in the gallbladder should have a cholecystectomy. Since neither ultrasound nor cholecystography can distinguish benign from malignant lesions, all gallbladders that contain polypoid lesions should be removed.




156. The following statement(s) is/are true concerning the surgical therapy of cholangiocarcinomas.


a. A resectable distal bile duct carcinoma is best treated by a Whipple procedure
b. Klatskin tumors may require hepatic resection in attempt for potential cure
c. Regardless of the surgical resection for proximal biliary tumors, stenting of the biliary anastomosis is important
d. Complete hepatic resection with hepatic transplantation has been associated with overall good results
Answer: a, b, c


The surgical treatment of a cholangiocarcinoma is dependent upon its location. For lesions of the middle third of the bile duct, resection of the bile duct tumor with reanastomosis is the procedure of choice when possible. For distal lesions, the Whipple procedure (pancreaticoduodenectomy) is often necessary. Overall, both middle and lower third lesions have a better prognosis than tumors in the hilum. The prognosis for patients with hilar bile duct cancer (Klatskin tumors) is extremely poor with mortality rates of 80% to 90% in five years. Most patients are unresectable at the time of diagnosis. Unless contraindicated for other reasons, however, surgical exploration should be performed in all patients whose tumors are potentially resectable. Hepatic lobectomy is indicated for potential cure if the bile duct lesion extends into the hepatic parenchyma. Regardless of the surgical therapy, stenting of the biliary anastomosis is important since postoperative strictures or recurrent tumors are common and long-standing stents allow for cholangiographic followup and for dilatation should strictures arise. These stents have also been used for placement of iridium seeds used for local irradiation. More aggressive surgical therapy including complete hepatic resection followed by hepatic transplantation have been performed. However, the results have been associated with a very high rate of recurrence in the post-transplantation period and therefore at the present time this treatment is not recommended.




157. The following statement(s) is/are true concerning carcinoma of the bile ducts.


a. Carcinoma of the bile ducts is more common than gallbladder carcinoma
b. Similar to gallbladder cancer, bile duct cancer is more common in females
c. Unlike gallbladder cancer, there is no association of bile duct cancer and gallstones
d. Ulcerative colitis is associated with an increased incidence of bile duct cancer
e. Choledochal cysts may be associated with the development of bile duct cancer
Answer: d, e


Cancer of the bile ducts is even less common than gallbladder carcinoma and is seen in 0.01% to 0.46% of all autopsies. Unlike gallbladder cancer, which is more frequent in females, males have a higher incidence of bile duct cancer. The average age range of diagnosis between is 50 and 70 years. Similar to gallbladder cancer, there is an association between bile duct cancer and gallstones. This association is, however, not as direct as for gallbladder cancer with only 25% to 57% of patients with bile duct cancer having gallstones. Biliary tract infection is associated with these tumors; for example, patients with Clonorchis sinensis infection and chronic typhoid carriers have a higher incidence of bile duct cancer than the general population. Choledochal cysts are also associated with bile duct cancer. Patients with ulcerative colitis have a marked increase incidence of bile duct tumors which tend to follow a more aggressive course.




158. Which of the following statement(s) is/are true concerning the association of gallstones with gallbladder cancer?


a. The association of gallbladder cancer with gallstones is such that young patients with asymptomatic gallstones should undergo routine cholecystectomy
b. Calcification of the wall of the gallbladder is associated with an increased incidence of gallbladder cancer
c. Larger gallstones are associated with a greater risk of gallbladder cancer than smaller gallstones
d. The 20 year risk of developing gallbladder carcinoma in patients in the general population with gallstones is approximately 1%
Answer: b, c


The association of gallstones with gallbladder cancer is well known. Seventy to ninety percent of patients with gallbladder cancer have gallstones. The association of gallstones with cancer may be related to gallstone size, with larger stones having a greater cancer risk. The 20-year risk of developing gallbladder carcinoma in patients with gallstones has been estimated to be in the range of 0.13% for the general population. It is evident that this small risk of developing gallbladder cancer in patients with gallstones, even over the long-term, does not warrant routine cholecystectomy in all patients with gallstones. Calcification of the wall of the gallbladder, the so-called porcelain gallbladder, is associated with a 25% to 60% incidence of gallbladder cancer. In the presence of a porcelain gallbladder, a cholecystectomy should be performed unless contraindicated for other reasons.




159. A 65-year-old man presents with obstructive jaundice. The patient’s workup begins with a CT scan. Which of the following statement(s) is/are true concerning his diagnosis?


a. A CT scan demonstrating intrahepatic biliary obstruction with a decompressed gallbladder and a nondilated extrahepatic biliary tree will be consistent with a Klatskin tumor
b. The presence of biliary obstruction seen on the CT scan requires further evaluation with invasive cholangiography either percutaneously or endoscopically
c. Percutaneous transhepatic cholangiography would be the preferred technique for a suspected proximal cholangiocarcinoma in that it will better visualize the proximal extent of the tumor in the biliary tree
d. The placement of a transhepatic biliary catheter can prove useful in surgical management of proximal bile duct cancers
e. There is little role for angiography in the evaluation of a patient with suspected cholangiocarcinoma
Answer: a, b, c, d


Jaundice is the most frequent presenting symptom in patients with cholangiocarcinoma, occurring in 90% of patients. The diagnosis begins with abdominal imaging to detect ductal dilatation. A CT appearance of a proximal cholangiocarcinoma or a Klatskin tumor will show a dilated intrahepatic biliary tree with a decompressed gallbladder and normal caliber distal bile duct. When biliary obstruction is present, further visualization of the biliary tree is required either through percutaneous transhepatic cholangiography (PTC) or endoscopic retrograde cholangiopancreatography (ERCP). PTC is preferred for more proximal lesions because ERCP may fail to adequately visualize the proximal portion of the biliary tree. For lower bile duct lesions, ERCP may be the preferred route. The management of hilar tumors can often be facilitated by the placement of transhepatic percutaneous catheters which can facilitate both resection, reconstruction, and may serve as a route for administration of local radiation therapy. These tumors are often found to be unresectable because of direct vascular invasion into the portal vein or hepatic arteries. Therefore, selective celiac angiography can be helpful preoperatively to determine if these major adjacent vascular structures are involved with the tumor.




160. Benign gallbladder and bile duct tumors are extremely rare. Which of the following statement(s) is/are true concerning benign biliary tumors?


a. Cholesterol polyps are due to foamy macrophages filled with cholesterol
b. Inflammatory polyps and cholesterol polyps are not considered to be premalignant
c. Gallbladder adenomas, like adenomas in other gastrointestinal organs, are considered premalignant
d. Most gallbladder adenomas are associated with gallstones
e. An adenomyoma is a mucosal lesion of the gallbladder not considered to be premalignant
Answer: a, b, c


Benign gallbladder and bile duct tumors are extremely rare and can include pseudotumors or hyperplastic conditions which are thought to result from inflammatory states, or adenomas, which are most likely premalignant lesions. Cholesterolosis, or a strawberry gallbladder, is manifested by yellow spots visible on the surface of the gallbladder mucosa. This proliferation of foamy macrophages filled with cholesterol in the lamina propria may also result in the formation of polyps called cholesterol polyps. These polyps are thought to result from disturbance in cholesterol metabolism. Other pseudotumors of the gallbladder are inflammatory polyps which are composed of vascular connective tissue stalk with a single layer columnar epithelial cells and have a chronic inflammatory infiltrate. These lesions are not considered to be premalignant and are felt to be the result of chronic inflammation. Adenomas with hyperplasia of the epithelial layer of the gallbladder, like adenomas in other gastrointestinal organs can be sessile or papillary. Carcinoma in situ has been reported in these lesions, and they are thought to be premalignant. The cause of adenomas of the gallbladder is unknown; but most are not associated with the presence of gallstones. An adenomyoma of the gallbladder is a rare intramural mass or nodule. This lesion is characterized by a proliferation of the mucosal epithelium and hypertrophy of the mucosal layers of the gallbladder. Histologically in addition to muscular hypertrophy, invagination of the epithelial mucosa between muscle layers is evidenced. These lesions are also not considered premalignant.




161. Which of the following statement(s) concerning bile duct strictures due to chronic pancreatitis is/are true?


a. Most patients present with progressive jaundice
b. Strictures are classically long and tapered involving the entire intrapancreatic bile duct
c. Patients may be asymptomatic and diagnosed only by persistent elevation of serum alkaline phosphatase
d. An excellent option for surgical management is choledochoduodenostomy
Answer: b, c, d


The clinical presentation of patients with common bile duct strictures secondary to chronic pancreatitis is variable. A large number of patients are asymptomatic with a diagnosis of bile duct strictures suggested only by abnormal liver function tests. The serum alkaline phosphatase appears to be the most sensitive laboratory finding and is elevated in over 80% of patients. Although in most cases, transient jaundice may occur, progressive jaundice is rare. Cholangiography will demonstrate a classic long smooth gradual tapering of the common bile duct throughout its entire intrapancreatic segment. Biliary reconstruction is the appropriate management for most patients. Choledochoduodenostomy is preferred by many surgeons because it does not divert bile from the duodenum, is technically easier to perform, and leaves the jejunum intact for any associated procedures that may be required for decompression of the obstructed gastrointestinal tract or pancreatic duct.




162. If a bile duct injury is suspected at laparoscopic cholecystectomy, appropriate management includes which of the following?


a. Conversion to open cholecystectomy and intraoperative cholangiography
b. Small ducts (< 3 mm) demonstrated by cholangiography to drain a single liver segment can be ligated
c. If the injured segment is greater than 1 cm. in length an end-to-end ductal anastomosis is the procedure of choice
d. Postoperative external drainage should be avoided
Answer: a, b


In many cases, proper initial management of a bile duct injury recognized at the time of cholecystectomy can avoid the development of a bile duct stricture. Unfortunately recognition of a bile duct injury is uncommon during either open or laparoscopic cholecystectomy. It must be emphasized that should bile leakage be noted or if "a typical" anatomy is encountered during laparoscopic cholecystectomy, early conversion to an open technique and prompt cholangiography is imperative. If a segment of accessory duct less than 3 mm has been injured, and cholangiography demonstrates segmental or sub-segmental drainage of the injured ductal system, simple ligation of the injured duct is indicated. If the injured duct is 4 mm or larger, however, it is likely to drain multiple hepatic segments or the entire right or left lobe and thus requires operative repair. If the injured segment of the bile duct is short (< 1 cm) and the two ends can be opposed without tension, an end-to-end anastomosis can be performed with placement of a T-tube through a separate choledochotomy either above or below the anastomosis. For proximal injuries, or if the injured segment of bile duct is greater than 1 cm in length, an end-to-end bile duct anastomosis should be avoided because of the excessive tension that usually exists in these situations. The use of Roux-en-Y jejunal limb is preferable for creation of the anastomosis. Regardless of the type of anastomosis, all repairs at the time of initial operation should involve some form of external drainage either with a T-tube or intraoperatively-placed transanastomotic stent.




163. Which of the following statement(s) is/are true concerning the incidence of bile duct injury following cholecystectomy?


a. Data from the pre-laparoscopic cholecystectomy era would suggest the incidence of bile duct injury during open cholecystectomy to be 0.1–0.2%
b. The current incidence of bile duct injury during laparoscopic cholecystectomy is greater than 1%
c. The experience of the surgeon performing laparoscopic cholecystectomy can be correlated with the incidence of bile duct injury
d. Intraoperative cholangiography during laparoscopic cholecystectomy will prevent bile duct injury in virtually all cases
Answer: a, c


A number of large surveys encompassing thousands of patients undergoing open cholecystectomy would suggest the incidence of bile duct injury to be 0.1–0.2%. Although a number of early individual series of laparoscopic cholecystectomy reported bile duct injuries in 1% of patients, as larger series have been reported and surveys including thousands of patients have appeared, the true incidence would appear to be 0.3–0.6%. A number of factors are associated with bile duct injury during laparoscopic cholecystectomy including the experience of the surgeon. This reflects the steep learning curve with this procedure. Although strongly debated, there is no evidence that intraoperative cholangiography prevents bile duct injury during laparoscopic cholecystectomy. The use of intraoperative cholangiography may however detect the injury early in the course of the procedure and thus minimize the extent of injury.




164. Primary sclerosing cholangitis has a number of treatment options—both medical and surgical. Which of the following statement(s) is/are true?


a. A number of immunosuppressive oral agents can provide specific effective treatment for primary sclerosing cholangitis
b. Biliary reconstruction with long-term transanastomotic stents can be useful in selected patients with focal strictures at the hepatic duct bifurcation
c. Biliary reconstruction should be reserved only for patients with established biliary cirrhosis
d. Hepatic transplantation for primary sclerosing cholangitis can be associated with survival rates similar to other indications for transplantation
Answer: b, d


There is no known specific, effective medical therapy for primary sclerosing cholangitis. Although encouraging results from a prospective, randomized, placebo-controlled trial suggests that ursodeoxycholic acid significantly improves serum liver function tests and clinical symptoms. Because of the lack of effective medical therapy, an aggressive surgical approach is indicated for most symptomatic patients with primary sclerosing cholangitis. One surgical approach, in patients with a predominant stricture at the hepatic duct bifurcation, is resection of the bifurcation and long-term transhepatic stenting with silastic stents. Results in patients without established cirrhosis are excellent. However, in those patients with secondary biliary cirrhosis present before surgery, perioperative morbidity and mortality have been high and long-term results poor. Patients with established secondary biliary cirrhosis should be referred for hepatic transplantation. Recent reviews of the experience with hepatic transplantation for primary sclerosing cholangitis would suggest survival to be similar to those reported for hepatic transplantation for patients with any diagnosis.




165. The following statement(s) regarding the elective repair of a bile duct stricture is/are true:


a. A transanastomotic stent is necessary for a successful result
b. Long-term stenting for approximately one year is necessary for an anastomosis performed at the distal common hepatic duct
c. A Roux-en-Y hepaticojejunostomy provides the best route for restoring biliary-enteric continuity
d. Preoperatively-placed biliary catheters facilitate dissection and identification of the stricture and are useful in placement of transanastomotic stents when employed
Answer: c, d


Several principles are associated with successful repair of a biliary stricture. Although many surgeons favor the use of transanastomotic stents, a number of series have reported successful results without the use of such stents. The length of stenting depends upon the location of the stricture. If the injury involves the common bile duct or common hepatic duct at least 2 cm distal to the hepatic duct bifurcation, and adequate proximal bile duct mucosa can be defined, the use of long-term biliary stents is not necessary. In these situations transanastomotic stenting for 4–6 weeks postoperatively is adequate. When adequate proximal bile duct is not available for a good mucosa-to-mucosa anastomosis, long-term stenting of the biliary-enteric anastomosis with silastic transanastomotic stents for at least a year is recommended. For established strictures, simple excision and end-to-end anastomosis or repair of the damaged duct can rarely be accomplished because of the invariable loss of duct length as a result of fibrosis associated with injury. Thus in almost all cases, hepaticojejunostomy constructed to a Roux-en-Y limb of jejunum is the preferred procedure. The use of preoperatively-placed transhepatic biliary catheters can aid in the dissection and identification of the biliary tree especially in patients with prior attempts at repair where scarring and fibrosis may be significant. In addition, the biliary catheters can assist in the placement of long-term transanastomotic stents.




166. The vast majority of benign bile duct strictures occur following operations in or near the right upper quadrant. Other causes of benign bile duct strictures include:


a. Chronic pancreatitis
b. Ulcerative colitis
c. Primary sclerosing cholangitis
d. Intrahepatic arterial infusion of 5-fluorouracil
Answer: a, c, d


Most bile duct strictures are postoperative in nature with over 80% occurring after injury to the bile duct during cholecystectomy. A number of inflammatory conditions can also result in strictures of the biliary tree. The chronic inflammation and fibrosis associated with chronic pancreatitis can result in a stricture of the intrapancreatic bile duct. Primary sclerosing cholangitis is an idiopathic disease believed to be autoimmune in nature which is characterized by intrahepatic and extrahepatic inflammatory strictures of the biliary tree. Although primary sclerosing cholangitis is frequently associated with ulcerative colitis, this colonic disease has no direct causal relationship with benign bile duct strictures. A rare cause of benign bile duct strictures in both the intrahepatic and extrahepatic biliary tree has been the use of intrahepatic arterial infusion of 5-fluorouracil used to treat hepatic metastasis from colorectal carcinoma.




167. A 37-year-old female presents with obstructive jaundice due to a mid-bile duct stricture four months after laparoscopic cholecystectomy. Which of the following statement(s) are true:


a. Surgical reconstruction is the only option for management of this patient
b. Excellent long-term results can be expected in approximately 80% of patients following surgical biliary reconstruction
c. One year follow-up after successful repair is satisfactory regardless of the method of management
d. Surgical reconstruction offers a better chance of long-term success than either percutaneous or endoscopic dilatation
Answer: b, d


Excellent long-term results can be achieved in 70–90% of patients who undergo surgical repair of bile duct strictures. The definition of satisfactory results in most series requires patients with no symptoms of jaundice or cholangitis. Length of follow-up is important in analyzing results, however, because recurrent strictures can occur up to 20 years after the initial procedure. About two-thirds of restrictures are evident within two years and 90% within seven years. Although operative management of bile duct strictures in most cases can result in excellent results, the nonoperative approaches of percutaneous or endoscopic dilatation are suitable alternatives in many patients. Although comparisons between techniques are difficult, two retrospective comparative studies from single institutions have suggested that surgical reconstruction offers a better chance of long-term success than either percutaneous or endoscopic management.




168. Most patients with postoperative bile duct strictures after cholecystectomy present early after their initial operation. Patients may present in which of the following manner(s)?


a. Obstructive jaundice
b. An external biliary fistula
c. Progressive accumulation of bile in the peritoneal cavity (bile ascites)
d. Biliary cirrhosis
Answer: a, b, c


Most patients with benign postoperative bile duct strictures present early after their initial operation. Those patients suspected of having postoperative bile duct stricture within days to one week of an initial operation usually present in one of two ways. One mode of presentation is the progressive elevation of liver function tests, particularly serum bilirubin and alkaline phosphatase. These changes can be seen as early as the second or third postoperative day. The second mode of early presentation is with leakage of bile from the injured bile duct. Bilious drainage from the operatively-placed drains or through the wound after cholecystectomy is abnormal and represents some form of biliary injury. In those patients without drains, or in whom the drains have been removed, the bile may leak into the peritoneal cavity as bile ascites or may loculate as a collection. Patients with markedly delayed diagnosis of bile duct stricture may present with advanced biliary cirrhosis and other evidence of liver dysfunction.




169. Nonoperative dilatation, performed either endoscopically or percutaneously, can be successfully employed in which of the following etiologies of bile duct strictures?


a. Postoperative bile duct strictures following a hepaticojejunostomy used for reconstruction during a Whipple procedure
b. Complete transection of the bile duct during laparoscopic cholecystectomy (the so-called "classic laparoscopic cholecystectomy injury")
c. Primary sclerosing cholangitis
d. Oriental cholangiohepatitis
Answer: a, c


Nonoperative management of bile duct strictures is an available option at most institutions, however, it has some technical limitations due to the anatomic situation. In the so-called "classic laparoscopic bile duct injury" however, complete bile duct transection and discontinuity of the biliary tree eliminates the possibility of nonoperative management. Percutaneous dilatation of biliary-enteric anastomosis has been shown in a number of series to have a success rate approaching that of surgical reconstruction. Although limited experience with either percutaneous or endoscopic dilatation in primary sclerosing cholangitis has been reported, this alternative may provide at least temporary improvement in symptoms and radiologic appearance. Oriental cholangiohepatitis is an unusual infection of the biliary tree frequently associated with Clonorchis sinensis and other parasites. Cholangiography will demonstrate multiple strictures of both the intrahepatic and extrahepatic biliary tree with bile ducts filled with sludge and stones. Surgical management consisting of cholecystectomy and improving biliary drainage with either a Roux-en-Y choledochojejunostomy or choledochoduodenostomy is necessary in almost all patients. Access to the biliary tree for postoperative management of intrahepatic stones or sludge should be maintained, however, with transhepatic biliary stents.




170. The gold standard for evaluation of patients with bile duct strictures is cholangiography. The two routes for cholangiography are percutaneous transhepatic cholangiography (PTC) or endoscopic retrograde cholangiography (ERC). Which of the following statement(s) is/are true?


a. PTC is generally more valuable than ERC in defining the proximal biliary tree to be used in reconstruction
b. ERC is technically easier in patients with bile leaks because the biliary tree is usually not dilated
c. Parenteral antibiotics should be administered prior to either procedure to prevent cholangitis
d. Biliary stents can be placed via either technique to control biliary leaks
Answer: a, b, c, d


The gold standard for evaluation of patients with a bile duct stricture is cholangiography. PTC is generally more valuable than ERC in that it defines the anatomy of the proximal biliary tree that is used in the surgical reconstruction. ERC is often less useful than PTC because the discontinuity of the extrahepatic biliary tree usually prevents adequate filling of the proximal biliary tree. However, in patients with biliary fistulas, the proximal biliary tree is often not dilated making PTC somewhat technically more challenging. Parenteral antibiotics should be administered prior to either procedure to decrease the risk of cholangitis. Biliary stents used either to temporarily control biliary leaks or to stent a stricture after nonoperative dilatation can be placed by either the percutaneous or endoscopic route.




171. The management of a suspected bile duct injury depends on a number of factors, most importantly the mode and timing of presentation. Which of the following statement(s) is/are true concerning a patient presenting with a suspected bile leak after laparoscopic cholecystectomy?


a. Laparotomy should be performed immediately
b. Cholangiography should be performed to determine the nature of the injury
c. Operatively-placed drains should be removed to allow the fistula to close
d. The patient should be discharged to home to allow the leak to close spontaneously
Answer: b


Patients presenting in the early postoperative period with a biliary leak may be septic with either cholangitis or intra-abdominal bile collections. Sepsis must be controlled first with broad-spectrum parenteral antibiotics, cholangiography with percutaneous biliary drainage and percutaneous or operative drainage of biliary leaks. Once sepsis is controlled, there is no hurry in proceeding with surgical reconstruction of the bile duct stricture. The combination of proximal biliary decompression and external drainage allows most biliary fistulas to be controlled or even closed. At that time the external drains may be removed. The patients can then be discharged to home to allow several months to lapse for resolution of the inflammation in the periportal region and recovery of overall health status.

10 comments:

  1. I decided to share this because i am so glad today and happy that i am alive to see another new day and not just that but also to share the goodnews of how i survived a deadly stage 4 cholangiocarsinoma (bile duct cancer). I was told by my oncologist that i had just 6 months left to live and i was so scared to lose my wife. I was lucky to contact Dr Mrs Aleta who i told all about it and she is the nicest person i have spoken to. She recommended a herbal medicine for her which she took that cured her in less than a month. Well for more info about the medicine and cancer treatment simply reach her on aletedwin@gmail.com she can help you too. Contact her for any form of cancer too.

    ReplyDelete
  2. AM SAMFOR, FROM ISLAND, THANKS TO DR ONIHA WHO CURED ME FROM liver chirosis and FATTY LIVER PROBLEM WITH HIS HERBAL MEDICATION. I WAS DIAGNOSED OF FATTY LIVER PROBLEEM FIVE YEARS AGO, I VISITED MEDICAL DIALYSIS CENTER, AND THERE WERE NO BETTER IMPROVEMENT, UNTIL I SAW DR ONIHA'S TESTIMONIES AND RECONMMENDATIONS ONLINE, ON HOW HE HAS BEEN CURING PEOPLE OF LIVER PROBLEMS, AND I DECIDED TO CONTACT HIM, AND HE ADMINISTERED HID DERBAL MEDICATION ON ME, WITHIN A MONTH, I WAS CURED. IN CASE YOU ARE PASSING THROUGH A SIMILAR PROBLEM AND YOU NEED DR ONIHA'S ASSISTANCE, YOU CAN CALL OR WHATSAP +2347089275769 OREMAIL: DRONIHASPELL@YAHOO.COM

    ReplyDelete
  3. I would like to you can share more details or information about Liver disease. Specially how herbal medicine are effects on liver treatment. Thanks

    ReplyDelete
  4. I was diagnosed of hepatitis E November last year, after serious fever, yellowing of the skin, dark urine, pain in the abdomen vomiting and fatigue. I was given some Ribavirin treatment at the hospital and it subsided but it came back after 3 months no hope of cure, my sister came across Dr. IYABIYE’s herbal treatment recommendation online and reached out to him for help I became totally cured after the treatment Contact: +2348072229413 or +2348158577300 email: iyabiyehealinghome@gmail.com

    ReplyDelete
  5. Hello, I want to write a little review about the good work of doctor Ojamo, who cured me from HSV 1&2 for just 2 week with his herbal medicine, I never believe I can be cured again and have a good life like others, I always regretted the day I got diagnose with this virus, I become lost of hope when my doctor told me there is no cure for it, Well I am so grateful for my helper who get me cured with his herbal medicine, I went online in search of anything that can help me because I can’t deal with it forever so I found dr Ojamo email dr.ojamoherbalhome@gmail.com on a blog of someone who was cured by him,of the same herpes, so I quickly contacted him for help and explained all my pains to him, he told me not to worry about it there is a cure for real, I never believe until he sent me the herbs when I ordered for it and I got it within 5 working days that is how I took the medicine for 2 weeks and the following week I went for a test just to confirm I was 100% cured from this sickness what a miracle in my life, I am so happy right now, and forever greatful, you can also get in contact with him if you have such sickness through his whatsapp +2349077406037 or email address:: dr.ojamoherbalhome@gmail.com

    ReplyDelete
  6. My husband was diagnosed with early onset Parkinson's disease at 57.his symptoms were shuffling of feet,slurred speech, low volume speech, degradation of hand writing, horrible driving skills, right arm held at 45 degree angle, things were tough for me, but now he finally free from the disease with the help of total cure ultimate health home, he now walks properly and all symptoms has reversed, he had trouble with balance especially at night, getting into the shower and exiting it is difficult,getting into bed is also another thing he finds impossible.we had to find a better solution for his condition which has really helped him a lot,the biggest helped we had was ultimatehealthhome they walked us through the proper steps,am highly recommended this ultimatehealthhome@gmail.com to anyone who needs help.

    ReplyDelete

  7. i was diagnosed of parkinson disease 5 years ago,i started azilect then mirapex as the disease progressed in frebuary last year,and i started on parkinson disease herbal treatment from Ultimate Life Clinic,few months into the treatment i made a significant recovery,almost all my symptoms are gone,great improvement with my movement and belance,it been a year and life has been so good for me,contact them through there website www.ultimatelifeclinic.com

    ReplyDelete
  8. DR EMU WHO HELP PEOPLE IN ANY TYPE OF LOTTERY NUMBERS
    It is a very hard situation when playing the lottery and never won, or keep winning low fund not up to 100 bucks, i have been a victim of such a tough life, the biggest fund i have ever won was 100 bucks, and i have been playing lottery for almost 12 years now, things suddenly change the moment i came across a secret online, a testimony of a spell caster called dr emu, who help people in any type of lottery numbers, i was not easily convinced, but i decided to give try, now i am a proud lottery winner with the help of dr emu, i won $1,000.0000.00 and i am making this known to every one out there who have been trying all day to win the lottery, believe me this is the only way to win the lottery.

    Dr Emu can also help you fix this issues

    (1)Ex back.
    (2)Herbal cure & Spiritual healing.
    (3)You want to be promoted in your office.
    (4)Pregnancy spell.
    (5)Win a court case.

    Contact him on email Emutemple@gmail.com
    What’s app +2347012841542
    Website Https://emutemple.wordpress.com/
    Facebook page Https://web.facebook.com/Emu-Temple-104891335203341

    ReplyDelete
  9. Thanks for sharing this article.
    Want to know about the most certified Liver Hospital In Ludhiana? If yes then click on this link and get full detailed Information

    ReplyDelete

  10. Nice post of knowledge of liver transplant treatments Meet Dr kartik Goyal
    for more liver diseases treatments.

    https://gastrogynae.com/best-liver-doctor-ludhiana/

    ReplyDelete

Related Posts Plugin for WordPress, Blogger...
Related Posts Plugin for WordPress, Blogger...